Sunteți pe pagina 1din 125

Intro to Tort Liability

Unintended Injury-Liability
Hamontree(/injured in accident) v. Jenner(/had seizure while driving): BLL: A sudden injury which renders a driver unconscious will not be grounds for an action in negligence or strict liability Facts: 1967: was driving, had an epileptic seizure, and ran into s store, causing injuries and damages s history: had seizures back in 1953 obtained medication to control them doctors testified that they thought he was OK to drive he had no reason to know that he would have a seizure that day and he could do nothing to stop it once it started sued for personal injuries and property damages jury ruled in favor of Issue: Is strict liability in a tort a proper theory to apply to sudden illnesses which render a driver unconscious? Ruling: NO s 2 claims: Absolute/Strict Liability Similar questions asked as in negligence, but starts out a little further down the path Used in product liability claims: A manufacturer or retailer is strictly liable in tort when an article he places on the market, knowing that it is to be used without inspection for defects, proves to have a defect that causes injury to a human being The wants to apply that theory to drivers: Saying that the driver was responsible, even if he did not anticipate a seizure, and he could do nothing to prevent it Negligence Dropped because the doctor had said that he was safe to drive while under medication The court cannot apply the strict liability standard to the driver Different than placing a product into the market Would only add confusion to claims adjustments Notes: Def: Negligence: Conduct falling below the standard of care that a reasonable person would demonstrate under similar conditions Strict Liability: Liability for all injuries proximately caused by a partys conducting of certain inherently dangerous activities without regard to negligence or fault Up until about 1960, negligence was one of the only tort claim used Economic advantages of him driving Can they be weighed against the damages incurred in the accident? Claim of Negligence: Where a driver has constructive or actual notice of an illness that might make driving dangerous, this may be negligence Where an outside force (swarm of bees enters car, heart attack) causes the accident, no liability would be found Procedure for Tort Claims: Would deny all the claims when given a complaint 12(b)(6) motion failure to state a claim for which relief can be granted assuming that all the plaintiffs claims are true, the will still win Discovery Emotional and mental background of each party

SJ Would probably just be the that would try to bring the motion If there is a genuine issue of material fact, it must go to trial Trial: The witnesses present the evidence Move for JNOV because there is no proof of damages The essential component of a s case Must base the evidence on a preponderance of the evidence standard (51/49)

: was she already distressed? : was he in a state to be driving depositions medical bills and damage assessments

Vicarious Liability
Christensen (/injured driver) v. Swensen et al (/employee and employer) : BLL: For an employer to be vicariously liable for the torts committed by the employee, the employee must have been acting within the scope of his employment Facts: was on guard duty they dont get any breaks, only 15 minutes to eat while at their post the only restaurant close enough is where they all go, and no one has ever told them differently the menu is posted at their post When the called and placed an order, went and picked it up, and was driving back, she hit the brought suit against the employee and employer alleged that the employer was vicariously liable for damages Burns (employer) moved for SJ on grounds that Swensen was not acting within the scope of employment when the accident occurred, and the company was therefore not liable for damages Trial court granted the motion Issue: For an employer to be vicariously liable for the torts committed by the employee, must the employee have been acting within the scope of his employment? Ruling: YES Doctrine of Respondeat Superior: Employees are vicariously liable for torts committed by employees while acting within the scope of their employment 3 Birkner Criteria in determining whether the employee was acting within the scope of his employment : 1) The employees conduct must be of the general kind the employee is hired to perform must be business purposes, not personal endeavors two disputed facts: 1) purpose of employment S claims that guards are supposed to be seen around the plant traveling in a uniform heightened security for the plant B claims that guards are not hired for that purpose 2) sanctions for getting lunch Burns never disciplined the guards for taking the lunch at the caf But, they allowed a menu to be posted on the phone reasonable minds could differ 2) The employees conduct must occur substantially within the hours and ordinary spatial boundaries of the employment Hours: it was definitely within the hours of work Spatial Boundary: Even though the guard was not on the property at the time of the accident, reasonable minds could differ as to whether the guard was within the ordinary spatial boundaries of her employment There was no specific orders barring guards from leaving the grounds with the vehicles, although managers said this was prohibited

3) the employees conduct must be motivated, at least in part, for the purpose of serving the employers interest fast breaks benefit the company in efficiency breaks make the employees more productive reasonable minds could differ

Notes: Going to work? Have to go Mere situs rule: When a biker is hit when someone opens his car door The will claim it is a car accident The suit arises out of the use of a car Def: Respondeat Superior: Rule that the principal is responsible for tortious acts committed by its agents in the scope of their agency or authority Vicarious Liability: The imputed liability of one party for the unlawful act of another Justifications for the doctrine: Fairness Business cannot disclaim responsibility for accidents which may fairly be said to be characteristic of its activities Economic: Gives employers strong incentives to screen candidates with care and provide proper training Gives employers strong incentives to discipline employees for negligence Gives employers strong incentives to consider alternatives to employee efforts Mechanics Less activity What would happen if the employee had a bad past? Court may consider that the employee should have taken more care in screening the employee during the hiring Baptist Memorial Hospital(hospital where physician was working) v. Sampson ( /patient): BLL: Ostensible (apparent) agency must be shown by demonstrating that the principal, by its conduct, caused the to believe that the putative agent was an employee or agent of the principal, and the reasonably relied on that appearance of agency. Facts: was bitten by a spider claims that the physicians treatment in the hospital caused her long-term injuries The physician was an independent contractor The had to sign a form agreeing that the Hospital would not be liable for the independent contractors There were signs posted all over saying the same The court must consider on appeal (from SJ) whether the raised a material issue of fact that the Hospital was vicariously liable under the theory of ostensible agency for an emergency room Drs negligence Issue: Must ostensible agency be shown by demonstrating that the principal, by its conduct, caused the to believe that the putative agent was an employee or agent of the principal, and the reasonably relied on that appearance of agency? (is the hospital vicariously liable?) Ruling: YES Different than the theory of Respondeat superior responsible for employee, not an agent An independent contractor has sole control over his work, the individual who hires that contractor is generally not vicariously liable unless it acted in a way to make it liable for someone who is not an agent at all, or someone who is who acted outside the scope of his authority Elements of ostensible Agency (from Rest. of Agency): 1) The principal, by its conduct Hospital took no affirmative action to make patients think that the physicians were employees It did not fail to make reasonable efforts to disabuse that notion 2) caused the to believe that the putative agent was an employee or agent of the principal, and

Notes: Def: Ostensible Agency: The apparent authority granted to an agent to act on behalf of the principal in order to effectuate the principals objective. An employer who employs an independent contractor to perform services which are accepted in the reasonable belief that services are being rendered by the employer or his agents, is subject to liability Ex: A court found that a shopping mall was guilty of a slippery walkway, even though it hired contractors to care for it. The walkway invited the public, and the mall can not escape liability simply by hiring independent contractors for everything Rest. of Torts: Only requires a reasonable belief, not an affirmative act (as in the Rest.of Agency) The belief can be created by someone else

No conduct by the Hospital would lead a reasonable person to believe that the physicians were employees 3) the reasonably relied on that appearance of agency?

Negeligence
N = 4 elements Duty breach causation is legal causation satisfied? The element that most of the cases are about damages n= breach of a duty

Fault Liability
Strict liability: liability without fault products liability v. absolute liability: there is no absolute liability too many things are associated with injury absolute: transfer the cost of a car accident (when hitting a pedestrian) onto the car manufacturer simply because it was associated with the accident hard to say that the car was absolutely liable could also involve negligence Brown v. Kendall: Fault as the standard of negligence liability BLL: If in the prosecution of a lawful act, a casualty purely accidental arises, ie., the injury was unavoidable, and the conduct from the was free from blame, no action can be supported from an injury arising therefrom. Facts: Two owners dogs were fighting. One owner started trying to separate them with a stick. The other moved in closer When the owner with the stick started backing up in response to the dogs, she hit the other owner in the eye, causing serious injury. Brown brought an action for trespass in assault and battery Issue: Whether the owner with the stick is liable for damages when it was purely accidental, and the other owner was not at fault Ruling: NO The facts preclude the fact that it was intentional assume unintentional must show that the s intent was unlawful, or that the was at fault if the act was lawful, and accidental, and the was not to blame, no action can result Ordinary Care: Depends on the case

has no case if: both and were using Ordinary care was using ordinary care and was not neither was s care: the intent was not unlawful. If done properly, not harmful It was not intentional It was accidental, and unavoidable Notes: Def: Duty: An obligation owed by on individual to another Assault and Battery: Any unlawful touching of another person without justification or excuse. Contributory Negligence: Behavior on the part of an injured falling below the standard of ordinary carethat contributes to the s negligence, resulting in the s injury. Comparative Negligence: Doctrine whereby the court, in assessing the appropriate amount of damages, compares the relative fault of the parties and reduces the amount of damages to be collected by the in proportion to his degree of fault Adams v. Bullock: Unreasonable Risk BLL: A party will not be deemed negligent if it has taken reasonable precautions to avoid predictable dangers. Facts: runs a trolley line which is crossed by a bridge. The side of the bridge was protected by an 18-inch wide parapet. Pedestrians often use the bridge as a short cut, and children often play on it , a 12-year old boy, while crossing the bridge, in swinging a wire about eight feet long, brought it in contact with 's trolley wire which was between four and five feet below the top of the parapet of the bridge was shocked and burned. Issue: Should a party be deemed negligent if it has taken reasonable precautions to avoid predictable dangers? Ruling: NO The in using an overhead trolley was in the lawful exercise of its franchise. Negligence, therefore, cannot be imputed to it because it used that system and not another. Duty to adopt all reasonable precautions: There is no evidence that this duty was ignored. No one standing on the bridge or even bending over the parapet could reach the wire. Only some extraordinary casualty, not fairly within the area of ordinary prevision, could make it a thing of danger. No vigilance, however alert, could have predicted the point upon the route where such an accident would occur. At any point upon the route, a mischievous or thoughtless boy might touch the wire with a metal pole, or fling another wire across it from the bridge or the ground No special danger at this bridge warned the defendant that there was need of special measures of precaution. No like accident had occurred before. No custom had been disregarded. Ordinary caution did not involve forethought of this extraordinary peril. Insulation of trolley wires is impossible. The would have had to put the wires underground not his power or duty To hold it liable upon the facts exhibited in this record would be to charge it as an insurer. Holding: There was no evidence that defendant had failed in its duty to adopt reasonable precautions against injury from the wire. Hence a recovery by plaintiff cannot be sustained. Notes: The amount of care needed to sustain ordinary care will depend on the situation Basically, the owner should not ordinarily be held to be the insurer of all persons with whom his actions bring him into contact

Ex: What would happen if someone left wires uninspected after 25 years in an empty city lot, and someone started building on it and was shocked? Using reasonable care, should the person who put the wire there anticipated that this might have happened? Ex: A lady is waiting in line for her change, and steps to the right and trips over a mechanic who just knelt down to fix something. Mechanic is not negligent He did not have a duty to give warning The act was not a continuing obstruction U.S.(owner of Anna C) v. Carroll (Owner of the towing Co. who sank Anna): BLL: There is a duty of care to protect others from harm when the burden of taking adequate precautions is less than the product of the probability of the resulting harm and the magnitude of the harm. Facts: 1944 Companies: Conners Marine Co., Inc., (owner of the barge), Carroll Towing Co., Inc.(tug owner) The sent barge down to 'drill' out one of the barges, and had to throw off the line between the two tiers. The bargee of the barge had left at 5:00PM Due to their negligence in securing the Anna C, it broke loose, hit a tanker, and sank at 2:00PM the next day is suing to recover the cost of the barge from Carroll ( ) is trying to reduce he damages because the s bargee was absent during this occurrence. If someone else would have been able to help, the barge would not have sank Issue: Is the duty of care breached when a s conduct incurred a risk which could have been avoided with very few precautions and which if it inflicts injury will cause a great amount of damage? Ruling: YES There is no general rule that makes the owner of a barge liable in a bargees absence Since it is possible that every barge can break loose, and that it would affect other things, the owners duty to provide against resulting injuries is a function of 3 variables: 1) the probability that she will break away (P) 2) the gravity of the resulting injury (L) 3) the burden of the adequate precautions (B) Carroll Towing Formula: The owner will be found to be negligent if: B<PL If B<PL, then the failure to take precautions is negligence All the variables will vary with the place and the time Ex: Storm, crowded bay Bargees duty: It is custom that the bargee leaves at 5:00, but would return at 8:00 the next day The barge sank at 2:00 PM the next day, and the bargee was absent without an excuse He should have known he needed to be there since it was a busy time The sinking could have been avoided had he been there Holding: It is a fair requirement that there should have been a bargee on board during normal working hours (unless excused) Notes: Def: Duty of Care: A principal of negligence requiring an individual to act in such a manner as to avoid injury to a person to whom he or she owes an obligatory duty. Carroll Towing Formula: Discounting (multiplying) the cost of an accident if it occurs by the probability of occurrence yields a measure of the economic benefit to be anticipated from incurring the costs necessary to prevent the accident The cost of prevention is the burden of taking precautions If the cost of safety measures or curtailment (whichever is less) exceeds the benefit in accidence avoidance to be gained by incurring that cost, society would be better off to forego accident prevention.

Does it make a difference if it is a property injury v. personal injury? Take the least amount of precautions up to the point where the benefits resulting from such use no longer outweigh the danger to be anticipated from it This formula is used more for conceptual purpose than calculations Other perspectives: Compare the value of the average injurer would assign to precaution costs and the value the average victim would assign to the expected accident costs eliminated by the precaution Negligence has a pocket of strict liability For consistent performance (driving) Different than Hand formula: Involves quality of performance rather than high-repetition situations A man creates risks every day. A man just must not create substantial risks. Class Notes: Reasonable People: Can think of it like a bell curve with some outliers Some are very cautious, some are not, but all within the bell curve are reasonable Outliers may be unreasonable people Courts probably think of it more as the average person (the mean of the bell curve) Trespassers: 30-40 years, ago, the land owner had no duty to a trespasser to keep his land safe Now, there is a slight duty to at least warn Seen in jury decisions

The Reasonable Person


Bethel(/injured passenger) v. NYCTA(/bus company): Basic Negligence standard of Reasonable care Overrules the United Airline case- no longer heightened care for common carriers BLL: A common carrier is subject to the same duty of care as any other potential tortfeasor Facts: was injured when a wheelchair accessible chair on a bus collapsed beneath him claimed that NYCTA was negligent on a theory of constructive notice (through a receipt to a repair) because it should be held to a high standard of care because it was a public bus (common carrier) Jury found for Issue: Is a common carrier subject to the same duty of care as any other potential tortfeasor? Ruling: YES Old standard: Utmost care, so far as human skill and foresight can go Negligence standard of reasonable care: No difference as a matter of law Only differences as a matter of fact Why use standard one: Today, just as safe to travel by public transport as private Cannot make a standard for each situation Standard of conduct must be an external and objective one Do this by creating a fictional person the reasonable person of ordinary prudence Takes into account the circumstances Perceivable risk Gravity of harm to others Relationship between parties It is sufficiently flexible to account for different torts. Holding: Strict standard of care for common carriers is not longer valid. Only the reasonable person standard of negligence applies Notes: Standard of Care: A uniform degree of behavior against which a persons conduct can be measured when determining liability in negligence cases Reasonable person standard of care is proportionate to the danger of the act

Guns, etc. Reasonably prudent person Imaginary, but not imperfect What a reasonably prudent person would do in the circumstances to avoid negligence represents the general moral judgment of the community what it feels is not always what it does Capacity for care: The s conduct is the critical determinant, and the conduct is measures against external, objective norms (not subjective ability) Exceptions: When a man has a distinct defect that makes taking certain precautions impossible Blind, infant Mental illness is iffy Does not excuse him from other responsibilities It is assumed, however, that a man possess ordinary capacity to avoid harming others unless a clear incapacity is shown, and it does not hold him liable for unintentional injury, unless, possessing such capacity, he might ought to have foreseen the danger Take moral considerations (other than external objective considerations) into account only to draw a line Ex: old man places a hay stack next to his neighbors house, which ends up lighting on fire Cannot claim that , acting in the best of his judgment, should not be responsible for not being intelligent Court found that it could not create different standards for each individual Ex: 73 year old who suffered a stroke and continued to drive The drivers actions were not wholly beyond his control He retained some control, and remains liable Ex: claims that a car accident was the result of wigging out Mental Deficiency: Rest 2nd 283B:Unless the actor is a child, his insanity or other mental deficiency does not relieve the actor from liability for conduct which does not conform to the standard of a reasonable man under similar circumstances 4 justifications: 1) difficulty in drawing the line 2) unsatisfactory evidence of mental deficiency 3) if mental defectives are to benefit from society, they should pay for their harm 4) the defect will stimulate others around them to look after them Rest 2nd 283C: if the actor is ill or disabled, the standard of conduct to which he must conform is that of a reasonable man under like disability Heart attack, dizziness, delirium, etc. are regarded as circumstances taken into account in determining what a reasonable man would do. Superior Attributes: Rest 2nd 289b: must exercise superior attributes as the actor himself does Children: Parents are rarely vicariously liable for their children May be fond negligent for permitting their children to do something Traditionally, children are held to the standard of a reasonable person their age and intelligence What is the capacity of this child? How would a reasonable child of like capacity act under similar circumstances When children engage in adult activities, courts have applied adult standards Driving a car (even if in an instructional course), boat Not: skiing (an activity for all ages) Lack of competence is no excuse Emergency Doctrine: Person confronting an emergency is required to exhibit only an honest exercise of judgment This does not apply, Must still use a reasonable person standard Class Notes: Collapsing of different standards of care into one standard of care

The Roles of the Judge and the Jury


Baltimore & Ohio Railroad Co. () v. Goodman(/killed in a train accident):

The roles of judge and jury BLL: A person who has failed to exercise reasonable care to avoid an accident is not entitled to recover damages from the other party Facts: It was daylight, and the was familiar with the track When approaching a track, the could not see because of some box cars. He slowed down, and listened, and then proceeded He was truck and killed Issue: May a party to an accident recover damages despite the fact that he did not use reasonable care? Ruling: NO Nothing suggests that the should be relieved of his own death He had the responsibility to get out and look Question of due care is generally left to the jury, but since the failure to follow a standard of conduct is so evident, the court can decide. Notes: Reasonable care: The degree of care observed by a reasonably prudent person under similar circumstances (ordinary care) Contributory fault: If the is just a little bit at fault, he loses completely Was a standard in the past and used in this case Contributory Negligent: If the is partially negligent, then the amount of damages is reduced proportionately Pakora (/car driver) v. Wabash (/Railroad Co.): The roles of judge and jury BLL: Unless reasonable minds could not differ on the point, the standard by which negligence is measured is for the jury to decide. Failure to get out of a vehicle and look before crossing a railroad track is not contributory negligence as a matter of law Facts: A string of box cars cut off the s view to the north. He listened, but heard no bell or whistle There was a line of cars behind him He reached the main track and was struck by a train coming from the north Issue: Was s failure to get out of vehicle and walk forward to look down the track before driving forward contributory negligence as a matter of law? Ruling: NO It was his duty to look, but that does not mean, that by going on, he was negligent any more than being in the dark The crossing was a frequently crossed track He thought that a train would surely blow its whistle Different than Goodman: The train was plainly visible in Goodman He failed to look and see, and failed his duty To get out of a vehicle is an uncommon procedure, and can sometimes be just as dangerous This is why we must be cautious in framing a rule Whether his conduct is ordinary is suitable for the jury Notes: Other examples: was hit in the eye while watching a high school baseball game and sued the school district the field was equipped with a backstop and two fences the court found that there was no basis for a jury since the school district fulfilled its duty of reasonable care to the as a matter of law, and no question of negligence remained for the jury to decide Dissent thought that baseball reflects changes in public perception and safety measures, and the jury needs to address these changes was getting her hair bleached, she lit a cigarette, and the cotton in her hair caught on fire the court found that it was up to the jury to decide whether the hairdresser failed to act upon a risk that was foreseeable to him Cardozo (Pakora) v. Holmes (Goodman): Holmes basically wanted to write a train statute Cardozo wanted to let it go to the jury

Depends on how many cases they have seen on this, etc. Andrews(/struck) v. United Airlines (/common carrier): The roles of judge and jury BLL: Even a small risk of serious injury to passengers may form the basis of a common carriers liability if that risk could be eliminated consistent with the practical application of airline travel. Facts: When checking at the gate, a briefcase fell from the overhead bin and struck and injured the He doesnt know who opened it or who put it there, but he claims that United could foresee the injury and didnt prevent it Court granted SJ for United Issue: May even a small risk of serious injury to passengers form the basis of a common carriers liability if that risk could be eliminated consistent with the practical application of airline travel? Ruling: YES The is not an insurer of his liability not strict liability United had a duty of a common carrier, the utmost care and vigilance of a very cautious person towards its passengers Responsible for even the slightest negligence The degree of care is only such as can reasonably be exercised with the character and mode of conveyance and the practical application of its business s arguments for going to the jury: United has had 135 incidents Only added a warning United could have taken preventative steps, such as adding netting or making passengers carry lighter items s arguments out of the 135 incidents, not much injuries adding netting and restrictions would frustrate the passengers and increase costs has made a sufficient case to overcome SJ: jury must decide whether the hazard is serious enough to warrant more than a warning given Uniteds heightened standard of care United has not demonstrated that cost or interference would be grossly disproportional Jury could find either way Notes: Common carriers owe a reasonable standard of care under the circumstances, and they are liable for even the slightest negligence The issues of duty and breach are to be decided by the jury if the jury could decide either way May have been different if the passenger had stowed it in the middle of the trip (no supervision while boarding) Class Notes: This case, like Bethel, is not an accident directly related to the carrier Under utmost care, jury would probably find United negligent But under standard care, probably not But, given that there was a common carrier standard of care in this state, the was given the benefit of the doubt Emergency care doctrine: The failure to use reasonable care is excused in an emergency Not too applicable anymore because it is superfluous Can this be related to the utmost care standard? Common carrier is on its way out Utmost care does not mean care without reference to cost (B<PL) If B > PL, may not be required to act with the utmost care. This case should not necessarily be used for utmost care since this case was more of a doubt for SJ case Trimarco( /tenant) v. Klien ( /apartment owner): The roles of judge and jury BLL: Evidence of custom and usage by others engaged in the same business is admissible as bearing on what is reasonable conduct under all the circumstances, which is the quintessential test of negligence. Facts: was very badly hurt when he fell through the glass of his shower door. The door was a glass door, not tempered glass that doesnt shatter.

The shower was installed in 1953 Since the 1950s, it had become a practice of using the tempered glass But actually, the custom was to replace the glass if it broke claims that by 1976 (date of accident), the glass door no longer conformed to accepted safety standards. Jury awarded damages CoA reversed appeals Issue: Does evidence of custom and usage have a bearing on what constitutes reasonable conduct under the circumstances of a particular case? Ruling: YES When certain dangers have been removed by a customary way of doing things safely, this custom can be used to show the has fallen below the standard required and is negligent When there is proof of a custom and proof that the conformed, there is due care When there is proof of a custom and proof that the did not conform, and the ignorance is a proximate cause of the injury, this may establish liability (according to the jury) Test of reasonableness Before deciding whether it is a common custom and usage, must decide if it is reasonable Proof of custom and usage: Reflects the judgment of many Relevance and reliability has a direct bearing on feasibility Practicality of precaution in operation Readiness through which it can be employed Does not have to be universal Sufficient to be fairly well defined and in the same calling or business so that the may be charged with knowledge of it or negligent ignorance. It is for the jurors to decide whether or not it is reasonable and whether or not the evidence in this case does establish a general custom or practice. Notes: Todays standards: Courts do not use the custom and usage standard to define the standard of care However, the custom and usage can used in deciding whether the actor has behaved reasonably If a can prove he has adhered to the prevailing customs, it lead the court to 3 points: 1) If an industry adheres to a single way of doing something, the court may accept the s argument that there are safer way to do things than how the does the court may require the to assert the feasibility of the alternative can show that the should have known about it there would be no great upheaval in a judicial determination complying with the custom 2) even if the can show a feasible alternative, if it was not used anywhere, there may be no reason for the to have known about it 3) existence of a custom that involved large fixed costs may warn the court of the social impact of a jury or court decision that determines the custom to be unreasonable. Ex: the fell in a blackout in a hotel room He suggested the hotel should have emergency lighting, but no one else was using it While industry guide is not conclusive, it is useful unless, under the particular circumstances of the case a reasonable person would not conform to the industry-wide standard. can also seek to show that other people in the industry use the alternative, even though it is not a custom however, it cannot just be one or two people using it Argument: this has been known for 11 years constructive negligence custom is well known therefore, possible that he is negligent for jury to decide the one area where custom evidence is dispositive is medical malpractice relevant in most other suits still remains the duty of the jury to determine if it was reasonable care Martin ( /killed in buggy) v. Herzog (car driver):

Violation of a statute can be negligence per se BLL: Failure to perform a statutory duty constitutes negligence per se, which can be prima facie evidence of contributory negligence (which must be overcome by proof in the opposite direction). Facts: When the Martins were driving into town, they were not using the lights (in violation of the statute). When going around a curve, the Martins were hit by the s care who was slightly over the center line, and Mr, Martin was killed. There was no evidence of any functional problems with the car The beams did not reach far enough to the side to see the buggy The s charged the with negligence in that he did not keep to the right of the center of the highway The trial court found that driving without the lights on, even though in violation of the statute, was not negligence per se, and the jury would decide if it was negligence Issue: Does the failure to perform a statutory duty, absent some excuse, constitute negligence per se? Ruling: YES By the very terms of the hypothesis, to omit, willfully or heedlessly, the safeguards prescribed by law for the benefit of another that he may be preserved in life or limb, is to fall short of the standard of diligence to which those who live in organized society are under a duty to conform. Can apply separate claims of negligence and statutory violation, but here they can go together Does not apply the same for ordinances and regulations Usually not negligence in itself, just evidence of negligence Why jury does not have the choice to decide if it is negligence: Jurors have no dispensing power by which they may relax the duty that one traveler on the highway owes under the statute to another The omission of these lights was a wrong, and being wholly unexcused was also a negligent wrong. Notes: Negligence per se: Conduct amounting to negligence as a matter of law because it is either so contrary to ordinary prudence or it is in violation of a statute. Prima facie: An action in which the produces sufficient evidence to submit an issue to the judge or jury for determination Criminal Statute v. Civil Statute : Can use the criminal statute for guidance in determining the standard of care, even if the is not being charged for a crime The suit is based on a common-law tort In the absence of any standard, it goes to the jury to decide what a reasonable man would do in the same circumstances When a legislative body has generated a standard from the experience of the community, the court accepts the formulated standard except where it would impose liability without fault 286 of Rest 2nd: The court may adopt as tha standard of conduct of a reasonable man the requirements of a legislative enactment or administrative regulation whose purpose is found to be exclusively or in part A) to protect a class of person which includes the one whose interest is invaded B) to protect the particular interest that is being invaded C) to protect the interest against the harm that has resulted To protect that interest against the particular hazard from which the harm results However, some cases have held that the court has the right to refuse to adopt the legislative act as the standard even if the criteria of 286 are met If it is outdated, obscure, etc. Most of the time, the statute does not do much in the case A can establish a violation of a statute, show damages, and rest can rebut the proma facie evidence Tedla ( /brother and sister killed) v. Ellman ( /car driver): Violation of a statute not always negligence per se BLL: Where a statutory general rule of conduct fixes no definite standard of care, but merely codifies or supplements a common law rule, which has been subject to exceptions; or where the statute is intended to promote public convenience or safety, then in the absence of clear language to the contrary, it is not negligence as a matter of law for one to violate the statute, if by doing so he is likely to prevent, rather than cause, the accident which is the purpose of the statute to avoid. Facts: Two junk collectors were walking along a busy highway with their carriages.

There were no sidewalks, and they could not walk in the middle They were walking eastward (with the light traffic) in the same direction of traffic State statute: Must walk against traffic feels that although he was negligent, they were contributorily negligent as a matter of law Issue: Is a violation of a statute always negligence per se? Ruling: NO Cannot apply Martin v. Herzog Creation of the statute: Legislature may think that a standard reasonable care is sometimes not sufficient It may then establish rules through a statute But, obeying the statute can not add to the danger of the situation Why this statute is different from the Martin statute: The statute does not proscribe safeguards to preserve a life In unusual circumstances (such as the instant case), following the statute may create more danger This statute gives only a general rule of conduct for all to be safe and aware, not a defined standard of care, because it cannot be strictly applied in all situations It merely codifies the common law rule which has always been subject to exceptions Promotes public convenience and safety The legislature directed pedestrians to face traffic to be more aware, not for the preservation of life Legislature never contemplated that traffic would be heavier in one direction Notes: Purpose of the Statute: A court cannot base the standard of care upon a statute where a party violates the statute when the statute is intended to protect against a different harm than actually occurred Ex: assured clear distance ahead statute Two drivers collided when in an emergency situation Statute did not apply because it meant a clear distance that one could reasonably rely on Assured does not mean guaranteed Ex: a statute that you could not ride on Sunday was hurt from a road obstruction negligently left by the city claimed that the violated the statute, and he was therefore not liable court held that the statute was designed to promote public order, not safety Ex: statute saying that you could not leave car running while filling up Person left car running, and it rolled into someone and hurt them Statute did not apply because it was designed to prevent fires Special Circumstances for violating a statute : There must be a special circumstance to which justifies violation of the statute Cannot just ask the jury to vase their decision on the reasonable person standard when there is a statute that was violated. Licensing: Licensing statutes have generally not been able to set standards of care Purpose is to protect the public from actions performed by unskilled persons. If that is the purpose, the must prove that the lacked the required skill (negligence), not just that he wasnt licensed. Ex: drivers, chiropractor Statutes barred from use in civil cases: Some specifically state that they cannot be used for the standard of care Ex: OSHA Compliance: Does compliance with a safety statute necessarily satisfy the standard of due care? Ex: smoker with the nicotine patch, while smoking, dies of a heart attack Patch complied with the FDA required warning, but it only said a possibility of fainting Whether the duty to warn has been fulfilled is to be measured by the common law of negligence, not the FDA Judicial nondeference may make sense when the administrative regulatory scheme is casual or sporadic (as with consumer products), but not for comprehensively regulated industries (aircraft, electric) that entail potentially mass-exposure hazards Once that determination has been made by a license agency, the courts should respect it

Proof of Negligence
There are many stages of proving negligence This section focuses on the s burden of proving that the s conduct fell below the standard of reasonable care Must prove what the did or did not do May sometimes have to prove the reasonableness Type of evidence: Real: documentary (skid marks photos, broken bottle, etc.) Direct: eyewitness may testify Negri ( /shopper) v. Stop and Shop ( /shop): BLL: A may make out a prima facie case of negligence by presenting circumstantial evidence that had constructive notice of a dangerous condition which allegedly caused injury to its customers and did not remedy the condition. Facts: While the was shopping, she slipped and fell and hit her head on the floor in some broken baby jars The jars looked dirty and as thought they had been there for a while A witness stated that she had not heard the jars falling 20 minutes before the accident Evidence also showed that the aisle hadnt been inspected for at least 50 minutes-2 hours before the accident Issue: May a make out a prima facie case of negligence by presenting circumstantial evidence that had constructive notice of a dangerous condition which allegedly caused injury to its customers and did not remedy the condition? Ruling: YES had sufficient evidence to prove constructive notice of a dangerous condition, and the did nothing about it the circumstantial evidence was sufficient to draw an inference for the liability Notes: Preponderance of the Evidence: the standard of proof requiring the tier of fact to determining whether the fact sought to be established is more probable than not Circumstantial Evidence: Evidence that, though not directly observed, supports the inference of principal facts. Mode of Service Argument: the way the produce/food is delivered should give the constructive notice Soda fountain, food in bins Exculpatory: evidence: in favor if the party Inculpatory: not in favor Gordon( /injured) v. American Museum of Natural History ( /Museum): BLL: To constitute constructive notice, a defect must be visible and apparent and it must exist for a sufficient length of time prior to the accident to permit to discover and remedy it Facts: slipped and fell on a piece of paper on the Museums steps alleged that it came from a concession stand contracted by the Museum claims that the Museum was negligent because it failed to discover and remove the paper before he slipped Issue: To constitute constructive notice, must a defect be visible and apparent and it must exist for a sufficient length of time prior to the accident to permit to discover and remedy it? Ruling: YES There is no evidence to show that anyone knew of the paper before he slipped on it The did not describe the paper as being duty or worn (being there for a while) Does not matter that he saw other papers on the steps 10 minutes prior There is no evidence of actual or constructive notice by the museum Notes: Actual Notice: Direct communication of information that would cause an ordinary person of average prudence to inquire as to its truth Liability in business: Ex: slipped on banana peel. Even though peel was blackened, it was insufficient evidence that the store had notice of the peel Ex: slipped on a wet floor sued the building for not wiping up the wet floor. The trial court allowed evidence showing the s premises was accident free for 10 years He erred in admitting this evidence because it was not relevant to the accident Even if it was relevant, the court must weigh the evidence. It had nothing to do with what the janitor did that day, Ex: slipped on lose bird seed in Kmart

sought to invoke the business practice rule which does not require constructive notice ot the business that there was a reasonably foreseeable risk of harm court would not allow this rule, and made the show how long it had been there, etc. the merchant must show the methods used to avoid the foreseeable risk of harm the business practice rule is used to address the hazard with customers picking items from bins would have been different if Kmart had a big bin of seed out 2 reasons why a might not present available evidence: 1) it was known but not favorable automatic rejection of the claim 2) the had not conducted an investigation may be dismissal if the monetary value is small Compared to Negri: The baby jar arose from something that happened inside the store The paper on the museum steps could have blown from an outside source (hotdog vendor) The has not created the risk, some 3rd party has If the museum paid the hotdog vendor to be there, then the museum might be acting as an agent for the hotdog vendor Byrne( /Injured) v. Boadle ( /Shop Owner): BLL: There are circumstances in which the mere fact that an accident has occurred gives rise to a presumption of negligence which then must be rebutted by the Facts: When the was walking by the s store, he was injured by a barrel of flour that fell from a window above s shop A witness testified that it was not lowered by a rope Issue: Are there circumstances in which the mere fact that an accident has occurred gives rise to a presumption of negligence which then must be rebutted by the ? Ruling: YES There are many accidents where no presumption of negligence can arise But cannot say that negligence cannot arise from the accident Ex: barrel rolled out of the warehouse Would be the duty of the warehouse to keep them safe This would be prima facie evidence of negligence- there is no need for the to prove that it would not have rolled but for negligence Same rule for the barrel dropping: The accident is prima facie evidence of negligence- there is no need for the to prove that it would not have dropped but for negligence The barrel was in the custody of the at the time The must prove any facts that rebut the prima facie evidence Notes: Res Ipsa Loquitor: the thing speaks for itself A rule of law giving rise to an inference of negligence where the instrument inflicting the injury is the exclusive control of the and where such harm could not ordinarily result in the absence of negligence Prima Facie Evidence: Evidence presented by a party that is sufficient, in the absence of contradictory evidence, to support the fact or issue for which it is offered. could show causation and damages, but could not show a duty or breach. McDougald ( /Injured) v. Perry( /Driver): res ipsa loquitor BLL: The doctrine of res ipsa loquitor provides an injured with an inference of negligence where direct proof is not available, if the establishes that the instrumentality causing his injury was under the exclusive control of the and the accident is one that would not, in the ordinary course of events, have occurred in the absence of negligence on the part of the one in control of the instrumentality Facts: was driving behind the s trailer the had a tire secured beneath the truck with a chain, and when the chain broke, the tire flew up and hit s window

claims that the chain was fastened when he bought the trailer in 1969. However, at the time of the accident, it was fastened with a nut and bolt claims that he did an inspection before he left, but he didnt inspect every link district court directed the jury: not to base it on negligence to base it res ipsa loquitur Issue: Does the doctrine of res ipsa loquitor provide an injured with an inference of negligence where direct proof is not available? Ruling: YES Agree to use res ipsa loquitor (RIL) Res ipsa loquitor: The thing speaks for itself A rule of law giving rise to an inference of negligence where the instrument inflicting the injury is the exclusive control of the and where such harm could not ordinarily result in the absence of negligence It is a rule of evidence Only applies to rare circumstances The direct proof of negligence can be absent, provided there are certain elements of negligent behavior present (immediate, precipitating cause). 328D of Rest. 2nd: Type of event: First requirement is that past experience must show that such events do not ordinarily occur unless someone has been negligent Ex: derailment of trains, objects falling from s home, gas leak Basis of Conclusion: Basis of past experience is a reflection of the common knowledge of the community It may be supported by evidence and expert testimony saying that such events do not ordinarily occur unless someone has been negligent Analysis: Spare tire escaping is the type of accident that would not normally occur without negligence It was in the control of the An inference of negligence comes from the circumstances of the accident Doesnt matter that other possible explanations are available The is not required to eliminate other possible explanations for the accident All that is required is evidence from which reasonable persons can say that on a whole it is more likely that negligence was associated with the event than there was not. RIL is used in this case because Notes: Different ways to apply weight to RIL at trial: Tell the jury that it may but need not find the negligent Tell the jury that it must find the negligent unless the presents plausible rebutting evidence Tell the jury that it must find the negligent unless persuaded that the was not negligent Inference view v. stronger view: Inference: showing was so convincing that the inference of negligence arising therefrom is inescapable if not rebutted by other evidence If the presents such evidence, the still bears the burden of persuading the jury that the was negligent. Other courts find that the jury need not infer negligence Can RIL apply where the wishes to prove specific acts of negligence? Ex: helicopter crash killed provided testimony that pilot waved to people before landing and had several drinks before taking off The judge instructed the jury that it could find negligence in the specific acts charged or infer it from the happening of the accident. What if the state has well-developed pre-trial discovery procedure? Ex: child came back from daycare with a lump on head, concussion, and crossed eyes Court found it was still OK to invoke RIL Ex: childs eye hurt while in daycare Inspected all the furniture and toys and found nothing that could have caused it

Court could not invoke RIL because the instrumentality producing the injury or damage was unknown or was not in the exclusive control of the Spoliation of Evidence: Sometimes the insurer of the will destroy the evidence so the cant get to it Sanctions may be imposed Automobile cases: Simple rule of rear end collisions: person who rear-ends is automatically liable Courts would generally not apply RIL if the brakes failed. has the burden Falling asleep at the wheel: The would have a hard time establishing the circumstances surrounding the at the time of the accident; therefore the burden is normally placed upon the . Ybarra v. Spangard: BLL: Where a medical patient suffers unexplained injuries to a part of his body not under treatment and not within the area being operated on, RIL applies and gives rise to a presumption of negligence covering all those s who had any control over his body or the instrumentalities which might have caused the injuries. Facts: Patient, went in for an appendectomy, and was laid on a table. The operating room people laid him on a table against 2 hard objects that were below his neck. After surgery he was unable to lift or rotate his arm, and developed paralysis and atrophy of the muscles around the muscles of the shoulder In the opinion of an expert, this was due to pressure or strain Issue: Does RIL apply? Ruling: YES Background of RIL: 3 conditions of RIL: 1) the accident must be of a kind that does not occur in the absence of someones negligence 2) it must be caused by an agency or instrumentality within the excusive control of the s 3) it must not have been due to any voluntary action or contribution on the part of the the absence of contributory negligence on the part of the partially captured in the 2nd element Purpose of IRL: The should not have the duty of producing evidence where is inaccessible to him and accessible to the When the submits himself to the care of doctors and nurses who render him unconscious, there would be no other way for him to recover unless the doctors and nurses voluntarily disclosed the person This would force the court to invoke absolute liability Application to 3 conditions: 3) it must not have been due to any voluntary action or contribution on the part of the of course satisfied 1) the accident must be of a kind that does not occur in the absence of someones negligence of course satisfied distinct injury to a healthy part of the body not the subject of treatment raises the inference of negligence burden shifts to 2) it must be caused by an agency or instrumentality within the excusive control of the s happened while was asleep and in s care s arguments: Where there are several s, and the responsibility of the instrumentality is divide among them, and the injury could have resulted from one act from one person RIL should not apply Neither the number nor the relationship of the s matters Every was bound to exercise ordinary care and each is liable The employers are liable for the doctors, and the doctors are liable for the nurses Nurses become temporary agents of the doctor Liability could be imposed upon doctor by doctrine of Respondeat superior It may be found at trial that one person is responsible and the others are not, but that does not mean that RIL should not apply was unconscious, so there is no way for him to identify who was responsible

Where there are several instrumentalities, and there is no showing as to which one caused the injury, RIL cannot apply. Enough that was injured from an external force applied while he was unconscious at the hospital Control Strict test is actual control There are exceptions Test has become one of right of control rather than actual control Ex: bursting bottle Bottler had constructive control Exception applies to hospital cases This ruling may not apply to all cases Applies where was unconscious, in medical care Notes: RIL: a rule of law giving rise to an inference of negligence where the instrument inflicting the injury is in the exclusive control of the and where such harm could not ordinarily result in the absence of negligence Where RIL was not applied: used wire for a neck surgery wire broke apart and caused other injuries RIL was inapplicable because common experience revealed that s negligence was one of several speculations, but not a probability Ybarra Rule: A who is struck on the head by a flower pot falling from a building can recover against all the tenants unless the guilty one reveals himself Sometimes seen as a minority rule, but if the facts are similar, likely that you will get everyone involved. Cases that reject Ybarra do not actually reject RIL liability (Barret v. Emanuel Hospital), just dont want to let it get out of hand. RIL applies where the cannot identify any evidence of negligence. RIL can also show up where the was conscious, but had no idea what the doctor was supposed to be doing

Medical Malpractice
Topics: Standard of Care Role of Custom Proof Sheeley v. Memorial Hospital: BLL: A physician is under a duty to use the degree of caution and skill that is expected of a reasonably competent practitioner in the same class, action in the same or similar circumstances Facts: gave birth a 2nd year resident performed the episiotomy, developed complications on site and sued trial judge excluded s expert testimony of a board certified OB/GYN Issue: is a doctor subject to the same care as other similarly situated? If so, what type of expert is the allowed to have testify to that standard of care? Ruling: YES Expert testimony: contends: thinks that same or similar locality rule should apply that it should be someone in the same field, position and area the expert is too qualified to testify for the standard of care applicable to a 2 nd year resident court: in medical malpractice, expert testimony is an essential requirement is proving the standard of care applicable to the , unless the lack of care is so obvious that a common layman would know court previously ruled that an expert testimony is acceptable if the expert has the prerequisite knowledge, skill, experience, training or education in the field of the Old rule: same of similar locality rule is just an expansion of the strict locality rule (where expert must come from same community)

can come from a similar community New rule: whether the procedure was performed with the recognized standard of care essentially whether the treatment was administered in a reasonable manner Requirements for the Expert: Knowledge and familiarity with the procedure, acquired through experience, education, observation, etc. Issues to consider are: resources available to the physician, length of time has been practicing, specific area of practice

Notes: Sometimes the national standard is only invoked if the board certified. Education is national Tests, books Doesnt matter if a different area because still held to the same standard of care Calling yourself a specialist submits you to the same standard of care of other specialists When an expert can opine that he has knowledge of the custom and standard of care, he can testify The standard for doctors is different than other standards of care Normally, the jury looks to the normative standard to what a reasonable person would do in similar circumstances For doctors, they are held to high, nationalized standards Some statutes impose limits on expert testimony Liability of hospitals: Hospitals can be liable for failing to provide safe facilities and competent physicians Difficulty in obtaining witnesses Most Dr.s do not want to testify against another in the field More reason not require that the expert come from a certain geo. Situation not needing an expert: Where the malpractice is within the knowledge of a common laymen Ex: leaving a sponge inside Ex: leaving an instrument Even though the could argue that it is not a custom to count the instruments before and after (which establishes due care), it does not establish the standard of care Ex: where the nursing home allowed a lady to go unrestricted (where there was specific instructions not to) and she broke her hip The standard of care was the compliance with the instructions Every 5 operations, there are 5 failures, and 3/5 involve negligence Then there are 5 medical malpractice cases that are filed Does RIL apply to the first case? There is a 60% chance that the accident resulted as negligence, therefore, the jury may find for the Then the following 4 cases would rule the same way, without any specific evidence that there was negligence. Conners (/injured) v. University Associates in OB/GYN(/Dr.) : BLL: A may use RIL in a medical malpractice action even though the introduces expert testimony to prove the s specific negligence causing injury Facts: had surgery to try to get pregnant the hospital doctors restrained her leg during the operation to keep the incision open, and afterwards she left all feeling in her leg had an expert testify about: the retractor, in that it required certain methods of ensuring no injury would result the injury would not have occurred even if s nerve had been abnormally situated contend that IRL should apply where a is allowed to have expert testimony because the jurors share a common experience that allows them to make a certain inference of negligence Ruling: Why the can use expert testimony The Rest. encourages the use of expert testimony In some cases there is no common knowledge about the injury

It does not matter if the knowledge is common or comes from an expert, the key question is still whether the accident would have occurred in the ordinary course of events The expert opinion can educate the jurors The jurors can determine the credibility of the expert Analysis: The testimony did not try to prove what caused the injury, just that it would not normally happen without some outside force It does not, by itself, support a finding of negligence A RIL was needed to allow the jury to make that inference If the jurors found that the expert was credible, they could infer that the physicians were negligent simply because the accident occurred. Notes: Is this case saying that the must prove certain aspects of the case through expert testimony? Not a clear case of RIL, like the barrel of flout example. Policy behind RIL: normally has better access to information as to how the accident occurred by allowing the jury to hear expert testimony and instruction on IRL, the court is ensuring the jury hears as much information as possible regarding the cause of injury and the usual causes of injury of this type Matthies (/injured hip) v. Mastromonaco (/Dr.): BLL: To obtain a patients informed consent to one of several other courses of treatment, the PT should explain medically reasonable invasive and noninvasive treatments, even when the chosen course is noninvasive Facts: fell in her apartment and broke her hip The Dr. recommended bed rest to her without giving her any other options for 2 reasons: She was frail, and the surgery would be risky Her bones were porous and the pins would probably not stay 40 years earlier she suffered a stroke from a blood transfusion he believed she should live in a long-term care facility She went into bed rest, but it got worse and lost all capability of using that leg asserts that she would not have consented had she known the alternatives and how each would affect her quality of life Issue: Whether the doctrine of informed consent requires a PT to obtain a patients consent before implementing a non-surgical type of treatment Whether a PT should discuss, in addition to the alternatives that he recommends, the alternatives he does not. Ruling: YES/YES choosing among alternatives is shared responsibility of PT and patient but the patient makes the ultimate call one patient may prefer a risky procedure as a tradeoff for improved quality of life, and the other may not informed consent does not only apply to invasive procedures : analyze under negligence, not battery focuses not on touching or invasion of the patients body, but on the PTs deviation from the standard of care does not matter if it is invasive, it matters if the patient has enough information to make an informed decision would be different for antibiotics patient needs only the information that is material to a reasonable patients informed decision PT should describe the material risks inherent in the procedure or treatment Test for materiality of a risk: Objective test Whether a reasonable patient in the patients position would have considered the risk material. If so, shows causation Does not matter if the says that she would have chosen a different alternative after the fact PTs duty to disclose is especially important when the alternatives are mutually exclusive Notes: History of standards employed for informed consent cases :

Originally, the majority rule was custom (what would PTs customarily do) Eroded because of patient autonomy Cant all be the doctor Another view is the subjective view: What would the patient want to know? Problem is that the patient could say at trial that he would have chosen differently if he knew of the risks and choices involved Everyone has their own beliefs about what is good for them, and no standard can be created. Arguably, this is the best one, but would require a lot of proof to show that the would have chosen differently Now, the objective standard is used: Whether a reasonable patient in the patients position would have considered the risk material. If so, shows causation In some states, the must produce expert testimony to establish inadequate information when alleging lack of informed consent In others it is battery Here the court said the PTs deviation from standard of care was the main focus in analyzing the informed consent claim

Duty
Obligations to Others
Courts are moving towards a general duty of care, rather than a specific duty for each case. Many times, Special relationship determined that duty of care Duty Generally: Concept: Generally, a person owes everyone else with whom he comes in contact a general duty of care. Normally, just the duty to behave with care that would be shown with a reasonable person Where a owes the less than general care: generally has no duty to take affirmative action to help generally has no duty to avoid causing unintended mental suffering to has no duty to avoid causing pure economic loss to in the absence of more tangible types of harm (such as physical injury) Failure to Act: No general duty to act: A person generally cannot be liable in tort solely on the grounds that she has failed to act. Duty to protect or give aid: If sees in danger, and fails to help (even though it would have been easy to), is not liable. Ex: passes by drowning in a pond. does not have to help. Non-negligent creation of risk: Ex: taxi picks up a man after his car stalled, and then s hit it. s sued, claiming that the taxi driver had a duty to warn them. Court held that the taxi driver had no affirmative duty to do any of those things He had not voluntarily assumed a duty to s and induced them to rely on it He did not create a peril or change the nature of the existing danger Exceptions: Business Premises: Usually a business must furnish warning and assistance to a visitor, regardless of the source of danger. Ex: inn keepers, common carriers Employers and Universities: Employers must assist employees, and Universities must assist students Actors (Doctors) who later realize what they did was wrong : If the person has done an act, and later realizes that he created an unreasonable risk of danger to that person, he is under a duty to exercise due care to prevent the risk from occurring even though at the time the actor had no idea that it would cause danger. involved in injury: If the danger or injury to is due to s conduct, or to an instrument under s control, then has a duty to assist, even if it is not s fault. Ex: car accident involving both and , where it is completely s fault

322 Rest. 2nd Torts: If the actor knows or has reason to know that by his conduct, whether tortious or innocent, he has caused bodily harm to another such as to make him helpless and in danger of further harm, the actor is under a duty to exercise reasonable care to prevent further harm Some states impose criminal statutes that require the to help if he is involved in the accident. and as co-venturers: Where both are in a common pursuit, some courts impose that the has a duty to give warning and assistance. Ex: 2 friends jogging, camping Assumption of Duty: Once voluntarily begins to help (even if does not have to), must proceed with reasonable care. Preventing assistance by others: will especially be found liable if he beings to help , and dissuades others from helping Ex: if stops to help an injured pedestrian, and others pass by thinking it is OK. Mere promises: Usually, a mere promise from to help (without actually starting to help), is not enough to make liable. Some courts will find liable if relied on the promise. Ex: if a relied on a promise, the court may find that the is liable if he does not keep the promise and the has relied on it sheriff and released prisoner. Duty to control others: If has a duty to control 3rd persons, can be negligent for failing to exercise that control. Special Relationship: A duty to control a 3rd person may come from a special relationship between and , or and another person. Ex: a business open to public must protect its customers from harm from 3 rd persons. Ex: storekeeper in a robbery. How close of a connection is required between the s conduct and the s injury? Look at policy Harper (/injured spine when diving) v. Herman(/boat owner) : Special relationship BLL: A boat owner who is a social host does not owe a duty of care to warn a guest that the water is too shallow for diving if the guest is neither particularly vulnerable nor lacks the ability to protect himself. Facts: , an older experienced boatman, invited a friend on his boat, who invited the , a 20 year old The knew the lake well pulled up near an island so that the people could swim The , without warning, dove in, hit his head, and severed his spinal cord brought suit, claiming that the had a duty to warn him that the water was too shallow for diving because: he was an inexperienced diver , being experienced, should have realized that needed his protection Issue: Did the have a duty to care for the ? Ruling: NO There is no affirmative duty to act unless there is a special relationship between the parties This is true even if realized that needed his protection. argues: special relationship was created when the allowed an inexperienced diver on his boat court: generally, a special relationship is only present between one person and common carriers, innkeepers, people who own land open to the public, and people who have possession of a person under circumstances in which that person is deprived of normal opportunities of self-protection. (Rest. 2 nd Torts 314A) Those elements are not present here No evidence of being vulnerable No evidence of having power over had no advantage of economic gain No evidence of expecting protection from Even though it has been said that actual knowledge of a dangerous condition tends to impose a special duty to do something about that condition, the party is not negligent if there was no duty to provide protection.

This case involved children in daycare who relied on the adult Notes: Farwell (/father of decedent) v. Keaton (/stupid friend): Co-venturers, beginning to aid someone and not continuing with reasonable care BLL: When companions are engaged in a common undertaking, there is a special relationship between them and implicit in this relationship is the understanding that one will render assistance to the other when he is in peril if he can do so without endangering himself. Facts: The two boys went off to follow some girls to a restaurant. The girls complained to their friends, who too them to the back to beat them up. Siegrest escaped, but Farwell was badly beaten. Siegrest gave Farwell an ice bag for his head, and drove around for a couple hours Siegrest left Farwell in his grandparents driveway. The grandparents found him the next morning, and he died 3 days later. Issue: Whether Siegrest had a duty to assist Farwell with reasonable care? Ruling: YES The existence of a duty is normally a question of law, but the factual circumstances can give rise to a duty There is not always a duty to aid someone in distress, but there is a legal duty to avoid affirmative acts which make it worse. If the does attempt to aid him, he will be liable for failure to use reasonable care. Must determine if attempted to aid Special relationship: Where there is a duty to aid a person in peril is usually predicated upon the existence of a special relationship between the parties. The has a duty if he knew or should have known about the other persons peril Analysis: The two boys were companion in a social venture There is an implicit understanding that one will help the other if he is in danger if it is not dangerous for him to do so. Siegrest knew or should have known that Farwell would be in danger if he left him until morning Courts will find a duty where, in general, reasonable men would recognize it and agree that it exists. Dissent: There is no authority that says that a close relationship establishes a legal duty Where a special relationship is to be the basis of imposing a legal duty upon one to ensure the safety of another, it would probably be co-adventurers who embark on a dangerous adventure, where each has the understanding that one will help the other in danger. A moral duty cannot rise to a legal duty where there was nothing the said that indicated that he thought medical attention was necessary, and the relationship between the parties has no affirmative duty to render assistance Notes: Majority recognizes an obligation of due care on 2 grounds: 1) that Siegrest voluntarily came to the assistance of Farwell 2) that Siegrest had an affirmative duty to care based on their pre-existing relationship Ex: was sober one of ten minors who were drinking. Suit was brought against the sober one for not restraining the driver. Court found for the sober one 324 Rest. 2nd: one who, being under no duty to do so, takes charge of another who is helpless is subject to liability caused by the failure of the actor to exercise reasonable care to secure the safety of the other within the actors charge, or the actors discontinuing his aid or protection, if by doing so he leaves the other in a worse position than when the actor took charge of him Ex: the water company is not liable if the water does not work from a fire hydrant to save a burning building Its the denial of a benefit, not a wrong 326 Rest. 2nd: one who intentionally prevents a 3rd person from giving to another aid necessary to prevent physical harm to him is subject to liability Ex: a bartender refused to allow someone from across the street to use his phone to call an ambulance for his friend

The court found that the interfered with the third partys efforts to rescue in a public business Ex: a worker suffered a heart attack at work, and the supervisor prohibited an employee who knew CPR from helping Court held that the supervisor violated his duty not to interfere with a rescue Strauss (/tenant) v. Belle Realty (/tenants realtor): Crushing liability BLL: A may be liable for negligence only when it breaches a duty owed to the Facts: A failure in s power system left most of NY with no power A tenant () of an apartment building fell when going down to the basement for water because there was no light. contracted with for his apartment power, but the basement was a general area of the building paid for by the realtor. brought suit against for negligence because of a failure to fulfill the duty of care to provide electricity. Issue: Because a is liable for negligence only if it breaches a duty owed to the , did the owe a duty to the ? Ruling: Was there a duty? Factors to consider: The did not have a contract with the in that area. Tenant was not a party in privity did not contemplate the affect on individuals when they contracted with the realtors. The agreement between and the realtor was no different that the contract between the and millions of other customers. belongs to a narrowly defined class Was the injury foreseeable? It was a system-wide power failure that was the utilitys gross negligence They cannot foresee every possible injury to every person that may have come into the apartment building Limitations on liability Public policy In determining the liability of a utility company, the duty of care in cases of the failure to provide service that could lead to enormous liabilities has not been extended to non-customers. Must question what remedy would lie in the case of reckless and wanton indifference to consequences measured and foreseen. Holding: Con Edison is not liable Dissent: Can take from the stockholders or increase the rates to pay for the judgments against them The holding ignores the burden placed on the injured The policy used is saying that the more persons injured through someones negligence, the less responsibility for those injuries The court should have required the parties to establish that the catastrophic probabilities are catastrophic to limit liabilities Notes: Only 4 states allow recovery against water companies in fires If they did, courts feel that water would no longer be a cheap commodity Ex: nurse was hurt when a fan fell from a patients room. had contracted with the hospital to perform all maintenance functions. Court found that there was a duty to the non-contractual because the injury was direct and demonstrable, not incidental or collateral. The services of the were not aimed at a faceless audience, it was the known group of hospital employees and patients. The cannot claim that it could not foresee that individuals would expect services with care Ex: pedestrian was struck by a driver who was driving a car out of the garage he worked in Court found that the garage was not liable because no duty arose out of the relationship between the garage and the driver This would have placed an undue burden on the garage. Uhr v. East Greenbush Central School District:

BLL: A private right of action cannot be fairly implied where it would be inconsistent with the statutes legislative scheme Facts: s parents brought suit against the school because the school failed to give a scoliosis test, which eventually necessitated surgery. According to Education Law 905(1), schools are required to test in between ages 8-16. Issue: Whether the Education Law 905(1) may be enforced by a private right of action Ruling: NO Three parts of Education Law: Education Law 905(1): Required examination Education Law 905(2): School shall not suffer any liability as a result of making such exam, that would not have existed prior to this law Education Law 911: Charges the Commissioner of Education with the authority to enforce the law, and adopt rules and regulations for such purpose The test for the right of a private action for violation of a statutory duty (as opposed to one from common law negligence): Statute may authorize it. If it is silent, use the three part Sheehy test: 1) whether the is one of the class for whose particular benefit the statute was enacted undisputed 2) whether recognition of a private right of action would promote the legislative purpose; and Two-part inquiry: What was the legislature seeking to accomplish? The legislature was seeking to benefit the populations health and avoiding costly hospital bills. Would a private right of action promote that objective? The risk of liability for failure to screen will encourage compliance with Education Law 905 Second prong is satisfied 3) whether creation of such a right would be consistent with the legislative scheme although the 2 overlap, the public and private methods of enforcement do not the statute carried its own enforcement system 911 charges the Commissioner of Education with the authority to enforce the law, and adopt ruled and regulations for such purpose also allows the Commissioner to withhold public funding for noncompliance 905(2) shows that the Legislature intended to immunize schools from liability Third prong is not satisfied Common Law Negligence: The school district did not assume a duty to the by creating a special relationship with them through the Education Law, and it therefore did not breach a duty by failing to perform an exam. Notes: Second Rest. 874A: Similar to the three part test Using this, some courts ask whether the state legislature, in creating a new statutory duty, either implicitly or explicitly intended to create civil liability. Duty to rescue: Common law will not generally require rescue Duty to report child abuse: Every state now requires reports by those who have knowledge of or reason to suspect child abuse. Duty to report crime: Several legislatures have penalized those who fail to report crimes they witness Federal Statutes: In the absence of a federal common law, the federal courts cannot create civil liability independent of Congressional enactments In the absence of express statutory provisions, they must decide whether to imply private rights of action. Statutory limitations on liability:

In addition to being used to create civil liability, statutes may restrict or preclude common law duties.

Obligations to Protect a Third Party


Tarasoff(/parent of decent) v. Regents of the University of California(/psychologist) : BLL: Once a therapists knows or should know that a patient presents a danger to a third party, there is a duty to warn or otherwise take reasonable actions to prevent danger. Facts: s patient told him that he intended to kill told the campus police, and they detained him for a short while, but then releases him Upon release, he killed her brought action claiming that had a duty to warn the of the danger. Issue: Did the therapist owe a duty to the to warn her of the potential danger? Ruling: YES Rule from Rowland v. Christian: Whenever one person is by circumstances placed in such a position with regard to another, that if he did not use ordinary care he would cause danger of injury to the person or property, a duty arises to use ordinary care and skill to void such danger Departure from that principle depend on the following factors: Forseeability Degree of certainty Closeness of the connection of the s conduct and injury suffered Moral blame attached to s conduct Policy Burden on and the consequences on the community from imposing a duty Cost and availability of insurance for the risk involved Forseeability: Common law does not require a warning, however, when the foreseeable harm requires a to control the conduct of another person or to give a warning, there is only a common law liability if the has a special relationship with the dangerous person or victim See Rest. 2nd 315 Although, the Restatement indicates that the relationship must be one that imposes a duty upon the actor to control the 3rd persons conduct Ex: parent-child, master-servant, possessor of land, controlling a homicidal maniac, patients with diseases Ex: if a doctor fails to diagnose a contagious disease, and others are infected. s contend that a therapist cannot accurately predict if the patient will actually act upon the threat however, the s do not assert that the failed to predict, only that he did predict and then failed to warn Once a therapist determines or should have determined through applicable standards of professional standards that a patient poses a serious threat to others, the therapist bears a duty to exercise reasonable care to protect the foreseeable victim. Given this principle, the therapists conduct must be measured against the traditional standard of negligence Weighing the damage done, the possibility of inaccurate warnings does no justify no duty The risk of the unnecessary warnings is a price we are willing to pay The warning does not constitute a patient-doctor breach of trust of privacy: Although there is a public interest to protect the rights of patients, that must be weighed against the interest in safety Same under the Evidence Code Concurrence: Agree that if the therapist has determined that there is a threat, he should have to warn Do not agree that a therapist should be liable for a failure to predict Dissent: Tort principles favor nondisclosure Privacy encourages effective treatment Therapists may be tempted to issue excessive warnings. Notes: Ex: passenger did not have a duty to warn the drover of a pedestrian.

There was no relationship between them or the pedestrian to make a duty, even if it was no troublesome for the passenger to say something However, if the guest was hurt in the process, he may have been found contributorily negligent for failing to invoke self-protection. Physicians failure to warn: Imposing a duty beyond the patient-physician relationship: Ex: surgeon operated on a women who knew that the disease she had was genetically transferable, but did not tell her When the child was born with the disease, the court found that the doctor had a duty to tell her because the obligation was obviously for the benefit of identified third parties (baby) Ex: a physician told his patient that she had tested negatively for a disease, and then her future partner contracted it Since the third party did not exist at the time, the doctor had no duty No Duty: Ex: a doctor failed to intervene in a medical malpractice when he was observing a surgery Court found that he had no duty since he was only observing for a staffing position Also a majority rule that doctors employed by companies or insurers to determine whether someone is fit for a certain job or insurable has not duty to reveal information if he finds something Ex: doctor has no duty to warn patient that he cannot drive unless he is not able to right after he is given medication Ex: When a prisoner threatened some child in his neighborhood, the court found that the county was no liable because the child was not identified. If they told the mother, how would the mother be able to warn every child in the neighborhood, telling them that her son might kill them NY approach: First case found that forseeability alone did not establish a duty That was overturned later. Suicide: Court found that the physician was not liable for his patients suicide because Tarasoff does not apply to self-inflicted harm or mere property damage. Duty to disclose that a patient has AIDS to his sexual partner. Normally physician does not have a duty to notify, and will not be liable to disclosing Parents duty to control child: Rest. 2nd 316: A parent has a duty to exercise reasonable care so to control his child if the parent knows or has reason to knows or has reason to know that he has the ability to control his child, and knows the necessity for exercising such control. Randi(/Student) v. Muroc Joint Unified School District(/School): Negligent Misrepresentation BLL: The writer of a letter of recommendation owes to third parties a duty not to misrepresent the facts in describing the qualifications and character of a former employee, if making those misrepresentations would present a substantial, foreseeable risk of physical injury to third persons. Facts: wrote letters of recommendation for a new school, saying all positive things, and not disclosing anything about his past sexual charges the employee then molested a 13-year old student brought suit for negligent misrepresentation, fraud, and negligence per se. Issue: Whether has sufficiently pleaded that s owed her a duty of care, that they breached that duty by making misrepresentations, or giving false information, and that the schools reasonable reliance on their statements proximately caused s injury Ruling: YES A party cannot be held liable for failing to disclose information, but it can be held liable for providing affirmative misrepresentations that present a foreseeable and substantial risk of physical harm to a third person. Fraud and Negligent Misrepresentations: 311 of 2nd Rest. of Torts: One who negligently gives false information to another is subject to liability for physical harm caused by action taken by the other in reasonable reliance upon such information, where such harm results

To the other, or To such third persons as the actor should reasonably expect to be out in such peril by the action taken Such negligence may consist of a failure to exercise reasonable care In ascertaining the accuracy of the info In the manner in which it is communicated Duty to : Case Law: Did the s owe the a duty of care? No special relationship However, foreseeable harm Garcia: Parole officer had a duty to exercise reasonable care in giving the victim info regarding the parolee who killed her This case is distinguishable from the instant case because no CA court has every decided whether the owed a duty to an unidentified 3 rd party Therefore, case of 1st impression Tarasoff: All s have a duty to exercise reasonable care to prevent others from being injured as a result of their conduct Ballard: The courts task in determining the duty is not to decide whether a particular s injury was foreseeable in light of a s particular action, but whether general the s category of negligent conduct is likely to result in the kind of harm experienced, and therefore the is liable. Foreseeability and Causality: Whether s injury was a foreseeable result of s representations and failure to disclose YES If had not written so highly of him, the school would not have hired him could reasonably foresee that after being hired, there was a chance he would molest students Moral Blame: This would depend on proof at trial Availability of Insurance or alternative courses of conduct : s had a choice to avoid liability: writing a full disclosure letter writing a no comment letter Public Policy Considerations: There is certainly public interest in preventing child abuse (must report it) s urge that there are competing social and economic interests may prevent employers from writing letters hurts employees s claims that it is unlikely that employers will decline to write letters if the employee is truly qualified and there is no foreseeable risk of harm there is a statutory provision that encourages open disclosure of relevant info for employees will protect the employer from defamation if the letter is not malicious malicious: know its false or speaking with the intent to injure Conclude: There is a duty to a 3rd party when writing a LOR not to misrepresent the facts in describing the qualifications of a former employee, if making these misrepresentations would present a substantial, foreseeable risk of injury to a 3rd person Misleading Misrepresentation or Mere Nondisclosure: s committed actual affirmative misrepresentation, not mere nondisclosure letters amounted to misleading -truths containing incomplete information knew that his past behavior led to his resignation if provided some info, it was obliged to provide all other facts which materially qualify or limit the other facts given claims that a negligent misrepresentation requires a positive assertion and does not refer to implied misrepresentations But s make positive assertions because they knowingly concealed material facts The letters recommended him for any position, implying that he was fit to interact with female students

Conclude: Misleading -truths are an exception to the general rule excluding liability for mere nondisclosure Reliance: The schools reliance was sufficient under 310 and 311. Notes: Why the school did not want to write a negative letter: Wanted to get him out of their school Did not want the teacher to sue back for defamation Need not be false Must cause injury to their reputation Does it matter that the 2nd school did not inquire into his past or ask the previous school for more info What if the was visiting from out of state, and not yet a student there She would probably win Negligent Misrepresentation: A misrepresentation that is made pursuant to a business relationship, in violation of an obligation owed, upon which the plaintiff relies to his detriment. Several courts have also made a list of relevant factors : NJ: Relationship of the parties Nature of the attendant risk Opportunity and ability to exercise reasonable care Public Interest IL: Foreseeability of the injury Likelihood of the injury Magnitude of the burden in guarding against the injury Consequences of placing the burden upon the Vince(/accident victim) v. Wilson(/grandparent of driver): negligent entrustment BLL: A person who enables an incompetent driver to purchase a vehicle may be liable for negligent entrustment Facts: The grandparent helped the boy buy a car, knowing that the boy did not have a license, he failed the drivers test several times, and he had a drinking problem. Boy hit brought suit against the car dealer, and its president trial court entered directed verdicts in favor of car dealer and its president, and submitted the question to the jury against Wilson. Issue: Did Wilson have a duty to prevent the boy from buying the car? Ruling: YES Previous rule of negligent entrustment: Liability arises out of negligent entrustment of a vehicle where the is the owner or has the right to control the instrumentality entrusted Courts apply that rule more broadly now: Courts hold that the fact that a had ownership and control over the instrumentality at the time it was turned over to an incompetent individual is sufficient Based on the Rest. 2nd of Torts 390: One who supplies directly or through a third person a chattel for the use of another whom the supplier knows or has reason to know to be likely because of his youth, inexperience, or otherwise, to use it in a manner involving unreasonable risk of physical harm to himself and other whom the supplier should expect to share in or be endangered by it use, is subject to liability for physical harm resulting to them. Applies to anyone who supplies the chattel. It does not matter in which manner it is entrusted (loans, gift, etc.) The key factor was that the entrustor knew or has reason to know of some reason why entrusting the item to another was foolish or negligent. Evidence indicates that Wilson knew or has reason to know sufficient to make out a prima facie case of negligence: the boy did not have a license, he failed the drivers test several times, and he had a drinking problem

Holding: Should not have directed verdict against car dealer and its president Was right in allowing question to go to the jury against Wilson Notes: Ex: father co-signed when his daughter bought a car. She made all the payments 3 years later, she hurt while driving drunk court found that the father did not have a duty because there are a large # of variables in financing In Vince, the court suggests the combined negligence of the entrustor and the person entrusted. What if the person entrusted is an 8-year old boy? Ex: court found that a car rental company did not have a duty to investigate the driving record of someone renting a car who was sober and had a valid license. Ex: Court refused to make a car dealer liable for failing to check if the person buying the car had liability insurance. NY has found that car dealers are negligent when leasing, but this is a minority rule Sometimes the car dealers will make the lessor pay $1000 as insurance Keys in the ignition: If someone leaves the keys in the ignition, and a thief injures a party while driving it, the owner may be found liable for negligence in violation of the statute Other courts find that the purpose of the statute was not for safety, but to limit the cost of stolen cars Factors to consider for these cases: Where the car was left The time it was left for The size of the vehicle (the potential to hurt people when driven) Guns: Cannot hold gun merchants liable for selling guns to incompetent buyers. This would unfairly regulate merchants. Negligent hiring/retention of employees: Similar to negligent entrustment Easy negligent entrustment cases Employer-employee cases, where the employee hires a bad employer Who is the supplier of the chattel: Look at a broad definition Unless the trustee is a member of the household, most likely will not be liable for giving something to a negligent driver This case is a little outside the norm Reynolds v. Hicks: BLL: The statute making it unlawful to supply liquor to any person under the age of 21 does not allow a 3 rd person injured by an intoxicated minor to sue the social hosts providing the liquor. Facts: A minor drank alcohol at the s wedding, and was then involved in an accident, hitting s brought suit, claiming that s, as social hosts, owed a duty to third person injured by an intoxicated minor Issue: Does a social host owe a third person a duty not to serve alcohol to minors? Ruling: NO RCW 66.44.270: It is unlawful for any person (except a parent) to supply liquor to a minor on his premises or while under his control A violation of a statutory duty of care is not negligence per se, but it may negligence Negligence per se: Conduct amounting to negligence as a matter of law because it is either so contrary to ordinary prudence or it is in violation of a statute. This court has found that a minor can sue the social host s wish to extend it to apply to 3rd persons Because of the differences between a social host and a commercial vendor, cannot apply : social hosts are not capable of handling the responsibility of monitoring their guests the commercial vendor has a proprietary interest the commercial vendor is usually better organized to monitor implications for a social host are widespread and unpredictable social hosts are unaccustomed to the pressures involved.

Therefore, commercial vendor can be held responsible in this situation, but not a social host Statutory intent: It was not enacted to protect 3rd persons because the statute allows a parent to give alcohol to a minor It protects minors against their own injuries, not 3 rd persons Concurrence: Do not agree that the parental exception prevents 3rd person liability 3rd persons are within the statutory class Dissent: Both the social hosts and the commercial vendor would be committing a criminal act, and should be held liable Where the minor obtains the alcohol is obviously the most important step Social hosts are in the best position to know the age of their guests Notes: This case does not really represent a cause of action for an accident, but rather intoxicating a minor If the host would have never gave the minor alcohol, it would have never happened. In the very few states that have ruled in favor of the against the social host, the legislature quickly enacted either complete immunity or strong protection for the social host Broader implications of encouraging danger: When the has encouraged danger without reference to any specific individual, the can be held liable Ex: when a radio show, that was the most popular for teenagers, held a contest to try to catch the DJ, a teenage driver ran someone else off the road and killed the driver. Court found the negligent Ex: A TV show have a vivid rape scene, and when some kids tried to reenact it, the TV show was protected by the 1st and 14th amendments

Landowners and Occupiers


Traditional Rule Definitions: Trespassers: All entrants to the land are trespassers until the owner gives them permission to enter Ex: someone walking in a park after closing time is a trespasser, and the park is not liable for anything O owes no duty 333: O is not liable to a trespasser for physical harm caused by his failure to exercise reasonable care To put the land in a condition reasonably safe for their reception To carry on his activities so as not to endanger them Exceptions: When O knows that persons constantly intrude in some area Children trespassers (339): O is subject to liability for physical harm to children trespassing thereon caused by an artificial condition on the land if O knows or has reason to know that children are likely to trespass O knows or has reason to know that the risk is dangerous to children The children would not discover the consition because of their youth and inexperience The burden of removing the condition is slight comparerd to the risk involved O fails to exercise reasonable care to eliminate the danger or protect the children Licensee: A person who has the owners consent to be on the property, but who does not have a business purpose for being there, or anything else entitling him to be on the land apart from the owners consent. The main class of licensees are social guests. Duty to a licensee: The owner does not owe a licensee any duty to inspect for unknown dangers. On the other hand, if the owner knows of a dangerous condition, she must warn the licensee of that danger. Needs actual notice of the danger Invitee: Includes Persons who are invited by the owner (O) onto the land to conduct business with O; or Persons who are invited as members of the public for purposes for which the land is held open to the public.

Open to the public includes those who come onto the property for purposes which it is held open, even if these people will not confer any economic benefit on the owner

Duty: Duty to invitee: O owes a duty of reasonable inspection to find hidden dangers. O must use reasonable care to take affirmative action to remedy a dangerous condition. Warning: Usually a warning to an invitee will suffice If O realizes that the warning will not remove the danger, then the danger must actually be remedied 342: O is liable to invitees if: He knows or by the exercise of reasonable care would discover the condition, and should realize that it involves an unreasonable risk of harm to such invites, and Should expect that they will not discover or realize the danger, or will fail to protect themselves against it, and Fails to exercise reasonable care to protects them against the danger Duty of Due care: Duty to inspect: O must use reasonable care while inspecting the premises for hidden dangers, even if the danger existed before O moved onto the property Only needs constructive notice of the danger Control over 3rd persons: Reasonable care may require that O control 3rd parties on his premises Open and Obvious Danger: The duty owed invites where the danger is open and obvious: Some courts rule that there is no duty since it is apparent to the invitee. Some courts feel that O is still liable, following 343: O is not liable to invitees for harm from obvious dangers unless O should anticipate the harm despite such obviousness or knowledge Invitee licensee: If the visitors purpose goes beyond the business purpose or beyond the part of the premises that is held open to the public (ex: using a private bathroom), the person will change from an invitee to a licensee See page 104 of Outline Carter (/injured Bible Study Boy) v. Kinney (/hosted the Bible Study): BLL: A social guest is a licensee, and a homeowner has a duty to protect him only from known dangerous conditions Facts: signed up to host a Bible study group at their house their was no contributions for hosting agrees that host received no tangible benefit from invitation was only through sign-up at the church, not to the general public shoveled the driveway at night, and overnight, ice formed. came, and slipped on the driveway and broke his leg had no special relationship with the members of the class Issue: Does a host owe a duty of reasonable care to fix hidden dangers for a social guest? Ruling: NO was a licensee, not an invitee: Historically, there are 3 classes of s: Trespassers: All entrants to the land are trespassers until the owner gives them permission to enter O owes no duty Licensees: Persons who enter with permission are licensees until they have an interest in the visit such that the visitor has reason to believe that the premises has been made safe to receive him Owes duty only to make safe dangers that he is aware of Needs actual notice of the danger

Invitees: Come for a business purpose or as a member of the public if it is open to the public Owes duty of reasonable care to protect against known dangers and those that would be revealed by reasonable inspection Only needs constructive notice of the danger A social guest is a subclass of licensees The invitation was not rendered for any material benefit, and was not extended to the public Does not mean that a person not invited for social purposes is an invitee Invitee only when invitation was rendered for a material benefit or extended to the public See Rest. 2nd of Torts 332: An invitee is either: Public invitee: A person who is invited to enter or remain on land as a member of the public for a purpose for which the land is held open to the public Business visitor: A person who is invited to enter or remain on land for a purpose directly or indirectly connected with business dealings with the possessor of the land Analysis: was not visiting for business agreed that the did not receive any tangible benefit, only intangible participation of did not open it to the public-only for church members

Notes: should not have argued that was receiving an intangible benefit, he should have argued that the ice on the driveway was a dangerous condition that the should have known about Justification for treating a social guest as a licensee (comment h to 330): There is a common understanding that the guest is expected to take the premises as the possessor himself uses them, and does not expect and is not entitled to expect that they will be prepared for his reception or that precautions will be taken for his safety Ex: When a stranger went to a friends church for Bible Study, and hurt herself in the parking lot, the court found that she was a licensee Court found that the cost of having to inspect must be tied to tits commercial business interests Activities: Traditional rule: Licensees and trespassers could not recover for active negligence while they were on the premises Ex: when someone rented a hall, and was hurt when someone moved the piano Modern: 341: Extends liability to licensees for failure to carry on activities with due care iff the occupier should expect that the licensee will not discover or realize the danger, and the licensee does not know or have reason to know of the activities and risk involved. Recreational Use of Land: Almost all states have enacted statutes that limited the liability of O of land used for recreational uses. Goal is to prevent persons on open land from suing for natural dangers on such land or demanding that warnings be posted. Willful misconduct is generally required for liability Heins (/injured Visitor) v. Webster County (/Hospital): BLL: Owners and occupiers have the duty to exercise reasonable care in the maintenance of their premises for the protection of lawful visitors Facts: The went to the hospital, claiming that it was both to visit his daughter, who worked there, and to talk about playing Santa Claus at Xmas claims that it was only to visit daughter slipped on some ice outside the entrance claimed that was negligent Issue: whether the court should abolish the common-law classifications of licensees and invitees and require a duty of reasonable care to all non-trespassers Ruling: YES Policy Reasons for an Against Abolishing Classifications :

Against: Able to predict the common law It is a stable and established system of loss allocation Abandonment would decrease the predictability O would be less able to guard against risk For: The status of their entrance should not determine the duty owed by O A mans safety is not less worthy of protection and compensation because he is not an invitee. It is more practical in an urban society Easier to inspect a small piece of land To eliminate the harsh and unpredictable state of the common law rule Common law is rooted in feudalism The foreseeabilty, rather than the status of the entrant, should determine the duty owed Classifications can be used to determine the forseeability Analysis: Present case illustrates the frustrations of keeping the classifications would recover had he been an invitee O would otherwise be held to a reasonable standard of care modern relationships between persons are not defined by this classification system class still remains for trespassers trespassers rarely brought suit anyway does not mean that O is an insurer of their premises, nor that they should undergo extreme burdens to maintain Factors to consider: Foreseeability or possibility of harm Purpose for which the has entered the premises The time, manner, and circumstances under which the has entered the premises The use to which the premises are put or expected to be put The reasonableness of the inspection, repair, or warning The opportunity and ease of repair or correction or warning The burden Cost, inconvenience Dissent: O will become liable for someone who does not have express permission to be there Public and private institutions and residential homeowners must be more aware Notes: Reasonable Care v. Invitee duty of care: For reasonable care, the O does not necessarily have to know the person is coming Ex: UPS guy For Invitee, must be invited. Courts against abolition of the classes: Reflects a long, careful, judicial history that defines the relationships of a person wishing to recover and the landowner wishing to keep their and the way they wish 23 have abolished the classes, 14 have kept them, the others have not addressed it Reasonable care may still depend on the status of the entrant If you are inviting 15 people over rather than 2, the standard would probably be higher. Landlord and tenant: Traditional rule: Insulates landlords from defective conditions except in the following situations: A hidden danger on the premises which the landlord is aware of and the tenant is not Premises leased for public use Common part of the building under the landlords control (stairways) Premises negligently repaired by the landlord Modern: Court imposes a duty where the landlord has made a promise Lessor has agreed, for consideration, to keep the premises in repair

Landlords promise to repair will forego the lessors effort to repair Landlord retains a reversionary interest in the land and thus might want to keep the repairs up Social policy factors: Tenants may often be financially unable to make repairs The tenant is only renting for short term, thus incentive to make repair himself is small Landlord is receiving $, therefore has the duty Liability for Harm outside the premises: Ex: when a highway driver was hit by another who had been distracted by the bungee jumping activity on the side of the highway Court found that the bungee jumping co. had no duty because there was no foreseeable danger to highway drivers A warning sign may have only distracted the drivers further There are many other distractions along the highway Criminal Activity: s sometimes sue for criminal conduct occurring on the premises Ex: a was assaulted in the common are of an apartment building Court found there was no duty because it is impossible to guard against these perils Posecai (/robbed customer) v. Wal Mart (/store where was robbed in parking lot): Duty to protect against criminal behavior from 3 rd persons BLL: Business owners do have a duty to implement reasonable measure to protect their patrons from criminal acts when those acts are foreseeable Facts: was leaving the store and went to her car to unload-it was not dark out. A robber came out from under her car, held her at gunpoint, and robbed her A security guard was inside the store, but could not see outside brought suit claiming that the store was negligent in failing to provide security in the parking lot Defense: History of crime: Area behind the store was a high crime area, but the store was not known to be one None of the other businesses employed security guards Only 3 robberies in 6 years, and one was in the middle of the night At 13 stores around are, there had been 83 offenses in the 6.5 years Issue: was the criminal act foreseeable such that the store owed the a duty to protect her from criminal acts of 3rd parties occurring on its premises? Ruling: NO Adopt the rule that although businesses are not insurers of their patrons safety, when the criminal act is foreseeable, that the store owes the a duty to protect her from criminal acts of 3rd parties occurring on its premises However, there is generally no duty to protect others from criminal acts of 3 rd parties 4 approaches to determine foreseeability: 1) Specific Harm Rule: O does not owe a duty unless he is aware of a specific imminent harm about to befall on them Courts generally find this too restrictive 2) Prior Similar Incidents Test: established by evidence of previous crimes on or near the premises Recency, frequency, and similarity to the crime in the case Can lead to arbitrary results 3) Totality of the Circumstances Test: nature, condition, location of the land number, nature, and location of similar incidents criticized as being too broad a standard most common approach 4) Balancing Test: seeks to address the interests of both business owners and their customers by balancing the foreseeability of the harm against the burden of imposing a duty on the to protect against the criminal behavior of 3rd persons a high degree of foreseeability is need to overcome the burden

rarely require security in the absence of prior incidents- most important factor Use Balancing Test: Security is a lot of money In high crimes areas, they are usually already economically depressed. A lower degree of foreseeability may only require surveillance cameras or extra lighting, Prior incidences: Only 3 on their premises for 6 years Small compared to the # of people who have been there over 6 years One was during off-peak hours Sams only operated during daylight Foreseeability is slight Concurrence: Feel that the court should have used the totality of the circumstances test It takes all factors into account Notes: This case is both a duty of rescue case and a duty of prevention case Usually courts can find that the should have done something to prevent the occurrence, but should have also done something to rescue the Did the security guard inside the store create a false sense of security, where the relied on the s affirmative action of providing a guard? Once you inform the public that there is security, you must perform Resisting Robbery and Apprehending Perpetrators: Is there a liability for trying to thwart robbers once the robbery has begun? Ex: a robber approached a bullet proof glass, and demanded $, unless she wanted him to kill a customer The store keeper refused to give him $, and he killed he customer The court found that the store keeper had no duty The store keeper should not be faced with such a hard choice Robbers are unpredictable

Intrafamily Duties
Spousal: At common law, spouses could not sue each other The wife was one with the husband Now, with the Married Womens Property Acts, the wife is her own entity Now spouse have the ability to sue each other Parent-Child: Was never treated as a unity However, parental immunity to suit traces far back Claims against a parent for intentional harm are permitted almost everywhere Most controversial cases are where parents negligently inflict harm Broadbent v. Broadbent: Child suing parent BLL: Parental immunity is abolished and a parents conduct is judged by whether that parents conduct comported with that of a reasonable and prudent parent in a similar situation Facts: The Mother left her son by the side of the pool, and went inside to answer the phone He fell in, and suffered major brain damage as a result Wife had personal umbrella insurance indemnity from Northbrook if she is liable Issue: Should the parental immunity rule apply? Ruling: NO Parental immunity does not originate from English Common Law. It is an old American case-law defined doctrine Hewlett: A child could not sue parent for being falsely imprisoned in an insane asylum McKelvey: A child could not sue parent for inhumane treatment from the stepmother allowed by the father Roller:

minor child could not sue father for rape Exceptions to this defined immunity: Parent is acting outside parental role, and as an employer Parent acts willfully, wantonly, or recklessly Child is emancipated Child or parent dies Third party is liable, the immunity does not protect the 3rd party If the tortfeasor is standing in as a parent Parental Immunity in AZ: Overruled in Goller: If the parent has a duty to the world at large, it is not immune from liability Policy Reasons in support of Parental Immunity and reasons why they are rejected : Suing ones own parent disturbs domestic peace Parent most often makes the decision to sue himself Suing ones own parent would create a danger of fraud and collusion Present in all lawsuits Awarding damages to the child would deplete family resources Case will not be brought if not insurance is available Could help pay medical bills Awarding damages to the child may only benefit the parent if he inherits it This is a concern for intestate law, not tort law Suing ones own parent would interfere with parental care, discipline, and control A parent should not be immune from willful, intentional conduct This would still protect the rights of parents to raise their children by their own methods and beliefs, but parents should not have an unlimited discretion The abolishment of Parental Immunity Adoption of Reasonable Parent Standard: Needs to be Objective Cannot be the duty to the world Cases could have nothing to do with being a parent to have a duty to the world Cannot hold a parent liable for negligent supervision Case would revolve around whether the parents lack of supervision caused the injury Almost everything a parent does involves care, custody, and control Reasonable Parent Test: A parents conduct is judged by whether his conduct comports with that of a reasonable and prudent parent in a similar situation Application to the Present Case: A trier of fact may find that the mom did not act as a reasonable prudent parent Concurrence: Agree with Rest 2nd: Parents are liable if their conduct is palpably unreasonable Notes: Must act whether the parent has made a parental act Not whether something went wrong The parent does not have a duty against the world, just the duty to act as a reasonable parent There is a heightened standard for the reasonable person standard as compared to the reasonable person standard If the parent is not acting in the parental role, apply the reasonable person test Third Parties: Most suits arise from a child (through someone) suing a 3rd person, and the 3rd person sues the careless parent Ex: , 20-year old disabled boy, was left alone in a store while his mom shopped. While she was gone, he was arrested for fighting. He sued the mall The mall sued the mom Court rejected the malls claim since the could not have sued his mother directly Harm to the Fetus:

A fetus born alive can sue a 3rd party for harm sustained before birth Did not extend this duty to the mom Would intrude into her privacy Could govern her diet, sleep, exercise, etc. Religious Beliefs: Should parental duties be modified by religious beliefs? Courts will sometimes integrate the religion into the parental standard of care Ex: A reasonable Catholic parent would Cannot threaten the life of the child Impact of Insurance: Umbrella Policy: Protects the insured for sums greater than those afforded by other insurances Insurers are sometimes put in the position of defending the insured who has every incentive to lose Some juries are influence by the amount of insurance available Some insurance companies exclude coverage for Intrafamily suits

Governmental Entities
Sovereign Immunity: At common law, King could do no wrong Became established in U.S. for Governmental Entities Municipal and State Liability: Tort claims v. Constitutional tort claims Government exercises authority over people Can private individuals do the same thing? Government acts as a market participant It interacts with people in the process that does not reflect the exercise of authority over people Therefore, the decision the gov makes may not involve a unique gov function Riss (/girlfriend) v. City of NY(/police): BLL: A municipality is not liable for failure to provide special protection to a member of the public who has repeatedly been threatened with personal harm. Facts: was terrorized for 6 months by her X she sought police help unsuccessfully He threatened her when she got engaged She again sought police help unsuccessfully Next day, a thug threw lye in her face and made her go blind Issue: Ruling: This case involves the provision of governmental service to protect the public generally The amount of protection that may be provided is limited to the amount of resources available and how they may be deployed For this court to determine how to allocate the resources would do so without predictable limits This is different from the predictable allocation of resources for hospitals and transportation Cannot remove sovereign immunity through the judicial process Must be through legislative enactment The analysis is not compared to the compensation for the losses sustained It would be different if the police exposed the civilian to further danger Dissent: Majority is saying Because we owe a duty to everybody, we owe it to nobody. Saying there is no duty is starting with a conclusion Question should be whether there should be liability for the negligent failure to provide adequate police protection There is no fear of financial disaster Les than 2/10 of 1% of citys annual budget goes to tort claims There are limits to the allocation of resources There will not be a lawsuit every time there is inadequate police protection Will need to consider fault, proximate cause, and foreseeability

Police will not have to be everywhere every time Courts have the right to interfere They do it all the time in reviewing administrative practices They are doing two things: Applying the principles of vicarious liability to the operations of the gov No basis for treating policemen differently Public officials can allocate funds by Improving public administration, OR Accept the cost of compensating injured persons Notes: If performing discovery look for: The # of cops they had How they were used Schuster v. City of NY: provided info to the police that led to the capture of a criminal told police that he was being threatened, but the police did not respond was killed 3 weeks after Court found that the government cannot be passive in this situation, it must be active. Sorichetti v. NY: Father had a long history of violent and abusive behavior He threatened to kill his wife, and police sent her home Court found that the police had a duty based on the protective orders, the fathers history, and the officers assurance that action would be taken Police assurance: Generally, there is no duty to provide police protection unless there is a special relationship Elements for a special relationship (from Cuffy v. City of NY): 1) an assumption by the municipality, through promises or action, of an affirmative duty to act on behalf of the injured party 2) knowledge of the municipality that inaction could lead to harm 3) some form of direct contact between the municipalitys agents and the injured party 4) that partys justifiable reliance on the municipalitys undertaking ex: if promise was made in the morning, and the injury happens at night, the party was no longer relying on that promise These factors are no universally followed (but are in Lauer) Municipal Transport: Ex: was assaulted when descending stairs to get to the public-operated train, and brought suit because there was no guard on duty Court found: public transport had no duty to protect someone from a third party when there is no special relationship although would probably find a nongovernmental common carrier liable if the court imposed a duty, this would force the public transport to reallocate their resources Ex: the sued the bus company when the bus driver failed to try to break up the fight on the bus Court held that the bus had the duty because a special relationship existed here 911 call: In NY, both direct communication and reliance by the caller are needed to create the special relationship for a duty Ex: where the caller is not the victim, there was no direct communication, and therefore no special relationship Other custodial relationships: Ex: the police agreed to provide a crossing guard when the regular one was ill The mother used to walk her child across a busy street every day, but then decided it was unnecessary since there was a guard The first day she didnt walk him, the crossing guard was ill, and the police failed to provide one The child was run over The court found that the police had a duty because they had made a promise to provide one Ex: was injured by another car

Police came, but did not identify any witnesses, etc. sued because the negligent actions of the police prevented her from suing someone the court held that there was no affirmative duty on the part of the police because there was no special relationship officers took no affirmative action they did not change the risk to the they made no promise there was no reliance School duty in supervision: Ex: a child ran away from school, and was run over 4 blocks away Court held that the school had a duty to supervise the child Ex: a child was run over after being dropped off at the bus stop Court held that the duty terminated when he was dropped off at the bus stop Ex: a child was released from school on foot, before all the buses left, and was injured Court held that the duty continues when the child is released in a potentially hazardous condition Educational Malpractice: Ex: Student sued the school for failing to teach him to read above a 5 th grade level Court found that cases involving school supervision did not control here: There are many factors that contribute a a students level of education: Home, physical aspects, environment, culture, emotional aspects The court cannot define a duty of care against which the schools conduct may be measured No apparent connection between s behavior and s ability to read Policy: To hold the school liable would expose them to an unlimited amount of tort claims Ex: A NY case Court held that if other professionals are held to a duty to the public they serve, so should the school However, court felt that policy precluded the claim Would interfere with the authority of the school recourse was available through the administrative process Ex: sued school because the school negligently determined that the student was mentally retarded because of a speech defect spent 12 years in school for the retarded followed that same theory that public policy prevented the court from interfering with the schools authority recourse was available through the administrative process Lauer (/father of son who died) v. City of NY(/Medical Examiner): BLL: Facts: 3 year old son died Medical Examiner performed an autopsy, and concluded that the childs death was a homicide from blunt injuries to the neck and brain The police focused investigation on the father After performing a more detailed study of his brain, the Medical Examiner found that it was a brain aneurysm that cause his death Police investigation stopped brought suit for negligent infliction of emotional distress Issue: Ruling: Discretionary Acts v. Ministerial Acts: Discretionary: A public employees discretionary acts (conduct involving the exercise of reasoned judgment) may not result in the municipalitys liability even when the conduct is negligent Ministerial: Conduct requiring adherence to a governing rule, may subject the municipality to liability when the conduct is negligent This is what the Medical Examiners conduct was The municipality is not necessarily negligent, just not immune from suit Must still find a basis for negligence

Duty: The duty breached by a municipality must be more than just a duty that is owed to the public generally 557 of NY Charter: Requires the Medical Examiner to keep records and inform the DA of its findings The intent of this statute is not to protect an individual, through tort claims, from invasion of property or personal interest. The statute was not enacted for the s benefit There was no special relationship between and the Examiner (look at Cuffy factors) Medical Examiner made no promises or assurances to the Medical Examiner assumed no affirmative duty upon which the relied Cannot extend tort liability without limit Dissent: The theoretical class would be a well defined class of people When someone has the knowledge to right the wrong, he should be legally accountable for failing to act reasonably To immunize this type pf conduct would reward gov agents who hide the truth Notes: No defamation case: Medical Examiner has an absolute liability against it Need to allow people to perform this type of job The issue of immunity is separate from the issue of duty Hard to determine when the gov is not acting to protect the public as a whole Could make this argument in many cases NY is trying to limit the # of cases This is really a duty of rescue case should have prevented something from happening to Official Immunity: Judges have immunity, no matter what Ex: judge told it was safe to testify against a gang rape was killed judge was immune from suit Police do not have automatic immunity: Police may be liable if a special relationship can be established Prosecutors: Have absolute immunity only when acting in their official capacity Friedman (/injured in the crossover car accident) v. State of NY(/police): BLL: Facts: Departmental studies had been conducted from 1962 to 1972: Risk of bounce back collisions and rear-end collision exceeded the risk of cross-over There were 3 personal injury claims bunched together on this appeal 1) Cataldo (1972): happened shortly after the studies court rejected that the studies reached a false conclusion 3) Muller (after 1972): decided to change its mind about the studies and construct a barrier 1) Friedman (1986): s car hit another car head on when she was side-swiped and crossed over the median. The DOT had studied the possibility of putting in a median 5 years prior to this accident No action had been taken DOT gave justifications such as funding, time, etc., but offered no evidence to support this Issue: Ruling: Duty: Municipality owes to the public the duty of keeping its streets in a reasonably safe condition This is measured against the proper limits on intrusion into their rules

Cannot override municipalitys decision not to construct a barrier with the jurys decision State has a qualified immunity Once the State is aware of a dangerous condition it must undertake a reasonable study A governmental body may be held liable when its study is inadequate or there is no reasonable basis for the plan It is discretionary analysis This doctrine calls into questions whether reasonable care is the median, or average could a reasonable official conclude that this study was acceptable? Court is saying that even if on the scale of reasonableness its on the low end, thats enough Also, it has a continuous duty to review its plan in light of its actual operation Analysis: Cataldo: There is no liability for reaching decision that a constructive barrier was not needed But, the gov failed to continuously review the plan in actual operation until 1972 However, both accidents occurred after the 2nd review in 1972 s argue that this second report was inadequate, failing to consider the history of the west curve barrier Weiss immunity doctrine: Cannot examine the criteria the State considered-Weiss prohibits this The report was based on reasonable public safety conditions Friedman and Muller: Stare had made a plan to remedy the dangerous condition within a reasonable amount of time Whenthe State knows of a ddangerous condition and does not take action to remedy it, the State can be held liable for resulting injuries Even if the decision was made for the design at the time was safe Friedman, it was too delayed 5 years State did not provide adequate evidence of inadequate funding or other priorities Muller: 3 year delay was unreasonable Notes: The court says that the gov has an absolute duty to keep the roads reasonably safe How is this different from Riss and Lauer? Could say that Police officer should not be responsible for a criminal act, but there is a general duty to keep a road safe The distinction is not entirely clear Both injuries were foreseeable Both could have been prevented by little hassle Do not discuss the merits of the case, need to discuss the duty Cope (/injured in car accident) v. Scott (/person who injured him, but real is the park service) : Discretionary acts BLL: Discretionary judgments regarding where and what types of signs to post are not fraught with public policy considerations and such judgments are therefore not necessarily protected from suit. Facts: was driving north on Beach Drive on a rainy evening was driving south, and slid into the other lane, hitting Beach Drive: Was originally designed for pleasure driving, but became a commuter route Road was slick since it had so much traffic Engineering study of the Park roads: 1986-1988, conducted 5 months after accident this section fell below acceptable skid levels indicated that signs should be posted and that skid resistance should be maintained Over 50% of accidents occurred when wet, while only 18% normally occur in other areas. Park Service priority for repair: 33rd out of 80 sections

record does not specify where signs were located in 1990, a slippery when wet sign was posted on each side sued for: failing to maintain the road failing to post enough warning signs for the curves and slippery when wet claims: action or inaction was discretionary and suit is barred under FTCA Issue: Are the two claims discretionary under the FCTA? Ruling: YES/NO What law applies to the FCTA: Gov is subject to the jurisdiction of the district court when it acts negligently in the scope of its employment Apply the Common law of the state Discretion under the FCTA: Government has the discretion to carry out their duties that are based on social, economical, and political factors. If gov does not have immunity from suit, must still prove that gov acted negligent Two step test to deciding whether discretionary: 1) whether any federal statute, regulation, or policy specifically prescribes a course of action for an employee to follow if employee does not follow this non-discretionary prescription, then there is reason for suit discretion may be applicable if the employee had a choice 2) whether the challenged discretionary acts of the government are of the nature and quality that Congress intended to shield from tort liability the choice is exempt from suit only if it is susceptible to policy judgments and involve the exercise of social, political, and economic judgment Ex: a gov employee may cause an accident through poor discretion, but that is not the type that is exempt Must be grounded in social, political, and economic goals Difficult to assess: All gov activity has budgetary constraints The mere presence of choice does not exempt the gov The issue is not whether the nature of the decision implicates a policy analysis, but whether the decision is grounded in Analysis of Copes arguments: 1) That the gov acts are not discretionary because they involve the implementation of gov policy That implementation and execution of policy decisions, particularly wrt warning the public about hazards resulting from negligence, is never protected Court: No matter at what level the decision was made, the nature of the decision, or the impact on others, the exception only applies where: The question is not negligence but social wisdom, not due care but political practicability, not reasonableness but economic expediency . 2) Gov failed to maintain the road: Park Road Standards Manual: Argues that it sets forth specific prescriptions (step 1) for surface type and skid resistance Court: Does not think the manual is such That the discretion exercised over the maintenance of the road is not subject to policy analysis Court: No regular maintenance would have prevented the road deterioration Could have only been prevented by limiting traffic, using a different material, resurfacing the curve, or milling grooves Determining the right choice would require a balancing of factors/policy considerations: Purpose of the road Allocation of funds Safety of drivers

Inconvenience of repairs The Park placed the repairs in the middle of the priority list Court does not want to second guess this discretionary analysis 3) Gov failed to post enough warning signs for the curves and slippery when wet (that a permanently slippery when wet sign is not enough) Court: Gov confirms that the National Park service uses the guidelines of the Uniform Traffic Control Devices when posting signs The manual is not a substitute for engineering and anesthetic judgment Although posting signs involves discretion, it is not discretion fraught with public policy considerations Would be a balance of policy considerations if: Trying to decide if the signs would take away from the preservation of the environment (for ex: Grand Canyon) Holding: Allegations of negligent road maintenance were discretionary functions, and therefore gov is immune from suit on this claim Allegations of negligent posting of warning signs were not discretionary functions, and therefore gov is not immune from suit on this claim Notes: If defending the gov, would try to immediately motion to dismiss for lack of SMJ Court rejects the categories previous courts found between: Operational acts (ministerial) Discretionary acts The gov probably tried to phrase rules to be discretionary Place a list of factors to be considered Preservation of Historic Appearance: Ex: was held up at gunpoint, and her husband was killed The parking lot was controlled by the National Park Service It was ill lit and had been the site of a prior crime Park Service alleged that the lack of lighting was an effort to return it to its historic appearance Court held that it should have either eliminated all lighting or assumed the duty of providing a safe area Cannot just have some lights Ex: tripped over an antique wooden threshold at a national historic site in AZ claimed that a handrail and a warning sign should have been present court held that the discretion of the Park Service was based on policy because it had gone to great lengths to restore it to its historic appearance The Feres Doctrine (Military Immunity): The court broadened the armed services exception beyond claims arising out of the combatant services of the military to encompass all injuries that arise out of or in the course of military service. Constitutional Rights: Claims against gov officials for violation of a constitutional right usually involve intentional wrongdoing Ex: was the boy of father who beat him, and brought suit against social services for not intervening after numerous complaints based suit on the denial of liberty court held that gov was not liable because it has no affirmative duty to protect individuals from invasion by other private parties

The Duty Requirement: Non-Physical Harm


Emotional Harm
Non-physical interests protected against unintended interference Mental Suffering: Accompanied by Physical Impact: If causes an actual physical impact to , is liable not only for the physical consequences but also for the emotional or mental suffering that flows naturally from it. These are parasitic mental-suffering damages.

Not Accompanied by Physical Impact: Where there has been no physical impact or direct physical injury to , courts limit s right to recover to mental suffering. No physical symptoms: Where there is not only no impact, but no physical symptoms of emotional distress at all, nearly all courts deny recovery Ex: barely misses running over has no physical symptoms, but is distraught for weeks. Few courts will allow to recover for her emotional distress Exceptions: Special circumstance Ex: telegraph companies in handling messages, funeral homes in handling corpses Abandoned: About 6 states have abandoned this rule The at risk : If , by virtue of his exposure to a certain substance, suffers an increased likelihood of a particular disease, may generally not recover for the emotional harm of being at risk Ex: releases toxic chemicals into the water, which causes to be more at risk for cancer. may not recover Intentional Torts: General rule applied only to negligent conduct by If s conduct is intentional or willful, may recover for purely emotional harm with no physical symptoms (tort of intentional infliction of emotional distress) Physical Injury without impact: Where s negligent act 1) physically endangers , 2) does not result in physical impact on , and 3) causes to suffer emotional distress that has physical consequences, nearly all courts allow recovery Ex: being so distraught from something that you have a miscarriage Fear for others safety: If suffers purely emotional distress (without physical consequences) and s distress is due solely to fear or grief about the danger or harm to 3 rd person, courts are split Zone of Danger: If was in the zone of danger (physically endangered but not struck), nearly all courts allow him to recover for emotional distress due to another persons plight. Ex: narrowly avoids running over , and runs over s child. can recover for emotional distress for seeing her child injured. Abandonment of Zone Requirement: A minority of states have abandoned the zone of danger requirement. So long as observes the danger or injury to X, and X is a close relative of , P may recover Different ways court can address emotional distress : Physical impact Emotional harm is not significant unless there is physical harm Zone of Danger: Cannot be harmed emotionally unless in danger Near miss Often take into account the scene of the near miss Busy street, etc. Foreseeability The act is just as negligent as if someone was hit Physical Manifestations: Courts feel that someone who has symptoms is truly emotionally distressed Bystander: Viewing a family member Pure Emotional Distress: Dont need to show an impact, zone of danger, or physical manifestations Dead family members screw-ups have long been looked at under this rule, and have not required more evidence Falzone (/injured) v. Busch(/tortfeasor): BLL: A physical injury may be directly traceable to fright, and so may be caused by it Where negligence causes fright from a reasonable fear of immediate personal injury, which fright is adequately demonstrated to have resulted in substantial bodily injury or sickness, the injured person may recover if such

bodily injury or sickness would be regarded as proper elements of damage had they occurred as a consequence of direct physical injury rather than fright. Facts: Husband was struck by s car Wife was sitting in a car near there, and the s car swerved and started coming towards her after it hit him It out her in fear of her safety She became ill and require medical attention Trial court granted SJ in favor of for wifes claim Issue: whether the may recover for bodily injuries or sickness resulting from fear for her safety caused by negligent , where was placed in danger by such negligence, although there was no physical impact Ruling: YES Old Rule for Non-Impact Cases: Ward: physical impact is necessary upon the to sustain a negligence action 3 reasons: 1) Physical injury was not the natural and proximate result of the negligent act it is not the natural consequences of fright for a person of ordinary physical and mental vigor 2) since it was the first case of that kind in NJ, the consensus of the bar must have been that no liability exists in the absence of impact 3) Public Policy might cause a flood of litigation would be hard to prove with no concrete evidence Why reasons in Ward are no longer valid: 1) the court denied as a matter of law something that is determinable by medical evidence Recognized in Spade: Great emotion may produce physical effects Medical knowledge has greatly expanded 2) three rules of law that are inconsistent with Ward have developed: where a person is injured attempting to avoid a hazard negligently created by another, he may recover for the physical consequences of fright even though the immediate injury suffered was slight and was not a link in the causal chain recovery is allowed for physical injury traceable directly to fright where there is any impact, however inconsequential or slight recovery has been permitted where physical suffering resulted from a willfully caused emotional disturbance 3) the second reason from Ward: should not be based on the opinions of lawyers cannot bar case because of lack of precedent in the State 4) Public Policy: there may be difficulty in proving the link, but that should not bar the suit should be able to prove evidence to the trier of fact in difficult cases, must look to the quality and genuineness of the proof and the medical testimony there is no other state that indicates an excessive # of cases the fear of expansion should not deter the courts from allowing the cause of action Recognize difficulties placed on : If does not know of action, he wont be able to save evidence But if does not bring suit or tell about it for a long time, this will play into the merit of the s claim Notes: could have also had a claim for loss of consortium what is s problem in this case: she has to prove that there was proximate cause for her depression Cardozo: Capriciously showers Applicable to both economical and emotional harm Pure v. Parasitil emotional distress: Zone of danger test: A way of ameliorating the harshness of the physical impact rule Motivations:

deterrence from further negligent acts by other s compensation Stare Decisis: Court states that stare decisis does not apply to torts So many different fact patterns However, people need to be able to rely on the law for guidance This court also looks to the circumstantial evidence The physical symptoms of the emotional distress Headaches, chills, nausea Other courts require more This is different than negligent infliction of emotional distress There is absolutely no physical impact Reasons why courts have disfavored emotional distress claims : Hard to weed out the false claims Hard to quantify the emotional distress Hard to prove the proximate cause Two questions could be raised here: 1) whether any physical impact is required to recover for emotional distress this requirement has virtually disappeared today 2) whether that emotional distress must produce physical injury, consequences, or manifestations court does not rule on this second one since it is apparently satisfied because he became ill Witnessing a family member injury: Although the court noted here that the same car that came towards the was the one that hit her husband, it was not relevant in the courts decision The NJ court had not yet ruled that it was possible for a to claim damaged for emotional distress from witnessing an injury to a family member Car crashed v. Airplane crashes: Ex: s were employees of a car dealership, who feared that a crashing plane was going to hit them Court held that fear only existed for a brief moment, and rejected the claims for infliction of emotional distress No one can go through life without negligently inflicting emotional distress on others. It may be foreseeable that an intense act will cause fright, but it is also foreseeable that it will be short-lived It is not foreseeable how people will react The safety precautions taken for air safety and control are already disproportionate to the precautions taken for auto traffic Dissent felt that the court was being too rough on weak people Airplane passengers: Fear of Imminent harm that does not occur Ex: passenger had severe anxiety when the plane tailspined and then had to do an emergency landing Court held that the made out a prima facie case because of the disturbing experience and the physical symptoms he endured made the claim real Two types of claims with Death: Wrongful death Brought by decedents heirs or estate Survival Claim: Brought by estate All claims that the decedent had at the moment of death Better to just have one claim Just bring claim after the death, because overcoming that near miss would have to be dealt with afterwards, not before death. Hard to determine how much they feared if they are not dead Ex: Could have been reading a magazine when the plane went down Emotional Distress of Victims who realize they are doomed : Most courts have adopted survival statutes: Permit the decedents estate to bring suit for any claims that the decedent may have brought but for the decedents death

Most courts have allowed recovery where was aware of impending death or injury even if the period of awareness was very short Ex: court upheld damage award for the s estate for pre-impact fright, which was proven by 71 of skid marks The fright lasted from 1-2 seconds Ex: evidence showed that the victim was conscious and survived for 10 seconds after the accident Court held that this was not an appreciable amount of time Ex: s negligence in reading a pap smear test led to a failure to detect cervical cancer, and it was too late to save decedent Decedent greatly suffered during the year after diagnosis Some states bar damages for intangible damages if the victim is not surviving at the time of the final judgment (CA) Metro-North Commuter Railroad (/Employer) v. Buckley(/employee): BLL: A worker cannot recover for negligently inflicted emotional distress unless, and until, he manifests symptoms of a disease. Facts: was exposed to asbestos for 3 years for one hour a day he would cover himself with insulation dust Since he attended an asbestos-awareness class, he has feared that he develop cancer Experts testified that there was a 1-5% chance He has gotten periodic check-ups, and there has been no sign of cancer sued under the FELA FELA: permits a railroad worker to recover for an injury resulting from his employers negligence Permits federal courts to create a common law of torts He sought damages for emotional distress and to cover for future check-ups argued that the FELA did not permit a worker to recover if he had suffered no physical harm Issue: Whether the physical contact with the insulation dust amounted to physical impact to make the employee liable for damages Ruling: FELA: Defines injury as: A provision that makes every common carrier by railroad liable in damages to any person suffering injury while employed if the injury results from the carriers negligence Abolishes common law doctrines that previously had limited recovery Does not make the railroad the insurer for all employee injuries Common law of torts: Does not allow for recovery for negligently inflicted emotional distress unless it falls within certain categories: Rest 2nd 924:Permits recovery where the distress accompanies physical injury For distress suffered by a close relative who witnesses the physical injury of a negligent victim For a negligent infliction of emotional distress, where the satisfies the zone of danger test. Recovery for emotional injury to s who sustain a physical impact as a result of s negligent conduct, or who is placed in the immediate risk of physical harm by that conduct 4 reasons why the court Disagrees with Gottshall (2nd Circuit): 1) What constitutes physical impact in the zone of danger test: Gottshall says that it includes a simple physical contact with a substance that might cause disease later Instant: Does not include physical contact that amounts to no more than an exposure Cases that have used the test often involve threatened physical contact that caused, or might have caused, immediate traumatic harm Ex: car accident, gas explosion, trains hitting car, etc. This is a good distinction, such that the court can adhere to public policy This may not be a good distinction because he was exposed to asbestos He was able to quantify the exposure, rather than just claim he was just exposed Hard to define how someone would react to this exposure 2) Gottshall seems limited. Gottshall cited many cases that cite to the zone of danger test.

those who are within the zone of danger of physical impact should be able to recover from fright 3) Common-law courts do not allow a to recover if he is disease and symptom-free Unless the is more likely that not to develop cancer 4) Lack of s evidence of emotional distress: He continued to work with insulation He continued to smoke He did not go see a therapist Contacts with the carcinogen do not separate the meritorious claim from the invalid emotional distress claim Large number of those exposed results in a potential flood of cases Common law does not analyze the genuineness of the emotional harm Rather it has created categories that reflect policy concerns If has physical symptoms Notes: Common Law comparison in Falzone and Metro-North: Falzone: must go claim by claim to determine whether the has a valid claim Metro-North: must develop a general rule that people can look to for liability Falzone implicitly adopts to a zone of danger test Although the court applied the FELA in this case, it draws principles from the common law into the Act, and discusses common law precedent Deterrence: Now, Metro-North has no reason to stop exposing its employees like they have been Other cases of Emotional distress without injury: Exposure: Ex: dumped toxic waste onto landfill. This exposed to carcinogens over a long period of time suffered no injuries, but it enhanced their chance of cancer Court held: In the absence of physical injury, damages for fear of cancer may be recovered only if proves that: 1) s had a duty owed to the , and because of the breach of that duty, was exposed 2) s fear stems from knowledge, taken from medical and scientific knowledge, that it is more likely than not that will develop cancer in the future HIV: Courts have tended to require that the prove that the needle was infected Most courts have adopted the zone of danger test that requires the prove that the needle was infected. Once the proves that the needle was infected, courts still differ in whether to apply objective or subjective standards to analyze the fear Ex: trash collector was stuck by needle that was negligently disposed of by doctor Court rejected the zone of danger test. Did not require an actual showing that the needle was infected Asked whether a reasonably well-informed citizen might fear Court wanted to use tort law to reduce ignorance of AIDS Would therefore not accept the level of the community as the standard of knowledge Court concluded that a was entitled to recover for damages for serious and genuine distress that would be experienced by a reasonable person of ordinary experience who has a level of knowledge that coincides with the information generally available to the public at that time Windows: HIV: A few courts will allow recovery in HIV cases for the window between the event that creates the concern and the results of tests showing that the infection did not occur Pregnant Women: Same for when someone does something negligently to a pregnant women, and she fears that her baby will be harmed or that she will have complications during birth until the baby is born Gammon (/son) v. Osteopathic Hospital(/hospital): Severe Emotional Distress (related to seeing family members injured) BLL: One may recover for negligently inflicted psychic distress not accompanied by physical injury Facts:

s father died in the s hospital when asked for the hospital to send his fathers personal belongings, they sent a bag with a severed leg began having nightmares, and his relationship with his family started to deteriorate he sought no medical attention Trial court: Charged the jury that severe emotional distress is distress such that no reasonable man should be expected to endure it Granted a directed verdict for Issue: Whether has established a claim, in tort, for the infliction of severe emotional distress Ruling: YES A persons well-being is as much entitled to protection as their physical well-being Normal cases allowed for emotional distress: Where the emotional distress is negligently and intentionally inflicted Where the distress accompanies physical injury For a negligent infliction of emotional distress, where the satisfies the zone of danger test. Showing of physical impact, objective manifestation, or special circumstances Discussion of past Cases (over 100 years) addressing this issue : Cases demonstrate a variety of difficulty in dealing with emotional distress with various fact patterns They demonstrate the awareness of the artificial devices used to protect against fraudulent claims and against undue burdens on the s Ordinarily Sensitive Person Rule today: We should rely on the trial process to eliminate the fraudulent claims Traditional tort principal of foreseeabilty provides protection against unduly burdensome liability claims for physic injury Court does not provide compensation for the eggshell physic is bound to foresee physic harm only when such harm reasonably could be expected to befall the ordinarily sensitive person. Define serious mental distress: Where a reasonable person, normally constituted, would be unable to adequately cope with the mental stress engendered by the circumstances of the event Ex: courts have held that recovery is allowed for negligent handling of corpses Here, do not need to extend the exception, only look to the rational used There is exceptional vulnerability of the family of recent decedents makes it highly probable that they will suffer emotional distress from the mishandling of the body. Shows that the should have reasonably foreseen Using that analysis, hospital and mortician should have reasonably seen that would be vulnerable to emotional shock when seeing the severed leg Hold that the jury could reasonably find that s failed to exercise reasonable care Holding is limited to cases of serious mental distress. Notes: Compare Falzone to Gammon: Falzone: required reasonable fear of immediate injury Gammon: refers to an ordinarily sensitive person This isnt just a telegram case What if the leg was sent to someone whos father hadnt just died? Wouldnt they be just as upset. Do both of these help control the expansion of liability? Rule: Eggshell Skull : Allows court to give a more recovery who has actual brittle bones, and who would be affected more Gammon is saying that they will not do the same for the eggshell psyche Negligent Infliction of Emotional Distress: Violation of a duty of care owed to another that occurs when an individual created a foreseeable risk of injury to the other person, which causes emotional distress resulting in some physical harm to that person Similar to corpse example: When a telegram arrives that negligently and incorrectly announces a family member death Courts normally allow recovery When a person has been incorrectly diagnosed as HIV+

Abuse: sued church for abuse from an adult member court denied the foreseeability claim for emotional distress absent some other wrongdoing, it only applies where a special relationship has been established Physical Manifestations: Some courts require the to prove that their emotional distress led to Physical Manifestations. Main Examples: Neuroses, psychoses, chronic depression, phobia, shock Medically diagnosable of objectifiable: Depression, loss of sleep and weight, social and professional dysfunction Show the causal link between the observation and the emotional reaction Ex: s watched their house burn down 1: post-traumatic stress disorder twice a week (diarrhea and heart palpitation), sleeplessness, depression, feeling of despair 2: sleeplessness, gastrointestinal disturbance, upset stomach, nightmares, depression, trouble with driving and working, overall depression court found that they met the required standard Portee (/father of decedent) v. Jaffee (/elevator company): Severe Emotional Distress when witnessing family members injured BLL: One may recover for emotional distress caused by injuries to another person Facts: s son was trapped in an elevator in the building they lived in (owned by Jaffee), and died while they were trying to rescue him Afterwards, mother suffered severe depression, and attempted suicide She required physical therapy and extensive psychotherapy Trial court granted SJ in favor of Issue: whether the court can apply a new standard of negligence to the observation of serious injury or death of a loved one Ruling: YES Progression of Negligence standards: Many times, the law of negligence needs nothing more than the duty of reasonable care Traditional argument for sever emotional distress is that liability unoccasioned by physical impact would lead to mere conjecture and speculation Falzone rejected this, saying that the mere risk of physical harm is sufficient Now, need to determine whether it is only mere conjecture and speculation in assessing a claim where there is neither risk nor physical harm Must look at the s specific interests that were harmed: More than just emotional tranquility It was parental love No loss is greater than that of a loved one The law should have more pity for one who sees a loved on critically injured or killed Dillon Factors that determine whether emotional injury was foreseeable : 1) distance the observer was standing from the injured loved one as the proximity becomes closer, the foreseeability that the will suffer emotional harm becomes greater 2) contemporaneous observation of the accident (as opposed to hearing about it from others) limits the rule to direct observation 3) the relationship between the and the loved one only the most profound emotional interests should be protected Analysis of these factors: 3) relationship most important factor because it limits the liability parental love shows the presence of deep, intimate familial ties that makes the harm to emotional tranquility so compelling in present case, the relationship of mother-son is sufficient 2) contemporaneous observation of the accident equally as important as 1)

to deny recovery because there was no risk of physical harm would impose an arbitrary barrier that bears no relation to injury of his emotional stability does not mean that there should be a cause of action for emotional injuries due to injuries of all close relatives must be circumscribed to negligent conduct which strikes the s basic emotional security the law of negligence may not inflict undue harm on the by imposing an unreasonably excessive measure of liability only a witness at the scene will suffer a traumatic sense of loss that may cause severe emotional distress 1) distance may be of some relevance in demonstrating the closeness of the emotional bond Ex: mother right there is more likely to suffer than the brother who lives across the nation The proximity will increase the likelihood that saw the event This factor is satisfied if meets 2) and observes the accident Additional Factor) the severity of the injury causing emotional distress A cause of action for emotional distress requires the perception of serious injury or death When confronted with accidental death, the expected reaction is shock and fright This is typically not present when the accident is anything less To impose liability for any emotional consequence of negligent conduct would be unreasonable This factor limits liability Thus hold that the following 4 factors are required for negligent infliction of emotional distress : 1) the perception of serious injury or death caused by the s negligence 2) a marital or intimate familial relationship between and injured person 3) observation of the death or injury at the scene of the accident 4) resulting severe emotional distress

Notes: s problems: there is a separate claim for perception and the aftermath of losing her son loss of consortium or wrongful death, and bystander recovery most bystander claims are not near misses- it actually happened They have to be separated out because there was no claim for bystander recovery in the past There used to be only recovery for lost wages Also, wrongful death acts continue to say that only pecuniary damages Pecuniary loss for a child is nothing today Loss of consortium for a parent-child relationship is not recognized in many courts Therefore must sue for bystander recovery when a parent loses a child Where liability is not found using the Dillon-Portee factors : Ex: witnessing a hotel fir on TV, where knew her husband was staying She did not see him on TV Court held that had failed to come within the sensory perception requirement Additional Portee element: Ex: saw his mother hit in a car, but she was only slightly injured Court found that the proper test was whether a reasonable person would believe that his mother would be seriously injured in the type of accident that occurred. Ex: reasonably mistakenly thought that her child had been badly injured in a car accident, but it ended up being another kid Mother died the next day from trauma Court denied recovery Cannot extend liability to everything-everyone experiences possible disturbances daily Distress based on mistake will vary with every person Cannot extend the circle of liability-it would be unreasonable to expand the class of person to whom the would be liable Ex: had been incorrectly and negligently informed that her son had been killed. She did not know of the mistake until 2 days later Claims were rejected because her reactions (crying, trouble eating and sleeping, and some medication) were inadequate. CA has narrowed its decision in this area

Ex: mother, who was nearby, neither heard nor saw her child, but was told about it and rushed to the scene to find her childs bloody and unconscious body Court looked to the Dillon factors as defining elements Absent exceptional circumstances, recovery should be limited to relatives who live in the same household, parents, siblings, children, and grandparents of the victim Court found that the viewing of the consequences was insufficient Many states have not gone as far as Portee and Dillon Ex: NY. Require that members of the immediate family be in the zone of danger in order to recover for witnessing injury of the family member Mitigates the possibility of unlimited recovery Also, the emotional distress has to serious and verifiable Many states have gone further than Portee and Dillon Ex: close relatives came by the scene of the accident shortly after it happened Court upheld claims for emotional stress because they were caused by observing the injury shortly after it occurred and before a substantial change in the injured persons condition or location Ex: However, in another case, where the mother did not hear about her sons accident until 4 hours after, and did not see his body until the day after, the court dismissed the claim Unmarried couples: Courts that find there is no chance to recover: Base ruling on the 3 things: 1) the state has a strong interest in the marriage relationship policy favoring marriage is rooted in the basis for defining the R & R in society married couples are granted significant rights 2) such claims would impose an undue burden on the courts to establish whether the relationship was stable and significant 3) need to limit the # of persons to whom a negligent owes a duty of care Courts that find there is a chance to recover: Need to look to factors such as: Duration of the relationship Common contributions Mutual dependence Same household Etc. Johnson (/mother of abducted child) v. Jamaica Hospital (/Hospital): Severe Emotional Distress indirectly (related to seeing family members injured) BLL: There is no cause of action for indirect psychic injuries unless the was within the zone of danger, and their injuries resulted from the contemporaneous observation of serious physical injury or death caused by the s negligence. Facts: s daughter was born at hospital. went home after giving birth, but her daughter remained behind for further tests when she went back a week later to get her, she was missing (same day as 2 bomb threats) while she was missing, mother brought suit for emotional distress daughter was returned about 4.5 months later brought suit against the hospital, claiming that the hospital owed her a duty to properly care for her child, and that it should have been foreseeable that an injury to the daughter would cause the parent emotional distress. Issue: Does the hospital owe the a duty of care for indirect physic injuries? Ruling: NO Assuming all of s complaint to be true, the failed to state a cause of action Bovsun: Damages may be recovered for indirect physic injuries in limited circumstances If was within the zone of danger, and If the injuries resulted from contemporaneous observation of serious injury or death There was no foreseeable duty to the parent: Kalina (where court found that there was no duty): Parents gave explicit instructions that their son was to be circumcised on the 8 th after he was born (Jewish religion)

The hospital did it on the 4th day Court found that the parents were interested bystanders to whom no direct duty was owed The law demands the equation to be balanced The damaged be able to point a finger of responsibility at the owing, not general duty to society, but a specific duty to him s in accepting a relationship with the baby assumed a risk of liability of a tortious act they did not assume any risk of liability that their acts might violate the personal sensitivities of others Analysis of instant case under Kalina: The direct injury of abduction caused by the s negligence was sustained in the action by the infant The foreseeability that emotional distress would result would not establish a duty to the s Policy: To permit recovery would be to invite an open-ended liability for indirect injury suffered by families where there is negligent care or treatment. Johnson and Lando (Prior holdings involving duty to indirect injuries) : They do not apply here Johnson: hospital negligently sent a telegram to the notifying her of her mothers death, when she had no actually died Lando: hospital negligently failed to locate a deceased body for 10 days, where it was found in an advanced state of decomposition So the court is saying that taking care of the dead body is important, but not a live child Each of those cases presented exceptional circumstances where there was a liability for emotional distress: A duty to transmit truthfully information concerning a relatives death or funeral. In this case, there was no duty. This is in accord with the majority rule in this country . Dissent: Prosser: The business of the law to remedy wrongs that deserve it, even at the expense of a flood of litigation. Also, a flood is unlikely here because the interference of custody was not a common occurrence There would be no further burden on the , because they would not have to change their hospital procedures Cannot assume the burden would be so great such that the court rules out the possibility of liability Notes: This case is consistent with other NY cases It excludes damages that would otherwise be available if the facts were a little different Foreseeability is a necessary element for liability, but not sufficient Similar Cases: Ex: dropped mother off at nursing home, and told them she had Alzheimers she wondered off court refused recovery for emotional distress because the s were not in the zone of danger and did not witness the event in person Ex: parents gave their child the incorrect dose of medicine because the pharmacy told them incorrectly Court denied recovery for emotional distress because the parents were not the patients Ex: An anesthetist had a machine, and had it fixed. It was fixed wrong and he killed a patient Does the dentist have a claim for emotional distress? Court found there was no duty Maybe if the machine would have blown up, there would have been Mother giving birth: Usually viewed as a special case Mother is not a direct victim, but she is also not a bystander because she is so close to the event Mother has an action whether she is conscious or not Why didnt the court look at it this way for the Jamaica case? The baby was born, and then the mother had left Damage to Property: Ex: sued because a garbage truck ran into his house, destroying his car and his art Court refused recovery for emotional distress by loss of property is taken care of through compensation for economic loss

Additional burden on the would mean increased taxes for city Damages to the house: Ex: s built house, and it was destroyed by flooding Court held that they could recover for ED upon a showing that a reasonable man would be unable to adequately cope with the mental stress caused by these circumstances. Many states reject this view Feel that the contractor, insurance, etc. should provide for such damages if they wanted to Pets: Most courts find that pets are personal property, and distress from witnessing damage to personal property does not give rise to an action Negligent Interference with Consortium: History: Before the MWPA, the wife had no claim for damages for loss of consortium, and the H had only a claim for loss of servancy. After the acts, the wife had a claim for loss of consortium, and the H no longer had a claim for loss of servancy, but only loss of consortium. Loss of consortium is now almost routinely allowed in various tort actions It does not necessarily apply to other relationships (parent-child, etc.) Non-physical Injuries: Some courts have extended loss of consortium to Non-physical Injuries. Ex: defamation of one spouse, which deteriorates the relationship Court ruled that since LoC is no longer based on the deprivation of services theory, there was no reason to require physical injury Parent-Child Relationship: Most courts allow parents to recover for the loss of companionship with their child Few courts allow the child to bring suit for loss of companionship with their mom or dad

Economic Harm
Traditional Rule: Where tortiously causes physical injury or property damage to X, but only pure economic loss to , may not recover anything Ex: A ship owned by damages a dock owned by X. , owner of a different ship, is required to dock elsewhere and must pay extra labor and docking costs. may not recover losses from . Rational: fear of open-ended liability Modern Rule: Most modern courts no longer apply a blanker rule of no liability for pure economic loss. Courts are most likely to award recovery where: 1) the injury to was relatively foreseeable 2) relatively few s would be permitted to sue if liability were found for pure economic loss; and 3) s conduct is relatively blameworthy Ex: , a railroad, negligently causes a fire. , airline with nearby operations, is forced to close for 12 hours and lose business. may maintain its suit, because it was part of an identifiable class who knew or had reason to know that it was likely to suffer damages from its conduct. Nycal Corp. (/mother of abducted child) v. KPMG (/Hospital): Pure Economic Loss test applied for accountants BLL: The potential liability of an accountant to noncontractual 3 rd parties is limited to those who can demonstrate actual knowledge on the part of the accountant that the 3rd parties would rely on the accountants report Facts: prepared an audit of Gulfs 1990 financial statements However, the financial statements were the responsibility of Gulf Gulf included those financial statements in its 1990 annual report, which was made available to the public Later in 1991, entered into discussions with Gulf to purchase a large sum of shares, and Gulf gave the its annual report purchased stock, which gave the operating control of Gulf learned of the transaction a few days prior to closing Gulf filed for bankruptcy in 1993, which made the s shares worthless filed suit against , claiming that the report materially misrepresented the financial condition of Gulf. Issue: Is an accountant liable to a 3rd party that it was not in privity with, and where it was not foreseeable that the 3rd party would rely on its report?

Ruling: NO Case of first impression Apply one of three tests from other jurisdictions: 1) Forseeability Test (Constructive Knowledge): derived from traditional tort law concepts An accountant may be held liable to any person whom the accountant could reasonably have foreseen would obtain and rely on the accountants opinion, including known and unknown investors This test is generally disfavored Apply traditional tort law to personal injury cases, not third party pecuniary loss The auditor prepares the reports from information provided from the client, and the client controls the finality To apply this standard would open up the to endless litigation to an indeterminate class Problems: There is no cost to consumers to get the report Because it is a free report, the accountant cannot charge the consumers to cover for liability Hard to price an insurance policy when you dont know how many potential s are out there 2) Near-Privity Test: limits an accountants liability exposure to those with whom the accountant is in privity or in a relationship sufficiently approaching privity an accountant may be held liable to noncontractual third parties who rely to their detriment on inaccurate financial report: 1) if the accountant was aware that the report was to be used for a particular purpose, 2) on which the known party was intended to rely, and 3) if there was some conduct on the part of the accountant creating a link to that party, which evinces the accountants understanding of the partys reliance. First 2 elements have analogs in instant case law, and the 3 rd does not 3) 552 Restatements Test (Actual Knowledge or Intent- not constructive knowledge): this comports most closely with the liability standard for professionals Intent v. Actual Knowledge: The accountant must intend that the information be used in a certain way, and he must have intended that because he had actual knowledge of the investment the tort of negligent misrepresentation committed in the process of supplying info for others: 1) One who supplies false information is subject to liability for pecuniary loss caused to them by their justifiable reliance if he fails to exercise reasonable care or competence in obtaining or communicating the information. The liability is limited to Loss suffered A) by the person or one of a limited group of persons for whose benefit and guidance he intends to supply the info or knows that the recipient intends to supply it; and limited group of persons: not required that the person be identified or known. It is enough that the maker of the representation intends to reach an influence either a particular person or class of persons it is not enough that the maker merely knows of the ever-present possibility of repetition. B) through reliance upon it in a transaction that he intends the info to influence or knows that the recipient so intends. Courts have applied this test as limiting the potential liability of an accountant to noncontractual 3 rd parties who can demonstrate: actual knowledge on the part of the accountants of the limited group of potential third parties that will rely on the report, as well as the actual knowledge of the particular financial transaction that such information is designed to influence. Note: the knowledge is to be measured at the time the report is published (not years from then) Analysis: s argue that using 552 will reward accountants who remain unaware but 552 will not excuse an accountants willful ignorance of info that he should have been aware of facts show that: did not know or intend that the would rely on the report

Notes: Approaches to accountants liability: 1) Forseeability Test: only a few states follow this rule it allows for more expansive liability 2) Near-Privity Test: NY approach Requires more than notice from the relying party to the accountant Ex: A single unsolicited phone call from to , made after the had completed the report, but before the final report had been prepared, was not enough. 3) 552 Restatements Test: Almost states follow this approach Ex in applying this test: 1) Client asks to prepare an audit to show to Bank B. Client does, and rejects it. Client shows it to Bank A instead, who gives them the loan. Bank A loses the money, and sues . owes no duty to Bank A. 2) Client asks to prepare an audit to show to an unidentified Bank. owes a duty to any bank they choose, even if the client was originally planning to go with Bank X, but goes with Y. 3) Client asks to prepare an audit to show to Bank B for a loan. Instead, B decides to buy an interest in the client. The client soon collapses and B loses everything. owes no duty to Bank B. this will probably change over time, since the intention was still the same 4) Federal Securities Law: SEC Bily Case: Investors claimed reliance on financial statements s prepared for a computer company Court found that the investors misjudged a # of factors, including product, market, etc. Investors should be encouraged to rely on their own prudence and power. Imposing a duty might lead to reduction in auditing services or increased prices Attorneys and Clients: The duty of care the attorney owes to a client Meeting filing deadlines: When attorneys fail to file within the SoL Many times the client must be able to prove that they would have had a good chance for success in order to claim malpractice Making strategic choices: Courts are not likely to second-guess attorneys decisions unless it lacked any plausible justification An expert is usually needed to show the jury the standard an the deviation Recommending settlement: Attorney may be liable if he gives advice to settle for a claim for too little Criminal Cases: Usually if he has been convicted of a crime, he cannot sue his defense attorney for malpractice without proving he was innocent of the underlying crime Problems when retrying a criminal case in a civil damage Emotional Distress: It is unusual for a client to recover for emotional distress Other courts suggest that when the attorney is retained for non-economic purposes (marital dissolution, adoption) damages may be foreseeable and recovered Attorneys and Third Parties: The duty of care the attorney owes to a third party When an attorney messes up a will, the attorney may be liable Ex: notary drew up the s will that was supposed to leave it all to him, but it left it to all the relatives She recovered for the difference Ex: Will was invalid because of the RAP.

did not prepare the report for the s benefit was an unknown, unidentified, potential investor was not aware of the transaction until after the purchase did not intend to influence this transaction did not know Gulf would use the report for the transaction the report was prepared for inclusion in their annual report, not to assist shareholders

Court felt that it would not impose an undue burden on the attorney, but the RAP was too difficult to apply that in this case Small # of states requires privity in will cases Duty is owed only to the client It is a difficult claim because the entire purpose of a will for the third party beneficiary is to get something for them Duty to client is discharged when the lawyer says legal, valid, and binding. Courts appear willing to extend duties to non-clients when the client has asked the attorney to provide info to the other side or to prepare docs for a deal Ex: when preparing a title report, offering statement, or opinion letter, it is the kind of info that an attorney knows or should know would influence 3rd parties Other Professionals: Public weigher: If the weigher asserts that something weighs more that it did, and the buyer relied on that for resale Court found that even though the parties are not in privity, the weigher knew that the end and aim of the transaction was to inform the buyer of the amount to be paid. Drug-testing company: Employer fired B because the drug Co. told the employer that he failed the drug test Court found there was a duty of care because the company knew that its actions would affect the group of workers being tested. Physician: Court found that a physician could be sued for understating the condition of a s medical condition, which led to a smaller settlement There is a duty to speak carefully if one speaks at all in this relationship People Express Airlines (/airline) v. Consolidated Rail Corp. (/had explosion): Pure Economic Loss loss to another business BLL: A who has breached its duty of care to avoid the risk of economic injury to a particular or identifiable class of s may be held liable for actual economic losses, aside from physical damage, if the knows or has reason to know they are likely to suffer such damages from its conduct Facts: A fire began in the freight yard of the when ethylene oxide escaped from a tank car during a coupling operation and ignited. s were aware from prior experience that ethylene oxide was a highly volatile substance when the fire occurred, the s helped determine how much to evacuate s were evacuated and prohibited from using their terminal for over 12 hours No physical damage to the terminal However, s claims that they lost $ in: ticket sales cancelled flights fixed expenses during 12 hours Issue: Whether a s negligent conduct that interferes with a s business resulting in purely economic loss, unaccompanied by property damage or personal injury, is compensable in tort Case of first impression Ruling: Physical harm v. No property damage: If there is property damage, the can recover for economic losses, no matter how slight the damage is may be liable for all proximately caused harm, including economic losses A per se rule has developed over the years to bar recovery for economic losses if there is no property damage Rest. 766C: nonrecovery for economic losses absent physical harm Reasons why courts have obeyed this per se rule: Limits damages Often the issue of causation Requirement of physical harm as an element of proximate cause Physical harm functions as link to the s negligent act and the s economic damages Fear of fraudulent claims, mass litigation, limitless liability, liability out of proportion to the s fault If the court can see the physical impact, there is clear evidence of damage and reason for recovery Similar to emotional harm cases

Even where the s have sustained physical harm, courts have found that the is not necessarily responsible for all the consequences. Courts new attitude: Some limitation is required is liable only for the harm that he proximately caused also limited to the duty owed the Fairness and public policy override the fear of fraudulent claims Pure economic losses incurred by innocent victims victims who may not be able to recover their losses Portee To overcome fraudulent claims: Must apply traditional concepts of duty and proximate cause to the facts of each case It will not create an opening for all s to recover The tort system is deigned to accomplish certain social objectives Allow methods of redress without overriding public policy Deterrence: Imposing liability discourages others from similar behavior safer products shifts costs of dangerous activities to those who are able to bear them Two methods of analysis: 1) Exceptions to the rule for nonrecovery: Rationalization for exceptions: The element of foreseeability is a more appropriate standard to determine liability than the per se rule. If the knew or should have known of the particular consequences of his negligence, including the economic loss of a particularly foreseeable , it is dispositive of the issues of duty and fault. The Special Relationship exception: Many cases involve a negligent misrepresentation Courts have based the finding of liability because of the special relationship between the tortfeasor and the foreseeable who relied on the quality of work or services of the to their detriment There was a duty of care because the s were particularly foreseeable, and the injury was proximately caused by the s negligence. This exception has been extended to auditors, weighers and telegraph companies If the knew or should have known that a particularly foreseeable class of s would be economically harmed, the may be liable Public Use: Where a s business is based on part on a public right, the has the ability to recover for purely economic losses Ex: where a fishing boat pollutes the river where all the fisherman get their fish to sell Pecuniary losses suffered by those who make direct use of the resource are particularly foreseeable because they are so closely linked, through the resource, to the s behavior Synthesis of cases: Based on these exceptions, can lay foundation for new rule that allows recovery 2) Traditional approach of foreseeability relating to duty and proximate cause (one court chooses) Particular knowledge of the economic consequences has sufficed to establish duty and proximate cause in contexts other that those already considered. Ex: a contractor who undertook work for the owner of the building had a duty to the tenants to complete the construction on time to avoid resultant economic losses Foreseeability v. Identifiable: Forseeability should be found within the general to particular range Broad view would support the rule that liability and foreseeability are directly proportionate to one another The more particular the foreseeability is, the more liable An identifiable class of s is not simply a foreseeable class of s Ex: persons traveling near the scene of an accident are a foreseeable class of s, but their presence is fortuitous and unpredictable The class would not be ascertainable

Could lead to an unlimited amount of s Ex: if there was a bridge operator on the highway, and people couldnt pass on the bridge and pay the toll, he would have a claim of action, but the people on the highway would not He is ascertainable, but the people on the highway are not since thousands of people pass over there every day An identifiable class must be particularly foreseeable in regards to the following: Type of persons Certainty or predictability of their presence Approximate #s Type of economic disruptions expected This is a heightened foreseeability claim that is targeted towards fixed s Its a fact-smothered inquiry Boundless liability: Courts must use notions of fairness, common sense, morality, and public policy to draw the line Facts of this case that lead court to decide that there is a cause of action : Close proximity of the two parties Obvious nature of s operations and foreseeability of economic losses resulting from an accident and evacuation s actual or constructive knowledge of the volatile chemical the existence of an emergency plan which called for a nearby area to be evacuated Do not suggest that actual knowledge of the economic loss is necessary, jut the particular forseeability Notes: Examples of recovery/nonrecovery for economic loss : Ex: city of NY tried to recover for losses after a blackout Court gave damages for looting, but did not for emergency wages Ex: tenants of a commercial building sued utility company when the power went out and forced store closure during a crucial week of sales Tenants had no direct contract they paid their landlord Court found that there was no duty of care The tenants were not a sufficiently narrowly defined class Ex: what if a hazardous enterprise was located somewhere, and someone else moved in next to him The tenants that move in are within the class of identifiable s If the damage was done before they moved in, may be different Ex: building was evacuated was a large firm suing to recover for economic damages including the relocation of their offices court denied recovery this case is different than a case where the whole block was close off, and all those stores lost revenue from the closure Ex: the time charterers of a boat were denied recovery for the loss of use due to the s negligence Court: There was no wrong done to the Cannot charge the with the full loss when there is no one bringing a claim Cannot just say that if the boat owner would have brought suit, the owner would have paid out the charterers. Ex: s barge negligently destroyed a bridge that was the only way to access 6 stores of s The court denied recovery because it was concerned about the disproportionate liability for the negligent act committed Ex: when a hockey player was injured, the team wanted to recover for the $ needed to recruit a new player Court: At common law, used to allow a master to recover for the injury to his servant Although a negligent wrongdoer should be responsible for his wrongdoings, it would place an undue burden on him to impose such a sever penalty for one act of negligence Similarly applies to a situation where the president of a company is injured, and the company suffers losses even though someone else filled in Net effect of the economy not affected: Individuals feel loss, but economy doesnt An Economic Loss Rule:

Some courts will apply contract law instead Ex: Performance contracts, professional services, construction industry The Economic Loss Rule was designed to prevent disproportionate liability to allow the parties to allocate risk by contract Contracts: designed to enforce the agreement Torts: designed to protect citizens by imposing a duty of reasonable care In the construction industry, need to allocate risk by contract to allow future business activity This view is rejected by Florida The rule of physical harm is analogous to the rule of impact for emotional distress cases Emerson (/Injured patient) v. Magendantz (/Gyn): Pure Economic Loss Negligent Sterilization BLL: Recovery may be allowed for the negligent performance of a sterilization procedure Facts: s had one child, and then, for economic reasons, decided not to have a second sought to have a sterilization procedure performed, and doctor performed it on Jan 10, 1991 On May 31, 1991, she discovered she was pregnant, and had a baby in Jan. The baby was born with congenital problems had a second procedure performed claims: has suffered physical pain and required additional invasive medical treatment have suffered mental anguish have lost wages and earning capacity due to pregnancy they have to spend money for the medical care and maintenance of new baby Issue: Is there a cause of action under RI state law when a physician negligently performs a sterilization procedure and the patient becomes pregnant and delivers a child from that pregnancy? If so, what are the damages? Ruling: Is there a cause of action under RI state law when a physician negligently performs a sterilization procedure and the patient becomes pregnant and delivers a child from that pregnancy? YES First impression in RI Only one state court has declined to give recovery under state law whether it was the man or woman This court is persuaded by the overwhelming majority and agree What are the damages? Three general types of remedies: Limited Recovery: 30 jurisdictions follow this usually grant compensation for: the medical expenses of the ineffective sterilization procedure costs of the pregnancy the expense of a subsequent sterilization procedure loss of wages medical expenses for prenatal care, delivery, and postnatal care sometimes emotional distress for the unwanted pregnancy and loss of consortium Child-Rearing: A number allow for the cost of raising the child Some balance that cost against the benefits derived from the parents (economical and emotional) from having a healthy child (3 states) Full Recovery: 2 jurisdictions allow for full recovery without having to offset it with the benefits derived from having a healthy child these courts apply traditional tort principles damages that are reasonably foreseeable and that would result from the negligent performance of the sterilization procedure Offset: 920: recommends to offset

some court (Wisconsin) refuse to offset both emotional and economical because they are insignificant This court adopts the limited recovery without any offset: It is impossible to determine whether a healthy child will benefits its parents at the beginning The child may grow up to be the President, and it may grow up to be a criminal Costs cannot be calculated Public Policy: This state would preclude the granting of rearing costs for a healthy child whose parents have decided against adoption By deciding not to give the child up for adoption, the parents obviously consider the benefit of retaining the child to outweigh the economic costs of child rearing No recovery for emotional distress arising out of the birth of a healthy child If the child was born with congenital defects, RI would follow Florida law: Special medical and educational expenses beyond normal costs should be allowed When a physician knows of the is on notice (in performing the procedure, through standard practice knowledge, or stats) that the parents have a reasonable chance of giving birth to a handicapped child, the doctor should have to pay for the expenses of rearing the child These damages would be offset by any governmental help Parents should be allowed emotional distress damages Concurrence/Dissent: Agree with the first question, disagree with the second Most states follow the 920: Permits full recovery for all damages proximately resulting from the doctors negligence, while letting the jury reduce the damage award by offsetting what is proven to be the benefit Should not force the patient to have to give up the baby for adoption in order to fully recover Any normal woman would have a hard time with this Disagree with the public policy view: Better to allow the parent to recover for the cost of rearing a child if it is a result of negligent care Public policy cannot support an exception to tort liability when it would impair a constitutionally protected right Parents have the right of privacy to use contraception to limit the size of their family Notes: In these cases, there are potentially 3 types of s Mother, father, and child Why the limited recovery rule does not help the parents of an unhealthy child The doctors negligence did not proximately cause the problem Baby was born with birth defects, despite not being sterilized. Is the birth defect relevant, or should the argue simply for procreation autonomy (the right to choose) Different reasons for choice Financial They may think they will not be good parents Mothers and babys health For cases of an unhealthy child, the claims are usually brought for incorrect medical advice to encourage having the baby Mitigation of damages: Rest. 2nd 920: Requires an offset for emotional benefits of having a kid Should recovery depend on whether the told the surgeon the reason for having the surgery? Ex: when a mother wanted to get the surgery done for fear of health problems with the baby, and the baby was born healthy, the court did not allow her recovery Wrongful Life: A medical malpractice action brought by the parents of a child born with severe birth defects against the doctor, claiming that but for the doctors negligent treatment or advice they would not have given birth to the child Courts usually deny this claim Problems with this claim: Court would be saying that some lives are better than others, and some are not worth living at all How can court asses the worth of not living at all

Different than assessing the difference between someones life before an accident, and after an accident Generally, a child cannot sue his parents for wrongful life Claims by other relatives: Claims are generally not extended to relatives of the child other than the parents Baby not born: Suits can also arise when a baby is not born due to the Dr.s negligence Ex: bad advice about a possible genetic disorder Wrongful Living: When a Dr. cannot obtain informed consent from the patient who is on his deathbed Courts generally do not alow recovcery

Causation: Actual and Proximate Cause


Generally, must show that both exist: 1) Cause in Fact: Whether the actual cause has been established Need to tie the s conduct to the s harm in a scientific or physical way But For: Scientific, empirical However, Often there is no empirical data 2) Proximate Cause: Legal Causation links the harm suffered by the with the s negligent act

Causation in Fact
Basic Doctrine General: must show that s conduct was the cause in fact of s injury Causation: The aggregate effect of preceding events that bring about a tortious result The causal connection between the actions of a tortfeasor and the injury that follows But for test: s conduct was a but for cause of s injuries Had not acted negligently, s injuries would not have resulted Joint Tortfeasor: There can be multiple but for causes of an event. 1 cannot defend on the ground that 2 was a but for cause of s injuries As long as 1 was also a but for cause, 1 is viewed as the cause in fact Concurrent causes: Sometimes s conduct can meet the cause in fact requirement even though it was not a but for cause Where 2 events concur to cause the harm, and either one would have been substantial to cause the same harm without the other Each one is a cause in fact of the injury, since it would have been sufficient to bring the injury about Ex: s truck sparks causes a forest fire, which combines with another fire. Either fire would have caused damage to s property. Therefore, both are causes in fact, even though not a but for cause. Multiple Fault: If can show that each of 2 or more s was at fault, but only one could have caused the injury, the burden shifts to each to show that the other caused the harm. Ex: Both s shoot their guns simultaneously, and is hit. If neither can prove it was the other one, both are liable. The market share theory: In product liability cases, courts often apply this theory If cannot prove which of 3 or more persons caused his injury, but can show that all produced a defective product, the court will require that all s pay a percentage of s injuries which s sales contributed to the market of that product at the time of the injury. Ex: When someone takes drugs that are produced from 200 manufacturers, and she develops cancer years later. Each manufacturer will be liable. Exculpation: Courts are split on whether a should be allowed to exculpate itself by showing that it did not make the particular items in question National Market Share: In determining the market share, courts usually use a national, rather than local, market concept.

No joint and several liability: Courts adopting the market share approach often reject the joint and several liability They allow to collect from any only that s proportionate share of the harm caused. If only sold 10% of the products, will only be responsible for 10% of the s damages Socially valuable products: The more socially valuable the court determines the product to be, the less likely it is to apply the market share doctrine. (Ex: when the product is a vaccine) Increased risk, not yet followed by actual damage : Where s conduct has increased the risk that will suffer later damage, but the damage has not yet occurred, most courts deny any recovery for that later damage unless he can show that he is more likely than not to occur eventually. But, some courts will allow recovery, minus the likelihood that the damage will occur Ex: if has a 20% chance of contracting a disease from a doctors procedure, courts will discount the recovery by 80% Indeterminate : Where its clear that injured some people, but cannot determine who Often in a massive tort case Class actions Stubbs (/Typhoid Fever Victim) v. City of Rochester (/Municipality): Causation in Fact-Basic Doctrine Substantial Factor BLL: In a negligence action, the fact that the s condition could have been caused by other factors other than the s negligence does not obligate the to prove that non of those factors was actually the cause. Facts: Facts of s history: supplied drinking water (Hemlock) and firefighting water (Holly) for the city Due to the citys negligence, the two systems were intermingled in May near the Brown Street Bridge, but it was not discovered until Oct. worked at a factory located one block from the Brown Street Bridge in which the Hemlock water was supplied He only drank Hemlock water at his house, and at work he drank the water out of his own cup He did not leave the area in all of 1910. In September, became ill Evidence/Testimony: Several witnesses during July, Aug, and Sept testified that it smelled and looked funny Dr. Goler (physician and health officer) went to houses in the neighborhood, and found the same thing He found that it contained large amounts of chlorine The district found the same thing The day after the contamination had been discovered, the doctor concluded that the outbreak of typhoid fever was due to the contamination in the water Dr Dodge, a bacteriologist, made a similar analysis of the water, even though he did not find any colon bacillus s physicians testified that he contracted the disease from the water Other resident of Brown Street drank the water in the fall and became ill Same with 57 other witnesses Court found that the did not have to call these witnesses, and just stipulate that they would testify in favor of the Statistics: From 1901-1910 Of the 223 cases, 180 were in Aug-Nov 1/3 of those 180 were in that district s claims: That even if it is liable for the negligence in containing the water, that The evidence fails to prove that contracted the disease from the contaminated water The must rule out the other 8 possible causes: That it came from a person, fruits, house fly, shell fish, and other means that are still unknown.

Issue: did the produce sufficient evidence from which inference might reasonably be drawn that the s negligence was the proximate cause of his illness? Or must the eliminate all other possible causes? Ruling: The s evidence is sufficient For to prove all other causes would be unreasonable Contact with a person To contract it from a person, it comes about 2 weeks later would have to prove that everyone he came in contact with 2 weeks before he got it did not have it he rode in a cab every day to work unknown causes: would be required to eliminate sources not yet defined If the has established facts from which it can be said with reasonable certainty that the direct cause of the injury was the one for which the was liable, it is sufficient s proof of causation: drank water daily lived in the area contaminated large # of cases evidence of habits, home surroundings medical testimony Notes: Class: Based just on these numbers, it sounds like the is the cause of the problem However, do not want the jury to speculate Even if the tried to argue that the could have been in the class of the other 50/180 that got typhoid fever every year regardless of contaminated water, the could argue that it was more likely than not that he was not in that smaller class The causation is less than 51%, but is it still a substantial factor? Do not want under deterrence There is a trend to punish s in order to deter further actions Sometimes the court will change the burden of proof to make the disprove the causation Further questions to ask: Was the pattern of typhoid always prevalent in this neighborhood? Was there a lot of flies that summer Did the eat a lot of fruit Etc. What is still unknown in this case is what the jury is supposed to do with this information Probabilistic Recovery for harm in the future: Generally, courts used tell the to sue when the second disease develops Reasons: Will deplete s funds for others will spend the $ before he gets the disease may never get the disease Now, it has been more accepted to allow the to recover if it is more likely than not it will develop Reasons: Difficulty of proof 20 years from now when it develops Many more intervening events Food poisoning: Generally courts do not allow recovery Zuchowicz (/Decedents Estate) v. United States (/Federal Gov): Causation in Fact-Basic Doctrine BLL: Where a negligent act increases the chances that a particular type of accident would occur, and such accident does in fact occur, a court may conclude that the negligent conduct was the cause of the injury Facts: s story: s admit that its doctors in the naval hospital was negligent in prescribing twice the allowed amount of Danocrine She took it for one month, then stopped

About 4 months later, she was diagnosed with PPH While waiting for a lung transplant, she became pregnant This made her ineligible for a transplant and increased difficulties due to PPH One month after giving birth, she died PPH (Primary Pulmonary Hypertension): It is very rare Only 197 cases from 1980s to 1992 Exogenous agents that may cause it Birth control pills Appetite suppressants Chemotherapy drugs Rapeseed oil L-T It is safe and effective when properly used There has been no formal studied of excess doses since no one receives it that high Issue: Ruling: Law: Action under the FTCA (using Connecticut law) Expert Testimony: The rarity of PPH combined with the fact that hardly anyone had ever received such a high does made it difficult for the to prove that there was causation linking PPH to Danocrine overdoses Therefore, based his case on expert testimony Dr. Matthay: Prof of medicine at Yale Nationally recognized expert in pulmonary medicine Examined the He was confident to a reasonable medical certainty that the Danocrine caused it Based on the temporal relationship of the overdose and the start of the disease, and Prior to the overdose, she was active and healthy Shortly after, she developed symptoms of PPH Weight gain, swelling, shortness of breath The differential etiology method of excluding other possible causes He excluded all causes of SPH He ruled out all previously known drug causes He described similarities between s case and accepted cases of drug-induced PPH Dr. Tackett: Prof of pharmacy Published in the field of effects of drugs on vascular tissues He believed that Danocrine, more likely than not, caused PPH by producing Decrease in estrogen Hyperinsulinemia Increases in free testosterone and progesterone He relied on various published and unpublished studies Was admission of the s Expert Testimony Manifestly Erroneous? McCullock: The decision to admit expert testimony is left to the broad discretion of the trial judge and will be overturned only when manifestly erroneous Federal Rules of Evidence: Permits opinion when the witness is qualified as an expert by knowledge, skill, experience, training, or education, and if the testimony will assist the trier of fact to understand the evidence Daubert v. Merrell Dow: Trial judge must ensure that the expert testimony rests on a reliable foundation and it is relevant to the task at hand This and the FRE leave in place the gatekeeper role of the trial judge screening the evidence Relevant Factors:

this was not an exhaustive list Was the trial courts admittance of the testimony an abuse of discretion? NO The trial court fulfilled Daubert requirement Preliminary assessment of whether the reasoning or methodology underlying the testimony is scientifically valid and of whether that reasoning or methodology properly can be applied to the facts in issue Were the district courts factual findings with respect to Causation clearly erroneous? FRCP 52(a): must be clear error s contend that it was not more likely than not that Danocrine caused PPH since it has never previously been linked to it Connecticut Law of Causation: However, under Conn law: In addition to proving fault, must establish a causal relationship between the physicians negligent actions or failure to act and the resulting injury by showing that the action or omission constituted a substantial factor in producing that injury. To meet the substantial factor requirement, must generally show that 1) s negligent act or omission was a but for cause of the injury 2) that the negligence was causally linked to the harm, and 3) s negligent act or omission was proximate to the resulting injury 1) s negligent act or omission was a but for cause of the injury must determine whether s act in giving the drug was the source of s illness and death, and it is not clearly erroneous that it was more likely than not that the drug was a but for cause, relying on the expert testimony that it was not just the drug, but the overdose that led to the illness and death it is not clearly erroneous that it was more likely than not that the drug was a but for cause, relying on the expert testimony must determine that the s negligence (the overdose), by a preponderance of the evidence, was responsible for the injury the mere fact that she was exposed to the drug is not enough Ex: car accident. Was the speeding the cause of the s injuries? Before, courts were reluctant to say in these circumstances that the wrong could be the cause Cardozo and Traynor: If the negligent act was deemed wrongful because the act increased the chances that a particular type of accident would occur, and A mishap of that very sort did happen Then - It was sufficient to support a finding that the negligent act caused the harm - Where such a strong causal link exists, it is up to the negligent party to bring in evidence denying the but for cause and suggesting that the wrongful conduct was not a substantial factor Since the FDA does not approve of the dosage above the level, the drug was wrongly prescribed in an unapproved and excessive dosage (strong causal link has been shown), and the has generally shown enough to permit the finder of fact to conclude it was a substantial factor Notes: The court is taking the Kumho approach, which looks to Daubert for the test, but uses flexibly Do not have to base all expert testimony on studies The preponderance of the evidence standard can also prevent evidence from coming in Establishing causation through reliance on expert testimony : Usually in mass tort cases, the s rely on expert testimony to prove the causation Frye test: Must be based on techniques that are reliable in the scientific community Daubert: Based on the Federal Rules of Evidence 702

If scientific, technical, or other specialized knowledge will assist the trier of fact to understand the evidence or to determine a fact in issue, a witness qualified as an expert by knowledge, skill, experience, training, or education may testify put admissibility on the map s could talk about it courts started taking the gatekeeping role seriously, and would throw cases out After using this test, juries must determine if it is credible Kumko: Court reiterated that the gatekeeping role was to be taken seriously, but the Daubert test should be flexible Also states that the court should apply the Daubert factors to non-scientific witnesses Applies to technical knowledge as well as scientific knowledge Nothing in Daubert or the Federalk Rules of evidence requires a district court to admit opinion evidence that is the ipse dixit of the expert. ipse dixit: something asserted but not proven Inferences: Ex: fell down stairs that were accepted as dangerous. The court was allowed to make a reasonable inference that the dangerous stairs played a substantial role in the fall, even though there was no proof that even fell (she did not jump, etc.) Alberts (/Patient) v. Schultz (/Physician): Causation in Fact-Loss of chance theory BLL: To recover for lost chance medical malpractice, a must present evidence that the harm for which he originally sought treatment was in fact made worse by the lost chance Facts: July 14th: went to 1 doctor with complaint of rest pain, which results from inactivity 1 did not order an arteriogram or other tests July 28th: his referral went through to go to 2 2 sent him to the hospital immediately a bypass surgery was attempted after no improvement, the leg was amputated on Aug 1st Issue: Does NM recognize a cause of action for the increased risk of harm to a patient as a result of a physicians negligence, and if so, do the facts of this case satisfy the requirements for proximate cause? Ruling: YES/NO s claims: 1 did not warn him of his condition or make a timely referral to a specialist 2 failed to perform the appropriate tests and procedures s expert testimony: Dr. Hutton- vascular surgeon He testified that In his opinion, 1 should have performed motor and sensory exams Should not have allowed nearly 2 weeks to pass Even the passage of 6 hours can make a difference His leg could have been saved if there were arteries suitable for bypass However, cannot tell from medical history if there were any He could not pinpoint at what time the leg condition became irreversible The Lost Chance/Loss of Chance Theory: General: Generally begins with the patients presenting problem, and going to the doctor A claim is usually brought when there is negligent denial by the healthcare provider to provide the most effective therapy for the patients condition Incorrect diagnoses, inappropriate treatments, failure to give timely treatment Probability Every patient has a probability that he will get better, even if it is small The doctors have reduced these odds The patient does not claim that the malpractice caused his entire injury, just that the physicians negligence reduced the chance of avoiding the injury sustained

Elements: The standard elements of duty, breach, loss or damage, and causation are required However, loss of chance actions differ from standard medical malpractice actions in the nature of harm for which relief is sought bears the burden of proof, which is done almost always through expert testimony Loss or Damage: The injury is the lost chance of a better result, not the harm caused by the presenting problem It is not the physical harm itself Rather, the causal connection between the negligence and the resultant injury must be medically probable The chance of a better result may be looked at as a window of time that existed before the malpractice took place Through the negligence, the window was closed The claim is not for the subsequent injury, but for the fact that it is now too late It is not speculative Based on actual physical harm Not similar to exposure cases Does not deal with potential future injuries must present evidence that the problem he originally sought treatment for was made worse Courts must distinguish between the original injury and the loss-chance injury Courts can limit loss-chance recovery to cases where the chance of recovery lost was sizeable enough to be material Cause: cannot bring an ordinary malpractice claim because his preexisting condition (peripheral vascular disease) precludes proof to a medically certain degree that the Dr.s negligence caused the loss of his leg However, the standard for proving cause does not change There must be a causal link between the negligence and the lost chance Proof to a reasonable degree of medical probability More likely than not caused by the negligence Preponderance of the evidence = reasonable degree of medical probability Calculation of Damages: Damages should be awarded on a proportional basis Most courts determine the value of a chance of a better result as a % of the entire life or limb The % of chance lost X the total value of the persons life or limb Loss of Chance as applied to this case: has not established causation his claim is based on loss of time he cannot demonstrate that there was a window of time during which the measure could have been taken to foreclose the need to amputate cannot show that a bypass would have saved the leg cannot show that he was a suitable candidate for the bypass, and then became unsuitable when it was actually performed the medical evidence is based on incomplete records and assumptions the expert stated that if the had better veins, he would probably find that he would have been suitable for bypass Notes: Loss of chance cases are brought that way because they are normally based on chances that are less than 51% Over 51% are usually brought as malpractice Loss of chance redefines injury so that you are not suing for the outcome of the raffle ticket Allow s to sue for a different type of injury Usually juries do not reduce the damages proportionately Jury would have to know what the right # is for 100% chance There is a misleading sense of precision Usually when the proves 51% chance, gets full damages Note 1: Usually have to show that more likely than not the doctor was negligent

Now, just have to show that he was negligent at all Sometimes the doctor can reduce the chance, but not eliminate it Ex: reduce the chance from 30% to 10% Some courts will give proportionate damages Other feel that it is a lost lottery ticket Should not be allowed recovery based on a statistical chance The acts of the did not actually cause the injury Concerns: The conduct of trials when admitting statistical data Can be unreliable and misleading Fairness The impact on the cost of medical services Joint and Several Liability General: If more than one person is a proximate cause of s harm, and the harm is indivisible, each is liable for the entire harm. Ex: 1 scratches s arm, and goes to the hospital, where, because of 2s negligence, loses her arm can fully recover from either, but cannot collect twice does not matter which is insolvent, because the solvent will have to pay the entire amount Indivisible v. Divisible Harms: Indivisible: not capable of being apportioned between s Each is directly attributable for the harm Joint Liability: All s are together responsible for the s harm If one is insolvent, the can collect from the solvent s bears the risk of an insolvent Several Liability: Liability is allocated bears the risk of an insolvent Questions may ask: Which caused which aspect of the s injury Who was more or less at fault The jury can allocate 1 as 10% at fault, and 2 90% at fault This can be impractical because sometimes the reason why the is suing is because some of the s are insolvent Joint and Several Liability: could collect everything from 1 if 2 is insolvent, even if he is only 10% at fault. This theory has been significantly qualified by legislative action Some courts will set a certain threshold for being liable (% at fault) Sole proximate cause A fiction that permits a court in the presence of overwhelming factually liability to one with overwhelming practical liability to the other Because so much of fault was 2, will assume that the 2 is the sole cause of injury This is not used very frequently Rules on Apportionment: Action in concert: There is no apportionment Each , if they act in concert, is liable for the others damages Ex: 1 and 2 drag race. 1 hits . Each is liable Successive Injuries: Courts usually apportion the harm if the harms occurred in successive incidents, separated by substantial periods of time Ex:1 owns property and polluted s property for 10 years. 2 buys the property and pollutes the next 10 years. Each is responsible for 10 years Overlapping: Where 1 is liable for both his act and 2s, but 2 is liable only for his own Likely where 2s negligence is in response to 1s

1 is liable for everything, and 2 is liable only for the separate injury or aggravation Ex:1 breaks s arm. goes to the doctor, whos negligence leads to the amputation of the arm. 1 is responsible for everything, and the doctor is liable only for the difference between a broken arm and an amputated arm Indivisible Harms: Death or a single injury: The s death or any single injury is not divisible Fires: The burning of s property is not divisible Different amounts: If acting in concert or concurrently: Each is liable for 100% Jury determination of culpability only provides for how much contribution each can seek from each other If acting successively: Can be divided proportionately Ex: 80%, 20% One satisfaction Only: Even if 1 and 2 are jointly and severally liable, is entitled only to a single satisfaction of her claim Therefore, if recovers the full amount from 1, she cannot recover anything from 2. Contribution: Def: If 2 s are J&S liable, and one pays more than his pro rata share, he may usually obtain partial reimbursement from the other . Amount: Generally, each has to pay an equal share Comparative negligence: In Comparative negligence states, the duty of contribution is proportional to the fault Limits: Most states limit contribution as follows: No intentional Tort: Usually an intentional tortfeasor may not get contribution from the other (even if they also acted intentionally) Contribution must have liability: The contribution (the one being sued for contribution) must in fact be liable to the original Ex: Husband and wife are hit by . H and were both negligent. W recovers fully from . H has immunity from suit from W. cannot recover contribution from H since there was no underlying liability to the original , W. Settlements: Settlement by Contribution : If settles, he may generally obtain contribution from the other s But he must prove that the other s would have been liable to Settlement by Contribution : Where 1 settles, and 2 (against whom later gets a judgment) sues 1 for contribution, courts are split among: Traditional Rule: Majority 1, the settling , is liable for contribution Reduction of s claim: Reject contribution, but reduce s claim against 2 pro-rata No contribution Rule: Discharge 1 Gives s strong incentive to settle Indemnity: Def: Sometimes courts will not order two J&S liability s to split the costs (contribution), but will instead sompletely shift the responsibility from one to the other. 100% shifting of liability Sample situations where indemnity is applied: Vicarious Liability: If 1 is only vicariously liable for 2s conduct, 2 will be required to indemnify 1.

Ex: If an employee injures , and sues the employee and recovers under Respondeat superior. The Employer has the right to indemnity from the employee (the employee must pay the employer back) Retailer v. Mfr: A retailer who is held strictly liable for selling a defective, injury-causing product will get indemnity from others further up the distribution chain (mfrs) Active/Passive: The party that was negligent in a passive manner can obtain indemnity from the active party Changes in the doctrine: ~12 states have abolished the doctrine, making a responsible only for his share some states have abolished the doctrine when the is less than a certain z% at fault (usually 50%) some use it only for economic damages some states have abolished the doctrine when the is partially at fault some have retained it only for certain areas of law Combination with intentional negligence: When the s negligence is combined with intentional tort or crime If the same is negligent and has performed an intentional act, he will be liable for both If one acted negligently, and the other acted with intent and is unknown, the court will not likely apportion it Ex: apartment owner is negligent in the safety of his building, which causes a rape. The unknown rapist will not likely be apportioned any liability, and the owner will take all Absent tortfeasors: Whether the jury will assign some liability to the absent tortfeasor depends on the state statute Immunity: Where one is immune from suit, the jury can assign liability to him because immunity is an affirmative defense. Non-delegable duties: Ex: City cannot delegate a duty to the city light supplier if a street light goes out. One who carries out a non-delagable duty is an agent Although the city would be entitled to indemnity from the light supplier Rest: 5 approaches: 1) joint and several liability one is liable for everything that may seek contribution from others 2) several liability must recover from each 3) joint and several liability with allocation if one cannot pay for their contribution, it is divided among all the solvent s according to their comparative responsibility 4) hybrid if above the threshold of responsibility, then J&S liable. If below, then only severally. 5) CA approach if economic damages, then J&S liable. If non-economic, then only severally. Summers (/Shooting Victim) v. Tice (/Hunters): Joint and Several Liability-Two possible causes for one harm BLL: When two or more persons by their acts are possibly the sole cause of a harm, and the plaintiff has introduced evidence that one of the two persons is culpable (responsible), then the has the burden of proving that the other person was the sole cause of the harm Facts: Two hunters were aiming for a bird, and shot in the direction of One shot hit his eye, and the other his lip Both s were using the same gun and bullets Issue: Are the s jointly and severally liable when they were not acting in concert, and there is no evidence to show that they were both liable? Ruling: YES Negligence of both was the cause As a direct and proximate result of the shots fired by the s, a pellet landed in his eye and lip This is based on the assumption that the court was not able to determine if only one or both was liable But the shot to the eye was from one only

Previous Case Law: Oliver v. Miles: Same situation, and both hunters were found liable and were acting in concert To hold otherwise would exonerate both from liability, although each was negligent, and the injury resulted from that negligence Applies to criminal law and drivers in a drag race injuring a 3 rd party should not be required to pin fault on only one : is in a much better position to offer evidence and determine which one caused the injury Ex: Ybarra (unconscious patient can sue all the doctors) It is up to the s to explain the cause of the injury Each is liable for the whole injury whether they were acting in concert ort independently . Notes: Is the doctrine limited to virtually simultaneous situations? Ex: where was injured in the same are of her body in 3 successive auto accidents Some courts find that if the jury cannot proportionate according to fault, it must divide the damages equally Other courts reject this theory and give all damages to 1 Ex: where the was injured by taking two drugs from the same company at one The court used the but for test, and found that since the act of taking either one alone would not result in the harm, there is no causation Hymowitz (/Daughter of DES user) v. Eli Lilly Co. (/DES Mfr): Causation in Fact-Market Share Liability BLL: Liability among manufacturers of a drug that causes injury shall be apportioned per market share Facts: was injured when her mom ingested DES during her pregnancy DES: Was invented in 1937 in England Was never patented FDA finally approved of it for pregnancy use in 1951 Then the FDA banned the use as a miscarriage preventative in 1971 because it showed latent effects of cancer in the babies born while mothers were using the drug Why it is difficult to bring a DES case: Hard to identify the manufacturer of the pill taken by certain people since There were about 300 companies in and out of the business for many years All DES pills were of a similar composition Due to the latent affects: Many peoples memories fade Witnesses die SOL may bar the case before the injury is found Courts have imposed a rule that the SoL begins to run upon the latent effects of exposure to any substance Statutes of Repose: Start from the date the product is sold Issue: whether a DES may recover against a DES Mfr when identification of the maker of the drug that caused the injury is impossible Ruling: YES Law: Since this is not a class action, but there are over 500 pending suits in the state, the rules must accommodate mass tort litigation In general product liability cases, identification of the is mandatory Other possible tort theories used for recovery when the is not identifiable Alternative Liability: However, in Summers v. Tice, the court looked to alternative liability: The rational for shifting the burden of proof to the s is that without that, the s would be silent, and the would not recover. With the shift, the would be forced to speak, or else be found jointly and severally liable

Where the of s who possibly harmed the is small, there is a good chance that one of them named is guilty and they will be able to exonerate themselves However, in DES cases, where there are a large number of s, it is equally as hard for the s to prove that one person is guilty, nor is it likely that all Mfrs will be before the court Likely that non of those named even caused the injury Alternative liability, therefore, is not the answer for the s Concerted Action theory:

Ex: drag racing cases Provides for joint and several liability on the part of all s having an understanding, express or tacit,

to participate in "a common plan or design to commit a tortious act"

Different for drug companies:

Drug companies were engaged in extensive parallel conduct in developing and marketing DES There is nothing to show any agreement, tacit or otherwise, to market DES for pregnancy use
without taking proper steps to ensure the drug's safety Concerted Action, therefore, is not the answer for the s

In short extant common-law doctrines, unmodified, provide no relief for the DES plaintiff unable to identify
the manufacturer of the drug that injured her. Court feels that it needs to modify the rules of personal injury liability, in order "to achieve the ends of justice in a more modern context"

Obviously the Legislature wants to protect the innocent victims because it has revived hundreds of DES cases. The manufactures that produced the drug for use for pregnancy should have to pay for it, even it they are not identified. How to apportion the damage among the unknown manufacturers : The court looked to ways other courts have addressed the issue, and decided to adopt the market share
approach: Sindell:

Justification was that limiting a s liability to the proportionate share of market it held will be roughly Brown: If the s are joined severally only (where not all the Mfrs are joined), and the liability is limited to the
market share, then the will recover less than 100%
The national market provides the best answer Do not want to restrict it to state markets This would lead to errors the could have bough the drug in another state

the same as the amount of harm that company could have caused

Court is aware that this might cause disproportion between liability of the manufacturers
and the injuries caused in that particular state,

But, the market share theory can be based on the amount of risk each created to the public-at-large (overall culpability of )

Exculpation:

A will be able to escape liability by proving that it was not a member of the DES manufacturers that
distributed the drug for pregnancy use There will be no exculpation allowed for a who, although a member of that class, appears n o to
have caused the injury

Several only: Liability of DES producers is several only, therefore the liability should not be inflated when all
market participants are not before the court

Dissent: Would adopt the market share liability, while shifting the burden of proof on the issue of causation to the s,
and would not impose liability on the s who prove they could not have caused the injury

The should not be able to recover from a that could not have caused the injury Notes: Court is saying that overall risk (% market share), as compared to specific risk, is sufficient for liability According to this courts reasoning, if someone was selling the drugs at a time after the time span, it would still be liable

This is the rule in NY, but you get problems when you go to different states: If a company only sells in CA, but it is still responsible for 25% of the market. Someone would have a good case, whether they lived in CA or not The problem is that there is no clear ordination for the rules in different state, and the NY rule has an extraterritorial effect. Should the cost of the product reflect the risk to the possible injures? Would the mother have to contribute in this case? The Mfr will always try to pass the cost on to customers But they cannot do it after the harm occurs people before the harm should not have to be liable for the cost before the harm is discovered Market Share Liability: The apportionment of liability between each participant in an industry equal to the participants market share. What happened to it? It is the high level way of eliminating the burden on the Has been used in asbestos, lead paint cases, etc., and has generally not been successful Most important is the concept of fungibility The ability to differentiate between manufacturers Many cases do not involve the identity problem as in DES where there was no patent Hard to imagine that it will ever happen again However, it is still available as a theory Has come up in handgun litigation in NY, and there are other fungible products out there that are susceptible to this (if similar to DES) Used when there is no other alternative Acting in concert: This court rejected this theory, because it was not a small number of s acting negligently towards a single Usually it is a that is participating in the negligent action, or encouraging it in some way Enterprise liability: Industry-wide liability Ex: 6 companies who comprise of almost the entire industry in the country Would be harder to apply if there were a large number of Mfrs Attempts to apply the market share liability doctrine to other products : Asbestos Fungibility is required: all products must be made according to the same formula Lead Paint Usually hard to bound Childhood vaccines Sometimes based on public policy Blood clotting factors: Usually the donor source is not constant, so hard to assign market share liability Paint ship products: So many different products Toxic Harms Reasons why it is different from everyday tort law : Problems of identification: Instead of trying to determine who did the harm, must determine whether there is identificable harm that can be isolated from the risks of everyday living Lag-time issues with diseases Problems of Boundaries: Toxic harm can affect people in the future, in the fetus, through genetic development, etc. It is hard to predict the amount of exposure Problems of Source: Frequently environmental harm is the result of aggregate risks created by a considerable number of independent causes Cannot use the traditional tests (but for, J & S, substantial factor) Present and future claimants: Exposure only injuries:

Medical monitoring, fear, risk of future injury Reasons to allow medical monitoring: Public health interest Deter other toxic exposure Mitigate future disease Fairness Bower Criteria for allowing medical monitoring: Page 397

Proximate Cause
General: Def: The natural sequence of events without which an injury would not have been sustained Even when has shown that was the cause in fact of s injuries, must still show that was the proximate cause of those injuries. The proximate cause requirement is a policy determination that a , even one who has behaved negligently, should not be automatically liable for all the consequences, no matter how improbably or far-reaching, of his act. Usually will not be liable for consequences that are very unforeseeable. Ex: If and X collide, and it is s fault, and X has dynamite in his car. The explosion causes to drop her baby when she was walking 10 miles away. will not be able to recover against because it was so unforeseeable that the injury would occur due to s negligence. Courts will generally hold that s careless driving was not the proximate cause of s injuries. Usually the argues that something unexpected contributed either to the occurrence of the harm or the severity An occurrence can have more than one proximate cause Foreseeability: is sometimes only liable for the consequences of his negligence that are reasonable foreseeable at the time he acted. Ex: Wagon Mound Unforeseeable : If the is not foreseeable, usually cannot recover Ex: Palsgraf Exceptions to the Foreseeability Rule: Extensive Consequences from physical injuries: When has suffered a foreseeable injury, is liable for all additional unforeseen injured Ex: Egg-shell skull General Class of harm but not same manner: As long as the harm suffered by is of the same general sort that made s conduct negligent, it is irrelevant that the harm occurred in an unusual manner. part of a foreseeable class: does not matter if the injury to was foreseeable if was a member of a class to which there was a foreseeable risk of harm Ex: riverbank property owners have a flood due to a boat smashing into the bridge and damning up the river The extraordinary in hindsight rule: Rest 2nd The s conduct will not be the proximate cause of s harm if, after looking back form the harm, it appears highly extraordinary that the s conduct would have brought about that harm Intervening Causes: An intervening cause is a force which takes effect after s negligence, which contributes to that negligence in producing s injury Ex: when a s car truck flies open when he is driving, and he pulls over and is then hit by a car. The Car company is not liable because the intervening act was not a foreseeable risk associated with the original negligence. Superceding cause: Those that are sufficient to prevent from being liable Unexpected Harm Although the harm that occurred was one that would be expected, the manner of its occurrence sometimes justifies exculpation of the Benn (/Injured Customer) v. Thomas(/Store): Proximate Cause- Eggshell

BLL: A tortfeasor who act, superimposed upon a prior latent condition, results in an injury may be liable in damages for the full disability. Facts: 1989: rear-ended the , and caused a bruised chest and a fractured ankle the died of a heart attack 6 days after the accident Expert opinion of s health: He had a long history of coronary disease he had a heart attack in 1985 and was at risk of having another feels that the accident was the straw that broke the camels back Other medical evidence indicated that the accident did not cause his death Trial court charged the jury with proximate cause liablity Issue: Should the jury be charged with the eggshell rule when the s negligent actions were not the main cause of the s death? Ruling: YES Eggshell Rule: the doctrine that the is liable in tort for the aggravation of a s existing injury or condition, regardless of whether the magnitude of the injury was foreseeable. It requires the to take the as he finds him, even if that means that the must compensate the for harm an ordinary person would not have suffered. The rule deems the injury, and not the dormant condition, the proximate cause of the s harm Duleiu: If a man negligently runs over the , it is not answer to the that he would have suffered less injury, or no injury at all, if he had not had an unusually thin skull or weak heart s contentions (and courts responses): 1) that s proposed instruction was inappropriate because it concerned damages, not proximate cause Christianson: Consequences which follow in unbroken sequence, without an intervening efficient cause, from the original negligent act, are natural and proximate; and for such consequences the wrongdoer is responsible, even though he could not have foreseen the particular results that follow. 2) that s instructions do not sufficiently convey the law the eggshell rule rejects the limit of foreseeability that courts usually require in determining proximate cause Rest. 461: the rule imposes full extent of liability to the injuries, not to those merely foreseeable the s instructions would be appropriate only to determine the initial damages, but not the eggshell liability (cause of death) the introduces substantial medical evidence that the accident was the cause of his death even though there is conflicting evidence, the jury should decide Notes: Possible arguments for the It was not foreseeable that the would die of a heart attack The accident was not the proximate cause of the death You should trace it as far as those injuries are foreseeable You cant deter someone past the point of foreseeability The s eggshell susceptibility to harm does not expand the s liability It would not make something negligent that was not originally negligent Can only enlarge the consequences of a negligent act Recovery from further resulting injury: should likely be able to recover from a resulting mental disease that happens as a result of other harm courts have been more likely to allow recovery when the person commits suicide after being injured however, will not hold attorneys liable for suicide of disappointed clients Secondary Injuries: When the person suffers more injury from negligent care at the hospital, the can usually recover everything from the Overseas Tankship (/Wharf Owner) v. Morts Dock & Engineering (/Workmans Employer): Proximate Cause- reasonable foreseeability of damages BLL: Even though injury may result from a negligent act, liability for that injury is limited to the risk reasonably to be foreseen.

Facts: spilled lamp oil 600 feet away from the s wharf. It spread over there though set sail, making no effort to clean it up felt he could continue to weld and fix his ship safely without setting any fires, as long as precautions were taken however, the oil caught fire because a cotton rag ignited from molten metal falling from the s ship. This caused damage to the s ship and wharf Issue: Should a be liable for all damages directly resulting from his negligent act? Ruling: NO Polemis: The workman who dropped the plank and caused the spark (that caused the fire) was liable If the is guilty of negligence he is responsible for all the consequences whether reasonably foreseeable or not The negligent actor is not responsible for consequences which are not direct Directness is enough eggshell skull rule applied to different types of injuries The is using this argument to say that the should be liable even if it wasnt foreseeable Polemis should no longer be good law: It is not consistent with justice or morality if the consequences are not foreseeable This would be too harsh a rule Reasoning why the should only be responsible for foreseeable consequences: It can be judged by the standard of a reasonable man that he ought to have foreseen them Foreseeabilty test corresponds with the common conscience of mankind Direct consequence test leads to no where but insoluble problems of causation Should consider elements of tort (duty, breach of duty, damage) But there is no liability unless the damage has been done The liability is in respect of that damage s liability depends on the reasonable foreseeability of the consequent damage it does not matter if was liable for some other foreseeable damage, must foresee the damage in question Notes: Why the court cannot use the directness rule Limit liability for the The damages must be the direct result of the negligent act Can only be negligent for the consequences that are likely to follow The direct test does not provide a corresponding check The damages could go on forever Foreseeability Situations where the court can link s conduct to the s injury: When it satisfies the but for test. When the s actions are extremely close in time and space to the harm Trolley car/falling tree: Court found that although the accident would not have occurred but for the trolleys speeding, speeding does not increase the probability of trees falling on trolleys. But for/Causal link: Both are ways to determine if the is liable Even if the but for test is satisfied, no liability ensues unless s wrong increases the chances of such harm occurring in general. McGaughlin (/Wharf Owner) v. Morts Dock & Engineering (/Workmans Employer): Proximate Cause BLL: Even though injury may result from a negligent act, liability for that injury is limited to the risk reasonably to be foreseen.
Facts:

An unconscious child was taken into the care of a fireman and nurse after she fell into a lake.

More heat was needed to revive the child, so the firemen got some "heat blocks".

The fireman claims that he went to the training 5 years back, and told the nurse to use a towel in between the blocks and her skin

Nevertheless, the blocks were applied directly to the , while the fireman, Traxler, stood next to her and watched. The 's aunt could recall no warning given by the firemen to the nurse as to the danger On the block was a label with the s name and a warning:

The "M-S-A Redi-Heat Blocks"

"Wrap in insulating medium, such as pouch, towel, blanket or folded cloth."

brought suit against : For loss of services against , the exclusive distributor of the heat blocks

It had failed adequately to warn the public of the danger and to properly "instruct" ultimate users.

s appeal:

contending that the s failed to prove any actionable negligence on the part of the Issue: Was the s act the proximate cause of the s injury? Ruling: NO
Jury could have found that

there was latent danger, and that the material was already protective

blocks said: "ALWAYS READY FOR USE" and "ENTIRELY SELF CONTAINED" was small and on the back was foreseeable that they might not read it they were meant to last a while

that the warning was not adequate

that other users, who had not receive the original warning, would use them

Trial court instructions to the jury:

That the would not be liable if "an actual warning was conveyed to the person or persons applying the blocks

that they should be wrapped in insulation of some kind before being placed against the body" for in that event the "failure to heed that warning would be a new cause which intervened

But the true problem presented is proximate causation, and not one of the general duty to warn or negligence of the distributor.

The court basically instructed that the could still be liable (even though the fireman knew), if it was reasonably

foreseeable that the blocks, absent the containers with warnings, would be used by unwarned third persons. The instruction was erroneous:

In other cases, the manufacturer failed to warn the original vendee of the latent danger, and there were no additional acts of negligence intervening between the failure to warn and the resulting injury or damage. This case did not involve the negligent failure of the vendee to inspect and discover the danger

The intervening negligence of the immediate vendee does not necessarily insulate the manufacturer from liability to third persons, nor supersede the negligence of the manufacturer in failing to warn of the danger. Under the circumstances, we think the court should have charged that if the fireman did so conduct himself, without warning the nurse, his negligence was so gross as to supersede the negligence of the defendant and to insulate it from liability.

This is the rule used when knowledge of the latent danger or defect is actually possessed by the

original vendee, who then deliberately passes on the product to a third person without warning.
Whether or not the distributor furnished ample warning on his product to third persons was not important


Dissent:

It is whether Traxler had actual notice of the danger and he deprived the nurse of her opportunity to read the instructions. The distributor could not be expected to foresee that its demonstrations would be disregarded.

The recovery by plaintiff should not, as it seems to us, be reversed on account of lack of foreseeability or a break in the chain of causation due to any intervening act of negligence on the part of a volunteer fireman. What happened here was that the container, with the instructions on it, was thrown away

This was a risk which the manufacturer ought to have anticipated in the exercise of reasonable care.

It is not necessary to anticipate the negligence of another.

Restatement of Torts ( 449): "If the realizable likelihood that a third person may act in a particular

manner is the hazard or one of the hazards which makes the actor negligent, such an act whether innocent, negligent, intentionally tortious or criminal does not prevent the actor from being liable for harm caused thereby."
447: "The fact that an intervening act of a third person is negligent in itself or is done in a negligent manner does not make it a superseding cause of harm to another which the actor's negligent conduct is a substantial factor in bringing about, if (a) the actor at the time of his negligent conduct should have realized that a third person might so act

Notes: Ex: when a worker died from exposure to dry ice, the company was not found negligent because there were not warning signs that were supposed to be on there. The employer knew of the danger and should have warned the worker Deciding what it is that needs to be foreseeable The exact consequences do not have to be foreseen Can be determined only after the significant facts of the case are determined There is no specified definition of how specific the facts must be

Palsgraff (/Injured women) v. Long Island Railroad (/Railroad): Proximate Cause- no damages if there is no negligence as the proximate cause BLL: When a negligent act results in injury to another, but the risk of harm to the injured party was not a reasonably foreseeable consequence of the negligent act, no duty of care exists as to the injured party, and, thus the negligent party is not held liable for the damages. Facts: bought a ticket to go to the beach, and went and stood up on the platform to wait for the train, standing next to some scales. Another train came into the station, that was not hers When the train pulled up, two men ran to catch the train as it was moving away The first man made it up, and the guard on the train helped the second man up. In the process, the guard dislodged the second mans bundle The bundle fell in between the train and the platform. It was unknown that it was a bundle of fireworks The bundle exploded The scales exploded from the fireworks, and the was injured
Issue: Was the negligent in that it breached a duty owed to the ? If so, was that breach the proximate cause of the injury? Ruling: NO There was no wrong to her: The s act was a wrong in relation to the passenger getting aboard, but not to the Nothing gave notice that the package was dangerous- it was not foreseeable that it would injure the The is suing for a wrong personal to her, and not a breach of a duty to her There was possibly a wrong to the person who was getting on the train, but not to her Ex: when someone bumps someone in a crowd, he is not responsible for the person that is bumped on the outer part of the crowd The wrongdoer is the person who is carrying the explosives, not the person who knocks it down The must show a wrong to herself (a violation of her own right) and not merely a wrong to someone else Cannot merely be conduct that is wrongful, but not a wrong to anyone The risk reasonably to be perceived defines the duty to be obeyed The needs to have reasonable notice (clear to an ordinary person) Here, it was clear that no ordinary person would have expected the bundle to be dangerous Negligence is not a tort unless it results in the commission of a wrong The commission of a wrong imports the violation of a right If the act was unintentional: must show that the act had the possibilities of danger so apparent so as to make the owe a duty Dissent: negligently knocked the package from the passengers arm the results depends on the theory of proximate cause, not negligence the relation between cause and effect the act is wrongful. It is a wrong to those who are within the radius of danger and all those who were there There is a relationship between the and those whom he injures Ex: When H is compensated for the loss of his Ws services There was no duty to the H Cannot just consider the negligence theory-must consider proximate cause Polemis:

The dropping of a plank was negligent because it might injure people or the ship Therefore, the owner of the ship recovered for damage Rule: Everyone owes to the world at large a duty of refraining from those acts that may unreasonably threaten the safety of others. If such an act occurs, not only has the wronged those whom he would reasonably expect to have wronged, but also those who are in fact injured, even if outside the radius of danger. This does not mean the will be suing by derivation or succession The s complaint is original if it is a result of the s negligence It does not matter if the result was unforeseeable One limitation: The damages must be so connected with the negligence that the latter may be said to be the proximate cause of the former What is the proximate cause: All causes will have an influence, not just one thing A cause that is proximate is essential But it is not the sole cause Cannot trace the effect of an act to the end, but you can trace it part way Proximate: it is indicative of public policy Ex: Fire started by train sparks burns first house down, which burns another house down. First house may recover from the train, the second one cannot. Ex: explosion from a car crash causes a nurse 10 blocks away to drop her baby. The person who caused the car crash should be liable for all that are injured, because they would not have been injured without the s acts The proximate cause cannot be determined from any one consideration It must be something without which the harm would not have happened Was there a direct connection between them without too many intervening causes? Could the result be foreseen? Was there a big gap in time and space? Notes: Class: Were the train operators negligent in the loading of the passenger Was the accident the sole fault/proximate cause of the ? From the facts described in the opinion, it sounds like the explosion blew the scales, knocked them over, and hurt the very unlikely and very unforeseeable likely that she was pushed up against the scales because of the crowd bruising train station should have measures to help prevent injuries in these type of emergency situations nurse ex: no recovery for : interest of is different than the interest of the negligent act property damage v. personal injury to extraordinary a consequence recovery for : Two possibilities of negligence in the air: Dropped package Helping people on a moving train Was it negligent also on the part of the person getting on the train? (contributory) The court did not consider the common carrier standard Would probably not change Cardozos opinion Mrs. Palgrafs interest were not relevant in the negligence This would differ from the dissent view, which focuses more on proximate cause Dissent: insists of logicy of duty whether there was a duty to the there was no negligence directed toward the interest of the ? Focuses on directness rather than foreseeabilty Gives a good explanation of proximate cause There is no defined standard If you cant identify, can go no further If there is a possibility, it is a question for the jury It draws a clear link between duty and causation What interest is protected by this legal duty? Cardozo: some harm to the passenger getting on the train

Question of expediency What is the law capable of The fact that you are able to trace cause and effect goes a long way Foreseeability is relevant, but not determinative Different than Wagon Mound, where foreseeability is everything-better way to look at the relationship between duty and proximate cause Duty when there is a foreseeable risk of harm Proximate cause when the risk materializes Would it have mattered if the was closer to the explosion? Probably not She was on the platform (s property) It would still be unforeseeable, and therefore not likely a duty Recurring fact patterns of causation questions: Rescue: Ex: was hurt while trying to rescue his cousin who fell due to the train conductors negligence Court found that Danger invites rescue, which is a normal reaction The wrong to the person being rescued is also a wrong to the rescuer It is enough that the original negligent act is the cause of both It may matter if the rescuer was hurt by some outside force. Ex: was not entitled recovery when he donated his kidney to his father, who needed a new kidney because of the doctors negligence. Because the rescuers actions were deliberate and reflective, and not made under the pressures of an emergency situation, he was denied recovery Time: Cases that involve long time periods between the acct and the harm are not usually barred by the SOL Tort limitations usually start to run at the time of the injury Even longer for minors. Ex: alleged that as a result of s negligent driving, he struck a 3-year old boy, who, 7 years later, shot the . Court denied recovery for against the because there was no risk of danger reasonably to be perceived, therefore there was no breach of duty, or negligence. Distance: Usually does not have an effect Ex: car accident caused one car to hit the traffic signal Another car caused an accident at a stoplight 2 miles away The court imposed liability Fire: Fire rule unique to NY Denies recovery to the 2nd building to catch fire The 2nd is not a natural or expected result of the first fire That result depends on many other things too (wind, heat, materials, etc.) Kinsman Cases: Kinsman I: River was full of floating ice, and 2 jams were floating down river Because of Kinsmans failure to respond adequately, one if its boats broke loose and hit another boat Due to the citys negligence, the bridge was not raised, and both crashed into the bridge A dam formed, which damages much property Courts findings: Kinsman: It was foreseeable City: It was foreseeable The dock upstream who failed to remove the lock Takes the view of Polemis (not Wagon Mound) Just because there is a small risk of great harm (rather than a large risk of small harm) does not mean that the should be exonerated from liability Some courts find that the is not liable unless his negligence contributed to the risk. Kinsman II: Court rejected claims based on the higher costs of unloading ships due to the flooding The connection between the s negligence and the claimants damages was too remote Andrews: It is a question of expediency of fair judgment Distinguished in People Express:

Was found that a s negligence depended on the ability to foresee a particular or identifiable class of s within the risk created by his negligence

Defenses
The Plaintiffs Fault
Contributory Negligence Common Law Rule: If s negligence has contributed proximately to his injuries, he is totally barred from recovery, no matter how slight his negligence was Still exists only in 4 states Was it always so strict? No, normally juries would just reduce s damages Courts would normally not overturn this either Todays Rule: Standard of care: The is held to the same standard of care as a Reasonable person under like circumstances Usually the court will make it difficult to establish that the was contributorily negligent Statutes: Some statutes are interpreted as barring the defense of contributory negligence Ex: statute requiring school bus drivers to flash lights, allow children to cross street, etc. Court decided that the statute was enforced to protect the school children against their own negligence However, very few statutes have the same effect Proximate Cause: Contributory negligence doctrine only applies where negligence contributes proximately to his injuries Use the same test for proximate causation that is used for the But for cause The conduct (s negligence) must be an actual cause for the s harm Many states have placed that burden on the Claims against which defense is not usable: May be used only as a defense to a claim that is based on negligence Intentional Torts: where s claim is for an intentional tort Willful and Wanton: If s conduct is willful and wanton or reckless Basically, the defense does not apply where disregards a conscious risk But if s negligence is merely gross, usually will be allowed Negligence per se: When is a member of a class for which the statute was enacted to protect But, if otherwise, usually the defense can be asserted. Last Clear Chance: The behaved carelessly and got into a dangerous situation that led to injury claims that had, but failed to realize, the last clear chance to avoid harm to the . had the chance to prevent it, but the did not Two types of situations: had gotten into a situation of helpless peril who was oblivious to the danger but who could have avoided the danger if it became aware at the last minute have actual knowledge of the s danger in time to avoid harm by the exercise of due care courts normally require that the have actual knowledge of the s danger in time to exercise due care typical case first event: s negligence second event: s negligence the s negligence makes the s insignificant difficulties without this doctrine: if you had to compare the s and s negligence, would have to explore every aspect of tort law Recklessness: Usually all courts hold that contributory negligence was a defense for negligent cases only s could not claim it if they acted willfully or recklessly Refusal to Impute Contributory Negligence: When the court imputes the negligence of one person on to another by calling one action derivative (rather than independent) from another, thus, giving the second action the rights only the first person had

Ex: Respondeat superior, wrongful death, loss of consortium, driver and passengers, parent-child Usually differs for s and s Imputing negligence to defeat actions leaves the uncompensated Changes: If a child is hurt by the negligence of his mother and a stranger, the court does not normally impute the mothers negligence to bar the childs action Virtually all imputed contributory negligence has been eliminated Role of the Jury: Relieves the harshness of the all-or-nothing contributory negligence rule Most juries rejected the judges instructions to find for the if they found any contributory negligence by Jury would normally just reduce the recovery proportionately Normally just applied the standard of comparative negligence Comparative Negligence General Def: Rejects the contributory fault all-or-nothing approach Finds that the and are responsible for their proportionately relative degrees of fault is not barred from recovery for contributing to the negligence, but her damages are reduced proportionately to her fault this has been accepted by 46 states Texas compares fault to cause of the Was the s high of fault very significant for causation? If there was intentional conduct by the , should the jury ignore the s negligence? Types of systems: Pure: who is 90% to blame will only recover for 10% of the damages this was the system of common law comparative negligence system Modified: Retains a portion of the contributory fault doctrine 1) can recover under the pure system only if her negligence is not as great (less than) as the s would normally look at the comparison between the and all the s together (because it is an indivisible injury) this process starts over with each claim (maybe if the s also have an injury, or if has several claims) would be bad if the jury returned a 50/50 verdict. They may not have been informed of this rule, and the jury is normally guessing at this %. 2) can recover under the pure system only if her negligence is no greater than the s Arguments against the Modified Version: 1) a party more at fault than the other who has to bear his own losses may also have to bear a share of the other partys losses 2) it becomes chaotic if several s are at fault. Must compare s fault to each s % of fault 3) even when s fault is greater than s, may still fully recover in cases such as last clear chance Most states that have codified these systems through statutes have adopted a modified version. Virtually all states that have developed the system through judicial decisions are based on a pure system Implementation Issues: Some use the UCFA Uniform Comparative Fault Act (most state do not follow it- pure): It is a pure system, but most states have the modified system 1 Effect of Contributory Fault: claimants negligence diminishes his amount of recovery proportionately to s fault, but does not bar recovery Fault: Includes acts or omissions that are negligent or reckless towards a person or property Ex: breach of warranty, unreasonable assumption of risk, unreasonable failure to avoid injury or mitigate damages 2 Apportionment of Damages: court shall make findings (or instruct jury), indicating: the amount of damages each claimant would be entitled to if contributory fault was disregarded only applies if the is a party (for example, a hit and run would not be allocated fault) problems: jury may allocate that missing s fault to the present

the % of total fault of all the parties to each claim in determining the % of fault, the trier of fact will consider conduct of each party whether the conduct was a mere inadvertence, or whether there was an awareness of danger the magnitude of the risk created (people involved) the significance of what the was seeking to obtain with his action the actors superior and inferior capacities the particular circumstances (emergency, etc.) busy intersection, police chase, etc. causal relation between conduct and harm degrees of fault and proximity of causation are intermixed should the jury only compare the negligent conduct that has at least a minimal causal connection to the harm? Once causation has been established, the determination of proximate cause remains a policy consideration If one partys negligence approaches 100% and the other 0%, the court may rule that the one party is the sole proximate cause court shall determine the award of damages to each claimant in accordance with their findings on the basis of J & S liability: upon motion (not more than 1 year after judgment entered) court may allocate the insolvent s damages to the solvent s proportionate to their fault % they may then seek contribution Ex: If was 40% liable, and 1 and 2 are each 30% liable, and the damages are 140K. is at fault for $56K of the damages, and each will pay $$42K. If 2 is insolvent, will be responsible for 40% of 2s insolvency, and 1 will be responsible for 30% of 2s insolvency. Therefore, the will receive $(56K 24K) and 2 will pay $(42K + 18K) 3 Set-off: a claim and counterclaim may not be set off from each other, except by agreement of both parties this is designed to cover situations in which insurance exists on both sides so that injured parties will maximize their recoveries sometimes the court will make everyone pay out, and then reallocate if each party is insured, the parties will not normally consent if each party owes the other $50K, then neither would get anything if there was a set-off. If both insurance companies paid out, then each party would win. 4 Right of Contribution: a right of contribution exists between two parties who are J&S liable for the same claim, whether or not judgment has been entered against all of them. Contribution is available to a person who has entered into a settlement only if His liability has been extinguished, and To the extent that the amount paid in the settlement was reasonable 5 Enforcement of Contribution: if proportionate fault has been established, party paying more can seek contribution upon motion if proportionate fault has not been established, party paying more can seek contribution through a separate action an action for contribution must be commences within one year of the final judgment 6 Effect of Release: a release covenant not to sue entered into by the claimant and the person liable discharges that person from all liability, but it does not discharge all others who are liable the others damages are reduced proportionately Reckless Conduct: Under contributory negligence, if the s act was reckless, the s negligence was ignored Now, under a pure system, the conducts are still compared Some states will bar recovery when the has acted socially offensive Ex: setting off a bomb in his hands Generally, the closer the is to willful and wanton conduct, the less it will be compared with the s conduct, and vice versa Intentional Tort or Crime by :

When the acts intentionally, and the negligently, most courts will refuse to compare the acts because the s intentional tort is of a different kind and they cannot be compared Aggregation of s conduct Should s conduct be compared to the conduct of all s combined, or to each one separelty? Multi-party Suits where not all s settle Ex: 1 and 2 are equally liable. settles with 1 for 10K, and then at trial the court finds that wins $30K. Options: 1) Old Uniform Contribution Act: collects $20K from 2, and then 2 can seek contribution from 1 2) New Uniform Contribution Act: collects $20K from 2, and 2 cannot seek contribution from 1 so long as 1s settlement was in good faith 3) UCFA (6):s recovery is reduced by 1s proportionate share, and can recover only $15K from 2 4) NY: court reduces 2s liability by the larger of 1) 1s equitable share 2) the actual amount 1 paid in settlement Imputing the negligence of one to another : Majority of States: Actions are derivative The defenses available against the direct victim are available against the Few states: Actions are independent Defenses are not available to both Different types of cases: Loss of consortium: Most states treat it as derivative Harm affected the couple/family as a unit Wrongful Death: Most states treat it as derivative Ex: if the childs negligence was not imputed to the parents, then the parents would be able to recover 100% for their child who was 99% negligent Bystander Emotional Distress: Most states treat it as independent Someone suing for seeing his father hit by a car is not suing for his fathers injury, only for having to witness it It would only matter if it was completely the s fault Parent-Child: Most states treat it as independent Changes brought about by the comparative Negligence doctrine : Last Clear Chance: eliminated Res Ipsa Loquitor: When the evidence clearly shows undeniable contributory negligence from the , RIL could be used if it met only the first two conditions Then the jury must use it to decide Rescue: Courts have agreed that rescuers, who were not held at fault earlier unless reckless, no longer needed special protection The drinking : Estates of people who were killed from drunk driving were able to sue the supplier of the liquor Deterrence Subsequent Harm: Where s were no longer liable for possible further harm or aggravation that resulted from a doctor after they were injured due to the s negligence Economic cases: Generally apply the doctrine Fritts ( /injured in an accident) v. Mckinne ( /doctor): BLL: The negligence of a party, which necessitated medical treatment, is irrelevant to the issue of possible subsequent medical malpractice Facts:

was involved in an accident, and had been drinking the underwent surgery to repair his facial features, and died when the performed a tracheotomy to allow him to breathe during the surgery lost blood when he tried to locate the artery, which should normally be in the chest, but was instead in ht neck wanted to bring in evidence of the s drinking problem since it caused the accident, and since it decreased the life expectancy of the jury returned a verdict in favor of Issue: Was the evidence of s past drinking problem admissible? Ruling: NO argues that the rupture of the artery was inevitable However, the evidence of s possible negligence in the auto accident is a matter unrelated to medical malpractice There are certain situations where it would be relevant: Where the patient failed to disclose certain things, where the patient failed to follow the doctors advice or delayed in treatment, where the court is trying to determine the life expectancy or future earnings A doctor may not avoid negligence by saying that the injury was caused by the patients own negligence Those patients are still entitled to non-negligent medical treatment The court should have bifurcated the issues if it were relevant Notes: In all the cases thus far, the s negligence came after the s. what if it was reversed? Ex: elevator stopped working 6 feet from the floor of the lobby. The jumped, and hurt himself. The court found that the s jump was not foreseeable in the course of events resulting from the s negligence. The s life was not threatened. The s jump terminated s liability. Avoidable Consequence Def: Even if the accident was entirely the s fault, the s recovery may still be reduced for failure to exercise due care to mitigate the harm. Ex: failure to get medical attention or to follow medical advice The court draws the line between treatments that involved a recognized risk and those that did not Duty to mitigate in the cases where there was not a recognized risk Ex of a recognized risk: if the proposed treatment could result in an aggravation or additional problems, or if the prospect of improved health is slight, then there is no duty to undergo the treatment Once grounds for an exception are established, the need not give reasons for rejecting the procedure Reasons based on religious beliefs Although a partys decision to honor her religious belief is not unreasonable, has not duty to subsidize her choice to sacrifice her like in the name of religion Failure to use seatbelts/helmets: Traditionally, the ability to mitigate came only after the original injury This is an anticipatory avoidable consequence Some state legislatures have taken action on this Avoidable consequences can at as comparative fault in states that have a 50% threshold The could recover nothing

Assumption of Risk
Express Agreements Emanuel page 116 Parties can sometimes agree in advance that the need not exercise due care in regards to the Generally through a written contract When the is hurt by s negligence, 2 issues generally arise: 1) will the courts enforce even the most clearly drafted agreement given the type of activity invovlved 2) if so, is the contract in question sufficiently clear Dalury ( /injured skier) v. S-K-I, Ltd. ( /ski resort): BLL: Facts: was injured when skiing when he hit a metal pole in the ski lift line He had purchased a season pass, and signed a contract that said, in part: RELEASE FROM LIABILITY AND CONDITIONS OF USE:

I freely accept and voluntarily assume the risks of injury or property damage and release Killington Ltd.,
its employees and agents from any and all liability for personal injury or property damage resulting from negligence, accepting myself the full responsibility for any and all such damage or injury of any kind which may result. also signed a photo ID card that contained this same language claims, on appeal, that the release was ambiguous as to whose liability was waived and that it is unenforceable as a matter of law because it violates public policy The court feels that the language was quite clear, and only addresses the public policy concerns Issue: Were there social interests that were affected that prevent the contract from being enforceable? Ruling: YES Case of 1st first impression in Vermont. In most of our cases, enforceability has turned on whether the language of the agreement was

sufficiently clear to reflect the parties' intent. Even well-drafted exculpatory agreements, however, may be void because they violate public policy. Restatement (Second) of Torts 496B comment e (1965): An exculpatory agreement should be upheld if it is (1) freely and fairly made, (2) between parties who are in an equal bargaining position, and (3) there is no social interest with which it interferes.

The critical issue here concerns the social interests that are affected . Various Court formulas for analyzing the public policy issue.

Leading judicial formula (by Justice Tobriner of the California Supreme Court): An agreement is invalid if it exhibits some or all of the following characteristics (Tunkl

factors): [1.] It concerns a business of a type generally thought suitable for public regulation. [2.] Party seeking exculpation is engaged in performing a service of great importance to
the public [3.] The party is willing to perform this service for any member of the public [4.] The party invoking exculpation possesses a decisive advantage of bargaining strength against any member of the public [5.] party uses a standardized adhesion contract of exculpation, and makes no provision whereby a purchaser may pay additional reasonable fees and obtain protection against negligence. Adhesion contract: take it or leave it

[6.] person is placed under the control of the seller, subject to the sellers risk of carelessness The Colorado Supreme Court has developed a four-part inquiry (Jones case):
Depends on: (1) existence of a duty to the public, (2) the nature of the service performed, (3) whether the contract was fairly entered into, and (4) whether the intention of the parties is expressed in clear and unambiguous language. In this case, court found that no duty to the public was involved in air service for a parachute jump, because that sort of service does not affect the public interest. Virginia Supreme Court recently concluded: In a Triathlon competition, that a preinjury release from liability for negligence is void as against public policy because it is simply wrong to put one party to a contract at the mercy of the other's negligence. Public policy forbids it, and contracts against public policy are void. These public policy formulations are good guidance, but if met, do not preclude further analysis. I We conclude that the "determination of what constitutes the public interest must be made considering the totality of the circumstances against the backdrop of current societal expectations.

Do ski resorts provide an essential public service? s argue:

they owe no duty to to permit him to use their private lands for skiing Skiing is not a necessity of life just a recreational sport It is a private transaction
Court:

Whether or not s provide an essential public service does not resolve the public policy concern. It is a facility open to the public. They advertise and invite skiers and nonskiers of every level of skiing ability to their premises for the
price of a ticket. When a substantial number of such sales take place as a result of the seller's general invitation to the public to utilize the facilities and services in question, a legitimate public interest arises. The major public policy implications are those underlying the law of premises liability.

In Vermont, a business owner has a duty "of active care to make sure that its premises are in safe
and suitable condition for its customers." There is a duty of care where the 's routine business practice creates a foreseeable hazard for its
customers.

We have already held that a ski area owes its customers the same duty as any other business. Defendants, not recreational skiers, have the expertise and opportunity to foresee and control hazards, and to guard against the negligence of their agents and employees. If s were not responsible, an important incentive for ski areas to manage risk would be removed with
the public bearing the cost of the resulting injuries.

A recognition of the principles underlying the duty to business invitees makes clear the inadequacy of
relying upon the essential public service factor in the analysis of public recreation cases. When a facility becomes a place of public accommodation, it "render[s] a 'service which has become of public interest' in the manner of the innkeepers and common carriers of old." "Acceptance of inherent risks" statute, 12 V.S.A. 1037

s argue: that Legislature is acting to limit ski area liability by allowing the use of express

releases. On the contrary: The statute places responsibility for the "inherent risks" of any sport on the participant, insofar as such risks are obvious and necessary.

Notes: An inherent risk does not include negligence The courts generally do not allow the to exculpate themselves for negligence Tort law wins over contract law Courts will insist on the magic words: You are released from negligence would it be better just to act for a little bit, so that the court does not hold the entire agreement unenforeceable? The intention in drafting these agreements is to be very vague. Different than saying any and all claims Even if the release is valid, courts have not generally bound one spouse to the other spouses signed release Iowa case: The H went to a race track and signed an agreement. H was injured, which caused loss of consortium for W. The court did not impute negligence onto W Minors: Why does the minor sign the release before the claim? Can the ski area foresee the third party parent suing? Imputing negligence (views): is still responsible for a certain %, so it should not matter what happened on the s side Crashworthiness: It became a standard that a car had to be able to generally withstand crashes Air bags, side beams, etc.

s claims could depend on the amount of damage due to the car Mfrs negligence seatbelts: should the be liable for the damages that would not have happened if he was wearing his seatbelt? Similar to the Fritts case, where the doctor argues that his negligence should be reduced by the s negligence in causing his injury Contributory Negligence Jurisdictions: Courts often refuse to allow the seat belt defense at all s failure to wear a seat belt does not count against his failure to recover Comparative Negligence Jurisdictions: Seat defense is more successful Various approaches: is liable only for those injuries that would have occurred even had worn a seatbelt is liable for all injuries, with a reduction made equal to s fault % is liable for all injuries, but s recovery is reduced by those injuries that could have been avoided Implied Assumption: General: may be held to have assumed certain risks by her conduct 2 Requirements: For to establish implied assumption, he must show that s actions demonstrated that she 1) knew of the risk in questions strictly construed the risk must be one that was actually known to , not merely which ought to have been known 2) voluntarily consented to bear that risk herself strictly construed Duress: There is no assumption of risk if s conduct left with no reasonable choice but to encounter a known danger Choice not created by : Where it is not s fault that has no reasonable choice, except to expose herself to the risk, the defense will apply. Distinguished from contributory negligence: Also, s implied assumption of risk will also constitute contributory negligence Ex: voluntarily, but unreasonably, decides to take her chances with a certain risk But may not if: it the act was really reasonable (normal people would do it) Reckless conduct by : s actions were reckless, and contributory negligence would not be a defense, but implied assumption of risk usually is Primary v. Secondary Assumption of risk: Primary: is never under any duty to at all Ex: foul balls at a baseball game Secondary: would ordinarily have a duty to , but s implied assumption of risk causes s duty to dissipate. Virtually meaningless today It only means that there is some fault of the to be compared to the s fault, and it is for the jury to figure out Effect of comparative negligence statute : When there is a comparative negligence statute, most states eliminate the secondary assumption doctrine, but not the primary If the states eliminate the secondary assumption doctrine, the court must look at whether the s acts were reasonable in order to determine if they will reduce his recovery Sports and Recreation: Participants often sue each other. Most courts hold that the participant assumes the risks that are inherent in the sport, which is a primary assumption, and is a complete defense under comparative negligence When the s actions are reckless, the other participants assumptions are secondary, and it is not a defense under comparative negligence Murphy ( /Amusement seeker) v. Steeplechase Amusement Co. ( /Amusement Park):

Assumption of Risk Sports and Recreation Implied Primary Assumption of Risk BLL: Once who takes part in a sport accepts the inherent dangers that are obvious and necessary if they are not so serious so as to justify the belief that precautions of some kind must have been taken to avert them. Facts: The ride, the Flopper is a ride that tosses people around on a moving floor , a vigorous young man, visited the park, and observed the ride before getting on when getting on, he felt a sudden jerk, and fell and broke his knee cap he claims that he fell on a wood patch Issue: was the injury a foreseeable risk of the adventure? Ruling: YES claims that was negligent because: the equipment was not able to prevent injuries to people without knowledge of its dangers it was dangerous to life and limb there was no proper railing or guard does not have solid evidence: the belt was not out of order, and there is no proof of a jerk the belt was already moving when he stepped on the power transmits smoothly to the belt, and cannot be done otherwise does not matter if there was a jerk because a fall was expected he claims that he fell upon wood contradicted by photos and his friends testimony even if there were defect, it was not one obvious or known Volenti non fit injuria (no wrong is done to one who is wiling): The basis of the assumption of risk doctrine What one has willingly chosen to risk is usually determined subjectively Once who takes part in a sport accepts the inherent dangers that are obvious and necessary. Like a spectator at a ball game It would be different if they are so serious, obscure, and unobserved so as to justify the belief that precautions of some kind must have been taken to avert them. The president of the theme park stated that there were no other accidents However, a nurse stated that there were some, but nothing too serious This testimony is not enough to show that the ride was a trap for the nave 250,000 people ride it every year A fall was foreseen as one of the risks of the adventure took a chance the case did not go to the jury about the defect in the floor the case went to the jury upon the theory of whether the negligence was dependent upon a sharp sudden jerk Notes: When the is arguing that there was an implied assumption of risk, he is arguing that there was no duty It was not unreasonable for the to engage in that risky behavior, and it was also not unreasonable for the to have that risky ride The can argue that he watched everyone on the ride, and did not see anyone else get hurt, so he could not have known the risk Suggests that it is reasonably safe But Cardozo is saying that the small amount of injuries means that it was reasonably safe Participants in sports: Sometimes participants sue other participants Some courts hold that Liability results only when the intentionally injures the or is so reckless that he is outside the range of the ordinary activity of the sport has consented to accept certain risks if there was liability for everything, people would be afraid to give it their all in sports would also encourage litigation it would be different to judge the acceptable conduct of the game when it is played differently everywhere Other courts hold that it depends on whether the was negligent

Depends on the rules and customs of the sport, risks inherent in the game, the presence of protective equipment, participants skills, participants knowledge of the rules. Ex: a skier colldided with another while he was drunk. Even though the had no duty to avoid collisions on the hill (inherent risk), he had the duty not to increase that risk Baseball Spectators: Ex: spectator gets hit by a foul ball behind home plate The duty of care owed by baseball field to the spectator: The field provided screening where the chances of getting hit were high This was sufficient protection as was reasonably expected duty was fulfilled The spectator assumes other risks of injury Davenport ( /Injured Lessee) v. Cotton Hope Plantation Property ( /Owner): Assumption of Risk not a total bar to recovery in a modified comparative negligence system Primary v. secondary BLL: A is not barred from recovery by the doctrine of assumption of risk, unless the degree of fault arising therefrom is greater than the negligence of the Assumption of risk is abolished as an absolute bar to recovery Facts: complained that a stairways lights went out, but he continued to use it (even though others were available for use) he tripped one time, and claims damages for injuries asserts that there was an implied assumption of risk, and this bars total recovery from the Issue: whether implied assumption of risk bars total recovery under S.Cs comparative negligence (modified) system. Ruling: Four requirements to show that there was an implied assumption of risk : 1) must have knowledge of the facts making the condition dangerous 2) must know that the condition is dangerous 3) must appreciate the extent of the danger 4) must voluntarily expose himself to the danger Categories of Implied Assumption of Risk: Primary: Arises when the impliedly assumes risks that are inherent in the activity Not a true affirmative defense Instead goes to the initial investigation of a negligence case of whether there was a duty owed to the It is another way of saying that has failed to state a prima facie case of negligence by failing to show that the owed him a duty. If no duty, no negligence Its just a part of the initial negligence analysis Secondary: Arises when the knowingly encounters a risk created by the s negligence It is a true defense because it is asserted only after the states a prima facie case of negligence by showing that the owed him a duty. May involve reasonable or unreasonable conduct on the part of the Reasonable implied assumption of risk When the is aware of the risk negligently created by the , but voluntarily proceeds to encounter the risk When weighed against the risk of injury, it was reasonable Different jurisdictions: Very few comparative negligence states have retained implied assumption of risk as an absolute defense Rhode Island: Comparative negligence is a subjective standard implied assumption of risk: objective standard implied assumption of risk is different than contributory negligence: implied assumption of risk: knowingly does not matter whether reasonable or not contributory negligence: there is no free will in encountering the risk

Notes: Express assumption of risk can still be a total bar to recovery. It is based on contract law, not tort law Rest 2nd of Torts 496E: Assumed risk must be voluntary It is not voluntary if the s tortious conduct has left the with no other alternatives to protect or exercise a right of which the has no right to deprive him. Minority View: States begin with the assumption of risk defense, without first looking to the negligence analysis Swimming Pools: Courts do not normally hold the swimming pool liable for someone who dives and hurts themselves The behavior is a superceding act of negligence absolving the s The pool is an open and obvious danger, and the obviousness of risk negates any duty Workers Comp: The doctrine of assumed risk was a major factor in the development of workers comp Trade-off between higher wages and insurance Recognition of the markets desire for risk for increased wages Workers were more afraid of not having a job than having a job with potential risk Roberts ( /Volunteer Firefighter) v. Vaughn ( /Tortfeasor): Implied Assumption of Risk Firefighters Rule BLL: The firefighters rule, barring recovery for injuries as a result of the negligence that gave rise to emergencies duties, does not apply to volunteers Facts: , a volunteer firefighter, was injured when trying to recuse someone who was pinned under the dashboard she kicked him and caused permanent disability received no pay, pension, benefits; he received only training and equipment Issue: whether, notwithstanding any negligence on the part of the that may have caused s injuries, is barred from recovery because he is a volunteer firefighter responding to an emergency Ruling: NO Firefighters Rule (from Kreski): Rule: Basically bars firefighters or police officers recovery for injuries sustained as a result of the negligence that gave rise to their emergency duties Justification for the rule: Public Policy They were already obligated to perform for which they received compensation (from the taxpayers) that added in the considerations of risk The negligence causes the occasion for the safety officers presence Taxpayers pay for the officers to deal with results from their own negligence The applicable duty of care toward the safety officer is replaced by the taxes paid to them The nature of the service The relationship of the officer to the public Rule may not have to be applied where rationales are not implicated Gibbons:

West Virginia: Requires actual knowledge of the dangerous situation ( majority rule) But bars it as an absolute defense Unless the s degree of fault is equal to or exceeds the combined negligence of the s (this is the modified system) An absolute bar to recovery is comparable to contributory negligence at common law Both say that it is different from contributory negligence But, should a be completely barred from recovery when he voluntarily assumes a known risk, regardless if that assumption of risk was reasonable or not? No- adopt West Virginias system Justification for adopting a comparative negligence system to allocate fault Justification for adopting a assumption of risk system not to allocate fault, but to prevent a person who knowingly and voluntarily incurs a risk of harm from holding another person liable Express and Primary are not affected by this decision

All risks encounters by officers do not fall within the rule If a third party, not connected to the scene, injures the officer through gross negligence, or reckless behavior, the rules does not apply Why volunteer firefighters are different than paid: They do not receiver compensation from the taxpayers for the risks involved Therefore not the same relationship to the public To deny recovery by saying that the duty of rescue theory applies because their acts are gratuitous is reaffirming the doctrine of assumption of risk that was rejected in Kreski as a rational.

Notes: Firefighters Rule is on its way out Will usually say that the firefighters rule as a backup Several states have abolished it. Risk invites rescue If someone has acted negligently, and someone else goes in to rescue him, he can sue the original person if he gets hurt while rescuing Geir: Shows how each state has different tort law Supremacy Clause: Federal Law triumphs over state law Want certain product Mfr to adhere to the regulations But sometimes need them to go further Need the common law to enforce the specific standard, not just the regulation agency Ex: pharmaceuticals

Strict Liability
Doctrinal Development
development/reasoning for introducing strict liability: permits a broader distribution of risk many people pay rather than one reduces the administrative burden makes proving fault easier Could help reduce losses Make business factor in the cost in the price It may determine how important it is to sell that dangerous product Helps to internalize the losses Negligence was a barrier to all the benefits Takes too long In cases where you cant really do anything, strict liability is appropriate In determining when to apply strict liability, must ask: What heavy lifting is strict liability doing for us? Different than RIL: Negligence is implied Animals: Trespassing Animals: In most states, the owner of livestock or other animals is liable for damage caused by the animals if they trespass on another owners land Even if the owner provides the utmost care in preventing them from escaping Non-trespass liability: The owner is responsible for any damage done by his dangerous animal who does not trespass Wild Animals: Owner is liable as long as the damage results from the dangerous propensity that is typical of the species in question Ex: lions can attack without warning, so the owner would be liable if one did Domestic Animals: Injuries from a domestic animal do not give rise to strict liability unless the owner knows or has reason to know of the animals dangerous characteristics Abnormally Dangerous Activities:

General: A person is liable for any damage he does while he is conducting an abnormally dangerous activity 6 factors to determine whether an activity is an abnormally dangerous activity: 1) if there is a high degree of risk to others 2) the harm that results is likely to be serious 3) the risk cannot be eliminated by the exercise of reasonable care the most important factor Ex: using dynamite to tear down a building 4) the activity is not common 5) the activity is not appropriate for the place where it takes place 6) the danger outweighs the activitys value to the community Examples: Nuclear reactor Use of explosives Crop dusting Limitations: Scope of Risk: There is strict liability only for damage which results from the kind of risk that made the activity abnormally dangerous Ex: is carrying dynamite on his truck. He hits , and kills him. must show negligence, since s death did not result from the kind of risk that made this activity abnormally dangerous Abnormally Sensitive Activity by : will not be liable for his abnormally dangerous activity if the harm would not have occurred except for the fact that was conducting an abnormally sensitive activity Contributory Negligence: Ordinary contributory negligence will not bar from recovery for strict liability Unreasonable Assumption of risk: But, assumption of risk is a defense to strict liability Whether reasonably or unreasonably assumed risk Workers Comp (WC): Generally, all states have adopted it Compensates the employee for on-the-job injuries without regards to whether it was the employers fault or the employees. Arising out of Employment: Must be an injury arising out of and in the course of employment Exclusive Remedy: The WC statute is the employees sole remedy against the employer The employee gives up the right to sue in tort, and cannot claim pain and suffering Intentional Wrongs: If the employee can prove that the employer intentionally injured him, the employee may pursue a tort action Ex: where the employer has willfully disregarded safety regs (rare) Third Parties: Employee can still sue third parties would be responsible for his injuries Ex: machine Mfr Products Liability: Now strict liability is usually only used in products liability cases Fletcher ( /Mine Owner) v. Rylands ( /Reservoir Owner): Strict Liability not naturally there BLL: A person who brings something onto his land which is potentially harmful if it escapes is strictly liable for all the natural consequences of such escape. Facts: decided to construct a reservoir on his land to hold water he hired contractors to do the work they did not know this at first, but soon after discovered that there were deserted mine shafts beneath the area to be used for the reservoir they did not know that these shafts connected to shafts beneath the s land the water from the reservoir filled the shafts, and flooded the s land Issue:

Whether the is liable for damages when he lawfully brought something on his land, which was harmless when there, but caused damages when it leaked naturally onto the neighbors land. It is agreed that there is a duty to take care of and keep what he brings on his land there, but it is unclear whether there is an absolute duty to keep it in at his peril, or merely to take reasonable and prudent precautions. Ruling: Possible options for rule: If there is an absolute duty would be responsible for all natural consequences If the is only responsible for reasonable care Then the would only be liable if he was negligent (did not show reasonable care) Rule: A person who brings something onto his land which is potentially harmful if it escapes is strictly liable for all the natural consequences of such escape. The can only escape liability only if he shows that the was at fault Reason for Rule: The should not be damnified for not fault of his own If there is something that was not naturally on the s land, which is harmless when confined, but problematic when it gets on the s land, the will be liable for damages But for the bringing it on his land, the problem would not have happened. Similar to animals: If an animal wonders onto another persons land, the owner is liable If the owner knows about the animals potential to attack others, he will be answerable Negligence not necessary: Negligence is normally required to recover for damages, but this case is different Different than negligent driving on a highway, where everyone puts himself out there with the risk did not voluntarily assume risk he did not know what the would do he could not control the s Notes: It was not a trespass case or a nuisance case: Trespass: There was no direct and immediate invasion of the s land The s water flowed beneath the land Nuisance: Nuisance is an interference with the s enjoyment and use of his land There was no continuous harm only one unexpected harm But for: Was there a cause in fact? Rylands ( /Reservoir Owner) v. Fletcher ( /Mine Owner): Strict Liability non-natural use BLL: When one uses his land for a nonnatural use that is dangerous, such as artificially accumulating water whose escape could cause damage to neighbors, he is strictly liable for any damage that may result to anothers property thereby. Ruling: If the had dug a reservoir, and it filled with water, and it leaked into the s land, the would not be liable If the s used the land for a non-natural use, then s were doing it at their own peril It does not matter how careful he was, or how many precautions he took. Notes: At first, Americans were not very accepting of this rule because: Changes and use of land are needed for the development of society You hold your property to risk, just as when driving on a highway The use of land depends on what the use of land was made for in the grant But today: It is a complex society Many areas are overcrowded, and even non-negligent use of land can lead to damages Now there is an environmental factor Sullivan (/Decedent) v. Dunham (/Dynamite blasting land owner): Strict Liability direct harm v. concussion

BLL: One is strictly liable for injury to a person or property which is occasioned by trespass resulting from his blasting activities. Facts: hired 2 men to dynamite a 60-foot tree on his property. The blast hurled a fragment of wood 412 feet onto the highway, which killed the , who was walking along the highway. s decedent sued under the states statute gives right to sue for wrongful act, default, or neglect causing death also covers trespass Issue: whether the s dangerous activities, though lawful and without negligence, that cause injury to another person, cause him to be liable for all injuries. Whether the is liable in trespass Ruling: YES Leading case law of trespass on land: blasted out rocks, which hit s house, and he was held liable for all damages without the showing of negligence the had the right to modify his property, and the had the right to possess his property undisturbed if these 2 rights conflict, then has more of a right public policy: it is better that one man should surrender a particular use of his land, than another should be deprived of the use of his land altogether the use of land is not an absolute right it is qualified and limited by the higher right of others to enjoy their property the law prohibits direct injury, regardless of the s motives Comparing cases of trespass of land to trespass of a person : had the same right from protection of injury as if she had been walking upon her own land St. Peter: Blast sent a piece of frozen earth into the s work and killed him was liable had a right to stand there Purposeful v. accidental: Hay case: Not liable for accidental explosion (concussion damage) in the absence of negligence But if direct and immediate, is liable Sic utere tuo: To protect a person and property from direct physical violence, which, although accidental, has the same effect as if it were intentional. Notes: The distinction between debris and concussion has virtually disappeared Indiana Railroad (/Railroad) v. American Cyanamid (/Chemical Mfr): Strict Liability abnormally dangerous activities BLL: Strict Liability will not be imposed against the fr of a dangerous chemical for accidents occurring during transportation. Facts: Acrylonitrile: Highly flammable and toxic, carcinogen loaded 20K gallons of acrylonitrile into a railroad tank car, that it had leased from North American Car Corp, in LA Missouri Pacific Railroad picked it up The car was supposed to switch over to Conrail at the Indiana railroad in Chicago. After it arrived, they found that it was leaking, and the Illinois Dept of Env Protection ordered a clean-up that cost the stockyard almost $1million. Issue: is the chemical mfr subject to strict liability for accidents that occur during transport of a dangerous, but lawful, chemical? Ruling: NO Look to 6 factors of Rest. 2nd 520 to determine whether an activity is an abnormally dangerous activity: 1) if there is a high degree of risk to others

2) the harm that results is likely to be serious can be a different result that may have happened used to prevent such accidents 3) the risk cannot be eliminated by the exercise of reasonable care the most important factor if it is missing, there is not action for common law negligence should probably start with this factor if there way the accident can e avoided, then look to strict liability, not negligence by denying the an excuse, the must take preventative measures instead, rather than just actig reasonably the greater the risk of the accident (1), and the greater the harm that results (2), the more we want the to take action to reduce the possibility of accidents the stronger the case for strict liability Ex: using dynamite to tear down a building Dangerous even when handled carefully 4) the activity is not common so there is no assumption that it was a highly valuable activity despite its riskiness if the activity is extremely common, it is unlikely that the risks are great or that there is no way to avoid them strict liability will less likely be enforced 5) the activity is not appropriate for the place where it takes place city v. rural 6) the danger outweighs the activitys value to the community Guille: man landed a hot air balloon unintentionally in a field of vegetables and caused damage. He was found strictly liable. These factors are used to govern cases which negligence cannot control Siegler: s gasoline truck broke away, and hit and killed the transporter was strictly liable, not the gasoline Mfr court that the RIL could also apply Negligence is applicable here (therefore, strict liability is not) : It could have been avoided The chemical, although toxic and flammable, will not eat through the container There are many possible reasons for negligence: Leaky tank Carelessness Failing to inspect the car Switching cars ruptured the lid Unlike Siegler, when a lack of care can be shown, such accidents are adequately deterred through an action for negligence If the tank is adequately maintained, the fear of a spill in negligible Using strict liability to deter these type of accidents would not do much The feasibility of avoiding accidents simply by being careful is an argument against strict liability Also Unlike Siegler, it is not the transporters that the is seeking to find liable, but the Mfrs. Cannot ask the Mfr to reroute: It would be too much cost A common carrier cannot refuse to ship a lawful commodity Notes: The Rest. states activities rather than acts for products liability purposes Ex: keeping a tiger in ones backyard is something that is abnormally dangerous Strict liability does not induce the owner to take due care, it provides incentive to get rid of the tiger all together This is because the exercise of due care is not an option for defense Rest 2nd has made owners of airplane liable to person on the ground The FTCA does not permit recovery against the U.S. government for strict liability Reason why RIL does not apply:

Cannot determine who was responsible for the defect Is the world really affected by strict liability? Not really Mfrs will likely act the same way, weighing the burdens against the accident rate s will still try to claim both negligence and strict liability wont really lessen the judicial burden Care gets you a little bit, but after that, continuing reasonable care can only do so much

Theoretical Perspectives
Enterprise Liability (Rabin): The safety of persons is more sacred that the safety of property Therefore, the owner of land does not have an absolute right, but is limited by the higher right of others to lawfully possess their property Sic utere tuo and trespass: Sic utere tuo: Use you own so as not to injure anothers property Trespass: the buffer against invasive conduct Both were the touchstones of traditional strict liability They were grounded in norms of interpersonal conduct Chavez: 18 boxcars exploded it was required that a common carrier accept dangerous, legal cargo court found that there was no logical reasoning for creating a public duty exception when the reason for subjecting a common carrier to absolute liability is because it has the ability to distribute the loss to the public the s are defenseless the social and economic benefits which are ordinarily derived from strict liability are achieved this case clarified the reasoning and firmly anchored strict liability in collective justice/enterprise liability vicarious liability: modern products liability theory offers both risk-spreading and safety incentives rationales for holding employers responsible for the tortious acts of their employees. Enterprise liability notions: Enterprises should bear the risks of accidents it produces because: 1) enterprise has the superior risk-bearing capactity compared to the victims 2) an enterprise is generally in a better place to respond to safety incentives Goals-Oriented Approach for Abnormally Dangerous Activities : Goals of Strict Liability: Courts often base cases on different goals, if any goals at all. Loss-Spreading: Goal: to spread losses caused by accidental injuries among a broad class of persons Imposing accident costs collectively, not individually, because it causes less social and economical disruption if it is shared by many people The decision to impose strict liability: Depends on whether the has the ability to incur and then spread the loss That depends on If the was able to anticipate and evaluate underlying risk If the was able to take steps to accumulate resources to insure against the loss If the is able to pass the cost onto consumers Should not be considered alone Workers comp, SS, etc. must also be taken in to account If loss-spreading were the only goal, there would be no stopping point short of holding the gov liable The dominant argument for enterprise liability stems from the fact that the industry is the most practical party to administer the loss-bearing Loss-Avoidance (Risk Reduction): Sometimes referred to the primary reduction of accident costs Aims at imposing liability by reducing the number and severity of accidents Requires the actor to systematically evaluate the risk of his activities and make sound cost-benefit decisions about the manner of operations as well as the level of and location of activity, safeguards, and alternatives

Activities that could reduce accident costs most cheaply Should not be considered alone, and should be given less weight than loss-spreading considerations because: 1) there is a contradiction within the goal liability may be imposed on the even if her is innocent, but the loss-avoidance goal assumes that the could have changed things for the better. 2) deterrence, or loss-avoidance incentives, requires an often unattainable knowledge of relevant risks by responsible decisionmakers the extent of the injury often will not coincide with the appropriate level of deterrence there is an absence of predictable outcomes of litigation and standards of behavior behavior disorders may cause different people to respond differently the system lacks the consistency to reinforce behavior 3) threat of liability can over-deter high prices or unavailability of products cause consumers to turn to substitutes that are less safe may only help to educate consumers about the true costs Role of negligence: It cannot be proven, so strict liability may be more of a deterrence than negligence law The reasonableness of the s activity: Would be nice to know, but courts would have to define the appropriate level of activity, which would be very difficult It is easier to just impose strict liability Loss-Allocation (Internalization): Objective is for the loss to be initially borne (internalized) by the enterprise whose activities are sufficiently connected to the loss in order to reflect the cost in the enterprises services Encouraging investment in safety Discouraging consumption in hazardous products and services It is intertwined with other goals It serves to inform the enterprises and consumers of the true cost of an activity and ways to reduce the net costs of accidents Downsides: If taken too far, can cause enterprises to internalize too many costs and will inhibit technological innovation and economical development. May cause alternate modes of behavior Selling on the black market Domestic enterprises will be less able to compete with international markets international markets are less burdened by the potential of liability Administrative Efficiency: The goal is to reach an acceptable level of administrative costs Reducing the tertiary costs of accidents: The systematic transaction costs involved in imposing liability Reduction in costs from using strict liability: Having to prove fault Promoted recovery in cases where the evidence was destroyed or unavailable Increasing costs from using strict liability: Increase in tort claims Fairness: Paradigm of reciprocity: Focuses on the relative magnitude and quality of the risks created by the activites of the and those of the victim A victim has the right to recover that is created by a risk greater in degree and different in order from those created by the victim Paradigm of reasonableness: Animates fault-based liability Represents a rejection of non-instrumentalist and the welfare of the community as the criterion for determining who should receive compensation Ask: was the risk reasonable? There is no clear meaning of fairness It is probably not one of the most central goals

Protection of Individual Autonomy: It is occasionally mentioned Economic Analysis of Law: Differences between negligence and strict liability : 1) care and activity: Strict tort liability means that someone who causes an accident is liable for the victims damages even if the injury could not have been avoided by the exercise of due care PL might be less than B B<PL applies to both standard negligence claims and strict liability Yet there is significant economic differences between the two The actor can impose more care or less activity in order to reduce the # of accidents But courts rarely determine the optimal level of activity This is a judicial shortcoming Ex: railroads v. canal? Railroads are more expensive due to more accidents, but they are more efficient Ex: drive slow, or drive less? Only if the benefits of engaging in the activity are obviously very slight will the court find that engaging in the act itself was negligence Dangerous activities: Wild animals: The best method to prevent accidents would be to get rid of your tiger But it would be different if the tiger was at the zoo Activity change would be prohibitive Explosives: Since construction goes on everywhere, it is unlikely that they will be able to stop using them There is an incentive to consider alternatives though Unhazardous activities: Often courts apply the label of non-natural to these activities, because the newness of the activity makes it more dangerous Little experience with its safety characteristics Best method for accident control may be to cut back on activity and spread it slowly while more is learned 2) Costs of administering these rules: There is one less issue , negligence, which should bring the costs down But there may be more claims to one action since there is no definite cause strict liability operated to insure victims of unavoidable accidents but only beneficial if the cost of insurance through the tort system is less than the cost to the victims of buying accident insurance policies usually not 3) Deterrence: Under strict liability, the is punished for reasonable conduct Under negligence, the is punished for unreasonable care Because of these difference, the tort system will not entirely pick one or the other

Products Liability
Intro
Def: the liability of a seller of a tangible item which, because of a defect, causes injury to its purchaser, user, or bystander. Usually a personal injury Liability can be based upon any of 3 theories (negligence, warranty, strict liability) Negligence: Ordinary negligence principles apply Privity: There is no privity requirement One who negligently manufactures a product is liable for any personal injuries proximately caused by his negligence May sue the manufacturer (instead of the distributor) Bystanders can sue if they can prove that they were a foreseeable

Class of s: Manufacturers: Careless design Careless manufacturing Careless inspections and tests Unsafe packaging and shipping Failure to obtain quality components from a reliable source Retailers: Usually not negligent for selling a defective product Normally brought on warranty, not negligence Other Suppliers: Bailors (rental car companies) Sellers and lessors of real estate hospitals MacPherson (/Car Owner) v. Buick (/Car Mfr): Products Liability Theoretical perspectives/decline of privity requirement BLL: where a product which, if negligently made, will be dangerous to life or property and it is foreseeable that third parties will use or come in contact with it, a cause of action exists in favor of the injured third party Facts: sold the car to a retailer the wheels on the car were from a reputable Mfr it did not inspect the car the bought the car from the retailer the car collapsed due to a defective tire Issue: whether the owed a duty of care and vigilance to any one but the immediate purchaser Ruling: YES Case Law: Narrow Construction: Thomas v. Winchester: The Mfr labeled a drug wrong, and sold it to the pharmacist The pharmacist sold it to the The court found the Mfr liable because his negligence put the in imminent danger. The danger was foreseeable Loop v. Litchfield: Mfr pointed out a small defect to the buyer, who wanted to buy it anyway The buyer leased it for 5 years The Mfr was not liable to the lessee Losee v. Clute: Explosion of a steam boiler The buyer had tested it The Mfr knew that his test was not the final one The finality of the test had a bearing on the measure of diligence needed Broad Construction: Devlin v. Smith: contractor built a scaffold for a painter the painters workers were injured the court held the contractor liable because he knew that if improperly constructed, it would be very dangerous he owed the workers a duty of care, irrespective of the contract with the master Statler: Coffee urn exploded Held that Mfr was liable because The urn became dangerous when not properly manufactured The rule does not just apply to things that are naturally dangerous (explosives) : If the nature of the thing is such that it is reasonably certain that it will place someone in danger when defectively made, then it is a thing of danger Its nature gives the warning of consequences to be expected

If there is knowledge that the thing will be used by third person, the Mfr is under a duty to make it carefully, irrespective of contract Elements needed to enforce liability: There must be a knowledge of danger Not a mere possibility a probability There must be a knowledge that in the usual course of events the danger will be shared by users other than the buyer May be inferred by the nature of the transaction The proximity or remoteness of the relation: There must be a chain of cause and effect between the and Ex: Mfr of a small part may not be liable to the end user Analysis: knew of the probable danger if defective it was apparent that others may be injured there was 3 seats It was apparent that the car would not be used by the retailer Therefore, the had a legal duty to the end user, not the retailer The is not absolved from liability because it bought the tires from a reputable Mfr It was not at liberty to put the product on the market without testing The more probable the danger, the greater need for precaution Notes: MacPherson has been generally accepted in the US Bystanders Sellers who put their name on the product Before liability of a Mfr can be found, it must be established that the defect was caused solely by its negligence and could not have resulted from the negligence of others or the s contributory negligence in misusing the product Privity of contract: A relationship between the parties to a contract is required in order to bring an action for breach

Warranty Usually governed by the UCC Express Warranties: An express warranty is an explicit promise or guarantee by the seller that the goods will have certain qualities UCC 2-313 An express warranty may be created by: (a) any affirmation of fact or promise that relates to the goods (b) any description of the goods when the seller describes the item he is selling, it can be an express warranty that the item has those characteristics (c) any sample or model if the buyer is shown a sample or model, it can be an express warranty that the other items will have those characteristics Implied Warranties: Implied Warranty of Merchantability: UCC 2-314(1): Unless excluded or modified a warranty that goods shall be merchantable is implied in a contract for their sale if the seller is a merchant with respect to goods of that kind. Merchantable: UCC 2-314(2)(c): The goods must be fit for the ordinary purposes for which such goods are used. Ex: If someone buys a car, and due to a manufacturing defect, the car does not go above 25 mph, this would be a breach of implied warranty of merchantability since a car is generally used for highway driving (above 25 mph), even though the seller did not expressly promise this. Always given unless disclaimed: The implied warranty of merchantability is always given by a merchant seller, unless it is expressly excluded by a disclaimer that meets stringent formal requirements imposed by the UCC. Fitness for a Particular Purpose:

Depending on the circumstances, a seller may be found to have impliedly warranted that the goods are fit for a particular purpose. UCC 2-315: If the seller knows or has reason to know that the buyer needs the goods for a particular purpose, and the buyer is relying on the sellers skills or judgment, there is an implied warranty (unless excluded under 2316) that the goods will be fit for that purpose. Exclusion or Modification of Warranties: The UCC limits the extent to which a seller may disclaim a warranty. Express Warranties: 2-316(1): Seller is free to disclaim express warranties, as long as it is done in a clear, reasonable way. Implied Warranties: Disclaimers of the 2 implied warranties (merchantability and fitness for a particular purpose) are tightly limited by the UCC: Explicit Disclaimers: The seller may make an explicit disclaimer of the warranty in compliance with strict procedural rules: 2-316(2): Merchantability: Must mention the word Merchantability. Does not have to be in writing, but if it is in writing, it must be conspicuous (all caps, boldface, large font, or different color) Cannot be in fine print 2-316(2): Fitness for a Particular Purpose: Must be in writing, and it must be conspicuous. BUT, it does not need to use any particular words, as in the disclaimer for a warranty of merchantability. For ex: there are no warranties that extend beyond the face of the contract OK if conspicuous. Implied limitations and disclaimers: Ways in which a limited warranty may be implicitly limited or disclaimed. 2-316(3)(a): Language of the sale: The language of the sale may implicitly disclaim the warranty If the sale is made as is it will implicitly exclude all implied warranties 2-316(3)(b): Examination of a sample or model: UCC 2-316(3)(b): If the buyer is asked to examine a sample, model, or the goods themselves, there is no implied warranty with regards to defects that an examination would have revealed. 2-316(3)(c): Course of Dealing: Am implied warranty can be excluded by course of dealing, course of performance, or usage of trade. Ex: if the agreement is to take the goods as is at a lower price. Warranty Example: Ryan v. Grocery Store: Ryan asked the for a loaf of Wards bread. Ryans husband at a slice, which had a pin in it, and he was injured Court held the store: Liable for breach of implied warranty of merchantability since the bread was not of such quality to be eaten. Even though the husband did not buy the bread His wife acted as an agent for him Not liable for fitness for a particular purpose because she asked for a specific brand of bread Strict Tort Liability Sometimes there is no evidence of a defect, but the accident or injury clearly shows that the product had a defect The Mfr is liable for those defects even if the defects were introduced at a later stage of production Justification: Risk spreading Accident avoidance Reducing administrative costs by avoiding long, confusing trials Escola (/Exploding Bottle victim) v. Coca-Cola Bottling Company (/Coke Mfr): Products Liability Res Ipsa Loquitor

BLL: RIL may be applied if can show that the condition of the product did not change after it left the s hands, the exercised due care, and the accident would not have occurred without negligence. Facts: Waitress picked a bottle of coke up, and it exploded She testified that she had handled it carefully An engineer from the bottling company testified that their screen/test is near infallible Issue: Ruling: Elements of RIL: Reasonable Care: It appears that because of Cokes common method of testing bottles: Not likely that the bottles contain defects when they are sent to the bottler that are not apparent by visual inspection A defect that would make the bottle unsafe would be discovered by reasonable and practicable tests Causation: Not clear whether the explosion was caused by an excessive charge or a defective glass Neither would be present if ordinary care was used Control of : had control of the inspection All elements of RIL are present: The may produce evidence to rebut this inference of negligence, and then it is a question for the jury to decide. Concurrence (Traynor): Should be absolute liability rather than RIL when: A Mfr places something on the market, knowing that it should be used without inspection, and it is proven to have a defect that harms humans Irrespective of privity of contract The source of the Mfrs liability is the negligence in the Mfg process Even if there is no negligence, public policy demands that the problem be fixed where it will be most effective The public cannot foresee hazards, but the Mfr can The risk of injury can be insured by the Mfr and distributed among the public as a cost of business s defense: the inference of negligence can be overruled by an affirmative showing if proper care however, an injured person is hardly in a position to refute such defense, since he knows nothing of the s Mfg process the retailer cannot bear the burden of the warranty : the Mfr owes no duty to the retailer, because the retailer had no intention of using the product the injured party is the real party in interest liability is not only owed to the immediate purchaser the Mfr must guarantee the safety of his product even when there is no negligence warranties do not arise only out of a contract because of mass production, a relationship has developed between the Mfr and the end user consumers rely on the good name of the Mfr, and sometimes pay more they rely on the increased inspection Notes: Strict liability used to apply only to food cases California has made further developments, and now it is accepted in a majority of states Greenman: s wife bought a tool fro a retailer, which was made by was hurt was found strictly liable warranty notice requirements do not apply to the consumer need to put the cost of the defect into the Mfrs hands Vandermark: Wrong-sized part on a Ford caused a crash was liable

it could not delegate the final inspection responsibility to the retailer retailer since it was an integral part of the marketing scheme, and it was able to put pressure on the Mfr, the retailer is also strictly liable for injuries the retailer and Mfr would just have to adjust the costs of doing business Elmore: Due to a defect in the car that was present at the time of sale, the was injured Bystanders were entitled to the same strict liability They should be given even more protection because they have no opportunity to protect themselves Strict Liability Extension to others Bailors Franchisors who impose quality control on their franchisees Not normally to companies that finance purchases of others Corporations of successor products: Products Rest 12: Successor liability exists if Agreement Fraudulent conveyance Consolidation/merger Continuation of predecessor Savage Arms: A continuity of enterprise exception Used Goods: Courts do not normally impose strict liability on sellers of used goods, lessors, and occasional sellers of goods They are too far along the chain to know of anything They cannot control the inspections Gov Contracts: A private contractor who followed gov specs to make a product is not liable for inadequacies in the design Causation: Sometimes difficult to prove in products liability cases, but important Hard to show a connection between the defect and the harm when the product is destroyed Hard to claim damage for something that was made worse due to poor safety (car accident) Restatements: 402A of the Restatement (Second) of Torts: one is liable for harm caused by defective products to the end user if: the seller is in that business it reaches the consumer without substantial change rule applies even if the seller has used due care there was no contract between consumer and Mfr a defect is: a product in defective condition unreasonably dangerous to the person at the time it left the s possession Restatement Third: Products Liability: Gives 3 types of defects: Departures from design When the harm could have been avoided or reduced by alternate methods/design Inadequate instructions/warnings

Manufacturing Defects
Def: the liability of a seller of a tangible item which, because of a defect, causes injury to its purchaser, user, or bystander. When a product comes off the assembly line different from, and more dangerous than, the intended product Usually latent dangers The dangers are not normally open and obvious (usually caught) Usually cause a personal injury

Design Defects
Rest. 402: Defect in the product usually must be found to be unreasonably dangerous. Must ring of negligence Prevents the seller from being the insurer of its products It is necessary to prove that there was a design defect and that it was the proximate cause of injury Usually must be used in the intended manner of the product How to prove that the product was defective Show that it failed to perform as safely as an ordinary customer would expect when used in an intended reasonably foreseeable manner If the can prove through hindsight that the products design embodies excessive preventable danger The design risks and inherent dangers outweigh the benefits Relevant factors: Gravity of the danger Likelihood that the danger would occur The mechanical feasibility of a safer design The cost of a new design The adverse consequences of an alternate design The jury looks to the product itself, not the reasonableness of the Mfrs conduct Soule v. GM: Products Liability Risk-Benefit Analysis BLL: Facts: It was raining, and she was not wearing her seatbelt was in a car accident the frame around her feet bent in when she collided s ankles were badly injured claims that the frame was defective from Mfg and Design because that was not a normal consequence of an accident there was no clear evidence of a defect presented expert testimony of improper welding and of other safer designs Jury was instructed under the ordinary consumer expectation test. Issue: May a products design be found defective on grounds that the products performance fell below the expectation of the ordinary customer, if it cannot be determine how the safely the product should have performed by a normal user? Ruling: Different ways to prove a design defect: Ordinary consumer expectation test. Ordinary knowledge of the product characteristics permits an inference that the product did not perform as safely as it should (402) If so (based on facts) no further proof is needed Ex: a car, going 5 mph, just lit on fire Often hard to use for complex products, because it would be hard to predict how it would perform in all foreseeable situations It is defective regardless of expert opinion Risk-Benefit Analysis: Excessive preventable danger Used when the cannot show that the reasonably minimum safety expectations of ordinary consumers were violated When there is more technical and mechanical detail involved Usually expert testimony is necessary It is defective unless the benefits outweigh the risks Involves technical issues of: feasibility, cost, practicality, risk, and benefit competing/alternative designs whether the design chosen was a reasonable one from among the feasible choices of which the Mfr was aware or should have been aware Products Rest. 2:

Must prove that a reasonable alternative design would have reduces the foreseeable risk of harm may be liable if the product has no RAD and the value of the product is minimal Factors to be considered: The magnitude and foreseeability of risks The instructions and warnings for the product The nature and strength of the consumer expectations The advantages/disadvantages of the product Will it ruin a unique feature of the vehicle to make it safer? Costs Production, repair, maintenance, customer choice Factors may be offset by evidence Competing designs may only be compared to the design in question if it is a comparable product It is better because the design defects is better with the knowledge of the Purpose of design Schematics

Notes: Other times where the risk-benefit analysis was preferred Air bags deployed in low impact collisions Use of consumer expectations test Asbestos installations In a crashworthiness case, must also consider the circumstances around the accident itself The irreducibly unsafe product: If there was no reasonable alternative Look at the need for the product Was it essential or a luxury? Causation: The supplier must anticipate uses that were not intended Camacho v. Honda: Products Liability Crashworthiness test for cars/motorcycles (not reasonable consumer test) BLL: where the danger presented by a product would have been within a users contemplation is not relevant in a products liability action Facts: collided with another car on his motorcycle he suffered leg injuries, and claimed strict liability against Honda for failing to provide crash bars 2 MEs testified that the leg bars were state of the art equipment, they were available when the bought his motorcycle, and other Mfrs were using them then. Issue: is the reasonable consumer test applicable? Ruling: NO Crashworthiness Doctrine: Def: Where the defect did not cause the accident, but where it enhanced the injuries The Mfr is liable in negligence or strict liability argues that motorcycles are inherently dangerous and cannot be made perfectly crashworthy court: there is no reason why cars would have to implement cost-acceptable safety devices to prevent foreseeable accidents and motorcycles shouldnt the crashworthiness doctrine does not require absolute safety, just reasonable, cost-effective safety for foreseeable use of the product motorcycle accidents are just as foreseeable as car accidents Strict Liability: The court relies on 402A argues: comment i to 402 says that

The rule applies only where the defect makes the product unreasonably dangerous to the ordinary consumer

Court: Requiring a to prove that the defect made it unreasonably dangerous appropriately places a limit on the Mfrs liability The fact that dangers are open and obvious to the consumer is not an absolute defense This would unfairly bring in the assumption of risk doctrine to a question of law Using that standard would make a product not defective even though there was a safer alternative This standard is particularly in appropriate when the reasonableness of the product must be proven by scientific and technical standards Mfrs have greater access to that information than the consumer Whether a product is reasonably safe depends on several factors: Must depend on that nature of the product, not the conduct of the Mfr Must consider the following risks and benefits: The usefulness and desireability of the product The safety of the product- the likelihood that it will cause injury The availability of an alternate The ability to use the safer alternate without inhibiting its utlility or significantly increasing the cost The consumers ability to avoid danger with reasonable care when using the product The users awareness of the inherent dangers based on common knowledge, warnings, or instructions The ability of the Mfr to spread the loss through the price There is some evidence to show that Honda could have provided a safer alternative, without adding much to the cost or changing the utility Therefore, SJ is inappropriate Dissent: The risk-benefit analysis is not appropriate either For example, If the benefits of the challenge outweigh the risks inherent in the design Should adopt the consumer expectation test Product is defective if: It is more dangerous than the ordinary consumer would expect when used in an intended or foreseeable manner, or An ordinary consumer is aware that motorcycles are dangerous It was his choice to purchase it The factors test should be used when the consumer cannot be expected to know about the technical info, such as with drugs Notes: Ex of an open and obvious danger: Blind spot of a car Disposable lighters Exception to risk-benefit analysis in food cases Ex: fish chowder Impact on Mfr liability for design defects Social and economic costs Competition with foreign companies

Safety Instructions and Warnings


Issues: Whether any words/symbols at all are needed Whether the warning, when given, was adequate Hood(/injured consumer) v. Ryobi(/Mfr): Products Liability Adequacy of a warning BLL: A mfr may be liable for placing a product on the market that bears inadequate instructions and warnings or that is defective in design Facts: purchased a miter saw in 1995 which had 2 blade guards that shielded the saw the users manual and warnings on the saw stated not to use the saw without the guard in place

the found that the guards prevented him from being able to cut through a piece, so he removed them the saw blades flew out and injured him claims that it was understood that the guards were there to prevent your clothing from getting stuck or putting your hand too close Issue: was the warning adequate? If not, do the benefits of a more adequate warning outweigh the costs? Ruling: YES Was the warning adequate: YES Rule: A mfr may be liable for placing a product on the market that bears inadequate instructions and warnings or that is defective in design Application: The warnings are clear and unequivocal The Mfr should not have to tell of all possible consequences it would be infinite A warning need only be reasonable under the circumstances If the warnings were followed, the injury would have been prevented The majority of consumers do not remove the guard Would the benefits of a more adequate warning outweigh the costs? NO The price of additional warnings is greater than the printing fees Making the label too crowded with detail will make the label less effective Notes: Adequacy: Other cases have developed criteria: For drugs: That the warning: Must indicate the scope of the danger Must indicate the seriousness of the harm that could result from misuse Must have physical aspects that alert a reasonably prudent person Must be conveyed by adequate means For household products: The intensity of the language The prominence in which the language is displayed The adequacy may be in question even if the did not read it Usually held as a question of fact, but sometimes as a matter of law Causation and the heeding presumption: heeding presumption: requiring the to show that the would not have heeded to an adequate warning either courts feel that this acts as a powerful incentive to Mfrs Safety Instructions: Certain uses to be avoided or specific instructions on how to apply a product Ex: flammability of cologne Sometimes the court may find that pictures were needed if the product would be expected to be used by non-English consumers The addressee: Normal rule is that the warning must be designed for a person who is likely to use the product Interplay of design and warnings When should a Mfr make the product less safe and instead just add a warning? Use risk-benefit analysis Need to consider the way the product is marketed The Mfr should not have to design against consumer carelessness When a warning cannot eliminate the possibility of an accident, the product should be more safely designed Lighters: They need not be designed safer just because there is a possibility that a child will use one. They are designed for adults Most consumers do not even read the warnings Cannot just eliminate all these products from the market

Consumers choice: Often consumers have the choice whether to choose a product that is more risky Misuse: When injury results from an unintended use of the product Ex: Drunk driving, but a defect in the car worsens the injury in an accident There is still a defect in the car Marketing a product that has previously been built up in a certain way Ex: cartoon action figure Edwards (/injured patient) v. Basel Pharmaceuticals(/Drug Mfr): Products Liability intermediate warning to the physician (learned intermediary doctrine) BLL: Facts: The used nicotine patches, and smoked, and then dies brought suit for inadequacy to warn of the dangers of overuse a thorough warning was given to the physician, but it was not included in the pamphlet given to the patient claims that the learned intermediary doctrine applies, and the warning required by the FDA was given to the physician and no liability exits the patient relies on the doctor only applies to prescription drugs, not over-the-counter Issue: whether the Mfr owes a warning directly to the consumer Ruling: YES Mfrs have to warn the consumers of possible dangers of their products for a foreseeable danger Exceptions to the learned intermediary doctrine: Mass immunizations There is generally no doctor-patient relationship When the FDA mandates a warning given directly to the consumer When a direct warning to the user of a prescription durg has been mandated through a safety regulation in order to protect the user, an exception to the learned intermediary doctrine exists, and a failure of the Mfr to warn the consumer can render the drug unreasonably dangerous The Mfr is not required to warn of obvious dangers Notes: Should this exception apply to cases where the patient is only trying to better his life, and is already well? Should the patient be informed in differences in success rates? The doctrine does not apply where the Mfr advertises its products directly to the consumer The advertisement must contain the warnings Prescription Drugs: Hard to determine the liability when harm results from something that was not foreseeable Products Rest: Weigh the risks against the benefits of the drug Allergy: Mfr does not have a duty to change the design to prevent allergic reactions when the drug is more beneficial to the general public The Mfr may be liable when there is a significant # allergic Vassallo (/breast implant patient) v. Baxtor Healthcare Corp (/Mfr): Products Liability Failure to warn even though the risk was unknown when the product was sold BLL: A is not liable under the implied warranty of merchantability for failure to warn or provide instructions about risks that were not reasonably foreseeable at the time of sale, or could not have been reasonably discovered through reasonable NPI testing Facts: claims that the silicone breast implants were negligently designed with inadequate warnings, and the breached the implied warranty of merchantability Issue: should the court change the implied warranty of merchantability law (which currently presumes that a Mfr was informed of all the risks associated with the product, regardless of the state of the art at the time of the sale, and amounts to strict liability for failure to warn of these risks) to adopt a state of the art standard that makes a Mfr liable for actual or constructive knowledge of the risks Ruling: Justification for old rule:

That a product that is unreasonably dangerous due to inadequate warnings is not fit for the ordinary purpose for which it is used regardless of the absence of fault on the s part This court is among the minority of states that applies the hindsight rule for the duty to warn The goal is to induce conduct that is capable of being perfomed-warn if they should have known The majority of states follow the Rest. 2nd of Torts 402A, comment j: The seller is required to give warning against a danger if he has knowledge, or by the application of reasonable, developed human skill should have had knowledge of the danger Unforeseeable risks arising from foreseeable product use cannot be warned against A Mfr is required to perform reasonable testing New law in this court: A is not liable under the implied warranty of merchantability for failure to warn or provide instructions about risks that were not reasonably foreseeable at the time of sale, or could not have been reasonably discovered through reasonable NPI testing A mfr will be held liable to the standard of knowledge of an expert in the field, and will remain liable for a continuing duty to warn against defects found after the product is sold Notes: State of the Art: State of the art at a given time is partly determine by the industry invests in safety research The industry custom or practice, or The safest existing technology, or Cutting edge technology Creates incentive for the Mfr to be more involved in safety research The burden of knowing this state of the art is placed on the He has more knowledge in the field One view of Differences between strict liability and negligence : Strict liability: Knowledge of harmful effects will be imputed to the Mfr on a showing that knowledge of the defect existed in the industry Once proof of this knowledge is shown, the must then show that the warning provided was inadequate or missing However, it does not make the Mfr an insurer Thus sometimes courts require that the Mfr had actual or constructive knowledge of the risk (negligence) Mfrs are not liable for risks that are known from tidbits of knowledge, but rather for risks that are recognized in light of the generally recognized and prevailing best knowledge available Negligence: must show that knew or should have known about the potential hazards of the product Discovery of danger after distribution: 10 of the Products Rest: a reasonable seller would warn if the seller knows or reasonably should know that the product poses a substantial risk to persons and property those who would benefit from the warning are identifiable and likely unaware of the risk a warning can effectively be communicated to and acted upon by the recipients, and the risk of harm is sufficiently great to justify the burden of a warning Misrepresentations: Def: Words that affirmatively mislead 9 of the Products Rest: a seller or distributor who makes a fraudulent, negligent, or innocent misrepresentation of fact concerning the product is subject to liability for harm to persons or property caused by the misrepresentation

Defenses
GM (/Car Mfr) v. Sanchez(/crushed by truck): Products Liability Comparative Responsibility BLL: A consumer has no duty to discover or guard against product defect, but a consumers conduct other than a mere failure to discover or guard against a product defect is subject to comparative responsibility Facts:

was reversing, and shifting the truck into park. He had mis-shifted, and the truck went into reverse and pinned him up against a gate No truck design would eliminate the possibility of a mis-dhift Issue: Does the doctrine of comparative responsibility apply to products liability cases? Ruling: YES Change from comparative negligence to comparative responsibility : s damages are reduced by the % of his responsibility % of his responsibility: the % that a party caused or contributed to cause in any way, whether by negligent act or omission, or by other conduct or activity violative of the applicable legal standard the harm for which damages are sought if the does not a breach a duty, this rule does not apply Rest 2nd 402A, comment n: Contributory negligence is not a defense when it is merely a failure to discover a defect or to guard against the possibility of a defect. However, if the negligence consists of voluntarily and unreasonably proceeding to encounter a known danger, and it is found an assumption of risk, it is a defense If the user discovers the defect and proceeds he is barred from recovery Rest 3rd comment d: When the failed to discover a defect, there must be evidence that the s conduct in faling to discover a defect did, in fact, fail to meet a standard of reasonable care Court: A duty to discover defects and to take precautions in constant anticipation that a product might have a defect would defeat the purpose of strict liability Rule: A consumer has no duty to discover or guard against product defect, but a consumers conduct other than a mere failure to discover or guard against a product defect is subject to comparative responsibility Public policy favors reasonable conduct by consumers regardless of whether a product is defective Was the s conduct a mere failure to discover a defect, or was it some other form of negligence unrelated to the product? The read the manual, and he failed to perform the safety measures that would have prevented the accident More is expected of a licensed driver than other persons He had a duty to take reasonable precautions when getting out of his car Notes: Determining the % of fault Must first establish causation, and then the relative degrees of fault Disclaimers and contracts: Products Rest 18: Disclaimers and limitations of remedies do not bar or reduce valid products liability claims 402A, comment m: the consumers cause of action is not affected by any disclaimer or other agreement

Work-Related Injuries
Tort suits by workers against their employers are usually barred, but usually the worker can still sue a third party as though workers comp did not exist Jones (/injured employee) v. Ryobi (/Mfr of printing press): Products Liability modification to the product workers comp BLL: if a product has been modified by a third party in a way that makes it unsafe, the seller of the product is relieved of liability even if the modification is foreseeable Facts: worked on a printing press the press was made and sold by with a plastic guard and an automatic shut-off switch the employer removed the guard and disabled the switch to increase production it was a common practice in the printing industry Issue: does the have a claim for strict liability even though the machine was modified? Ruling: NO In order to claim strict liability for a defective design, the must prove that she was injured as a direct result of a defect that existed when the product was sold The machine was substantially modified, and the modification caused her injury The Mfr is not responsible for the modification, even if it were foreseeable

The press was not unreasonably dangerous when used in the same condition as it was sold Dissent: The ability of the guard to be easily removed invited removal With the guard there, the machine was not effieicnt The obviousness of a defect or danger is a material issue of whether the product was unreasonably dangerous It is not, however, a defense to strict liability under 402A Notes: Liriano (/injured user) v. Hobart(/Mfr): Products Liability the role of a warning for a modification to the product BLL: a , whose design claim is barred because the substantially modifies it, may still maintain a claim for failure to warn of the dangerous consequences of such a modification Facts: 1993: , a 17-year old immigrant, was injured by a meat grinder 9sold in 1961) because the guards had been removed there was no warnings not to remove the guards in 1962, the became aware of issues of removing the guard, and started issuing warnings for customers Issue: whether the , whose design claim is barred because the substantially modifies it, may still maintain a claim for failure to warn of the dangerous consequences of such a modification Ruling: YES A Mfr has a duty to warn against the danger of unintended use if that use is reasonably foreseeable A Mfr is not liable for injuries caused by third party modifications to the product that render it substantially unsafe However, if the product is purposefully manufactured to permit use without the safety feature, a may recover as a result of injuries sustained from using the product without the safety feature The Mfr is in a superior position to anticipate uses of the product However, the duty is not open-ended, and it is measured at the time the product is sold A mfr does not have to deisgn a product that is impossible to misuse Using the risk-benefit analysis, it is more beneficial to warn against these other uses Less costly to implement a warning rather than a design change Cannot possibly determine all the possible uses it would lead to absolute liability Unlike design decisions that involve considerations of many different factors, a duty to warn case focuses more on the consideration of foreseeable risks and adequacy and effectiveness of the warning The Mfr has a duty to warn against foreseeable misuse, and therefore, also against foreseeable alterations. The Mfr has a duty to warn if there is a defect found after the product is sold The Mfr is in a superior position to consider the danger involved, the number of incidents reported, the burden of providing the warning, and the ability to track a product pos-sale No warning required as a matter of law: If the safety device present on the product makes it so obvious that it is dangerous to operate the product without the device, the court may rule as a matter of law that a warning would have been superfluous. If the risk is so open and obvious When the warning would have added nothing to the appreciation of danger Whether more warnings would crowd the label and not fulfill the purpose of the warning However, it is most often a question for the jury Notes: Modification as defective: Other courts have held that a product is defective when the modification is essential to the functionality of the machine, rather than to just increase productivity The machine must fulfill the purpose for which it is sold More depth to a warning It can not only warn to be careful, but it can also show ways to avoid harm Bulk supplier When a company supplied a product in large amounts to a large company who will use the product with many workers Some courts will find an exception to the modification rule The employer as buyer Should the Mfr be able to transfer responsibility for the employee injuries to the employer through negotiations?

Some courts hold that the product is not defective where The buyer is thoroughly knowledgeable about the product and its safety features available There are normal uses where the product is not unreasonably dangerous without the optional equipment The buyer is able to measure the benefits and risks of using the safety equipment Sometimes the employer signs an express agreement assuming the risk of the product The sellers may assert that the assumption of risk should apply to their employees as well Usually not found since the employer is not volunteering to use it without the safety features it is part of his job Some courts think that this makes the Mfr an insurer Workers Comp and Tort: Main issue: When should a tort suit be barred by workers comp statutes? Policy Options: The dominance of WC Policy: Most states deny the 3rd party Mfr any contribution from the negligent employer Policy: the employer has been promised full immunity from fault-based tort liability in return for purchasing no-fault WC benefits Cons: the employer will get away scott-free part of the proceeds from the -Mfr suit may be used to pay for the WC benefits for the employer Pros: Fast in the administrative process The dominance of tort policy: Proportion the burdens among all negligent actors Some states are moving to this Scales agrees with this one Policy: It creates an incentive for all parties to take reasonable measure to prevent accidents The employer will be responsible for injuries due to the workplace, no the defective product Blend of Tort Law and WC Policy: The employer is only responsible to pay up to the amount guranteed by the WC policy This acts as a set-off to the Mfrs liability, and doesnt let the employer get away scot-free Administrative Accommodation of WC and Tort: The Proposed Uniform Products Liability Act: State WC should be altered by eliminating the right of the employer to pay for WC out of the employees tort award Product Liability law should be altered by limiting the amount of the award to the WC benefits

Beyond Products?
Royer ( /knee transplant receiver) v. Catholic Medical Center ( /hospital): BLL: Facts: received a knee transplant, which turned out that the knee replacement was defective had to have another surgery the knee was provided by the claims that the is liable for the defective product because it was unreasonably dangerous claims that it was not a seller of goods for the purpose of strict liability Issue: whether a health care provide that supplies a defective prosthesis in the course of delivering health care services is a seller of prosthesis or is merely providing a service. Ruling: service 402A (Accepted in NH): One is liable for selling a defective good that is unreasonably dangerous if the is a seller engaged in the business of selling that product If the merely provides a service, there is no liability absent proof of a violation of a duty Limits to 402A: Did not apply to contractors or architects, because they are providing a service Other courts have concluded that health care providers are not sellers, and that the provision of a prosthetic device is merely incidental to that service

argue that because the separately charged the for the device, they were sellers the surgeon provided the service, not the court: the critical question is not whether the sold the device to the , but whether the was engaged in the business of selling these devices for the purpose of being held to strict liability without proof of legal fault must look at the relationship between the and the did not enter the hospital to buy a prosthesis, she entered it to obtain treatment was also billed (itemized) for the room, PT, drugs, etc. Strict liability only imposes liability when there is no possibility of negligence s are not claiming that the altered the prosthesis It would place an undue burden on the to have to test all the devices prior to surgery Would increase health care costs Notes: Use of strict liability for medical malpractice Applying strict liability to other services Usually not applicable unless the service is closely tied to a product Ex: cutting your lip on a wine glass in a restaurant strict liability against the restaurant Medical services differ Spreading the risk on injury will only result in higher medical costs The benefit of having healthcare is so important to the general welfare that it outweighs the benefits of imposing strict liability in tort on medical professionals

Tort Law and the UCC


Claims under tort law where the is trying to recover for economic harm Claims under the UCC where the is trying to recover for personal injury East River Steamship Co. ( /charterer of boat) v. Transamerica ( /Mfr of turbines): BLL: A Mfr in a commercial relationship has no duty under either negligence or strict products liability theory to prevent a product from injuring itself Facts: Counts 1-3 The high speed turbine on the first boat malfunctioned, and it was repaired The second 2 boats were inspected, and there were repaired also Count 5 A valve was installed backwards on the 4th ship Steam damaged the turbine It was repaired No damage was done to the rest of the boats or to any person Issue: In an admiralty case, whether tort law is applicable when a defective product is purchased in a commercial transaction, and the malfunction injures only the product itself causing purely economic loss Ruling: NO Admiralty substantive law is applied Purpose of Products Liability Law: Public policy that people need more protection from dangerous products than is afforded by the law of warranty Does Products Liability Law Apply: The law is used when a product that is reasonably certain to place a life and limb in peril is distributed without reinspection, and causes bodily injury This duty of Mfrs was extended to protect property as well for safety reasons Usually, the defective product damages other property. Here, there was no damage to other property, only to the turbines themselves The turbine is one single units since it is supplied by the The can bring this claim as a warranty/contract action to recoup the benefit of the bargain Can an injury to a product itself be brought in tort law? Majority view: Preserving a proper role for the law of warranty precludes using tort law if the defective product causes purely monetary harm

Minority View: Am Mfrs duty to make nondefective products encompasses injury to the product itself whether or not the defective product created an unreasonable risk of harm The safety and insurance rationales are the same where losses are purely economic In Between views: For cases when a product injures only itself, this view attempts to differentiate between the disappointed users and the endangered ones, and permit on the endangered ones to sue in tort. It depends on the nature of the defect, the type of risk, and the injury that arose This court: The minority and in between views are no good, because they do not give mfrs clear direction Traditionally losses that are purely economic are dealt with in contract law The concern for safety is reduced when the injury is only to the product itself The consumer can insure against Cost increases Inconvenience Displeasure of its customers Trying to cover those things through tort law would only affect the public by increasing the costs of products The maintenance of product value is for contract law express and implied warranties Can bring an action for a defective product as a breach of warranty action Easier to deal with in contracts because the parties makes their own agreements In exchange, decreases the costs The expectation damages available in warranty law for purely economic loss give the the full benefit of its bargain by compensating for foregone business A warranty has a built-in limitation on liability, whereas tort law could expose the Mfr to endless liability It would be difficult for the Mfr to account for all users downstream

Notes: Should the mandatory insurance system apply to all product injuries; when there is no physical injury? There is no physical injury here that needs protection from the tort common law Near Miss Doctrine When the damage to the product causes a high risk of injury, but there was actually no injury Can the Mfr be liable in tort for that? Probably not It would turn the contract case into a tort case Unequal bargaining power: Some courts feel that tort law should replace contract law when looking at the violation of a warranty if The product posed a risk of personal injury or property damage, or if The parties were of unequal bargaining power and the purchaser cannot reasonably insure against the consequential damages sometimes courts will use tort law when the party is an individual rather than a company if a person is injured by the defective product, but the defect only causes economic harm to the owner, but personal injuries to others, the people injured may bring a tort claim against the Mfr, but not the owner of the product if only an economic loss When personal injury claims are brought under contract law Implied warranties: May have a longer SOL If the claim is for property damage, the contract does not place such strict limits on recovery Express warranty: If the representation or warranty turns out to be false, anyone hurt because of that may bring an action for damages The product need not be defective There is no shared responsibility under the UCC Just denial of damages

Damages
Compensatory Damages

Seffert (/injured bus rider) v. LA Transit Lines(/Bus Co): Damages Pain and Suffering BLL: An appellate court should not reduce the amount of a jury verdict unless it is so large as to suggest that the jurors were motivated by passion, prejudice, or corruption Facts: was entering on the bus, when the doors were open, and the door closed on her and dragged her is claiming damages for: she suffered severe injuries her painful surgeries she can no longer walk more than 3 blocks she is crippled and will suffer pain for life her hospital bills medical care needed for the rest of her life fear of an amputation lost sleep humiliation, pain and suffering her life expectancy was 34 years jury awarded the following, as directed by s attorney: past and future pecuniary loss: $54K past and future pain and suffering: 134K past medical bills, loss of earning, etc: $18K future medical bills, loss of earning, etc: $25K s claim that the damage award for pain and suffering was excessive trial judge denied a motion for a new trial Issue: whether the damage award for pain and suffering was excessive Ruling: NO Why the trial judge/jury is more capable of determining the damage The trial judge denied a motion for a new trial, and the jury fixed the damages The amount of damages is a fact question, and the jury and the judge used their discretion to reach that amount The trial court sees all the witnesses and evidence The trial judge sits as the 13th juror If he feels that the damages are excessive then it is his responsibility to reduce the damages His denial of a motion for a new trial affirms the jurys decision Why the appellate court is not The appellate court can only interfere with their decision if the damage award was so large such that, at first glance, it shocks the conscience and suggests passion, prejudice, or corruption by the jury There are no fixed or absolute standards by which to determine this While the appellate court may consider past cases of similar injuries, each case must be decided on the specific facts of the case Injuries are seldom identical, and neither is the pain and suffering Why it is not too high according to the appellate court : It does not shock the conscience, even though it is high Dissent: It is so excessive that it does shock the conscience The amount exceeds the amount justified by the evidence She was able to go back to work, and she is already 42 The s attorney improperly mislead the jury Cannot create an exact amount Hard to put a monetary value to a s pain and suffering It would be impossible to ever fully compensate the person Here, there was no restraint and common sense of the jury Usually, the amount given for pain and suffering does no exceed the amount given for other damages Shown in other similar cases where the injury was more serious This is a factor to be considered Just because the did not object, he was denied a new trial because the error would have been sufficient However, that does not preclude an appeal

Notes: Calculating the amount: The chose the formula to show her per diem pain and suffering to justify the final amount The jury is able to focus on the smaller amount, which seems more justified Still just an arbitrary number though The s can argue that the court should look at comparable cases with similar injuries That her pain and suffering will not be the same every day, and will probably lessen over time Would pain and suffering awards disappear if insurance companies were not wiling to insure against it? Would people actually chose to get pain and suffering coverage You cant buy it because its not a demand-people dont want to pay separately Companies just make you pay for it no matter what A lot of states will place caps on it through Legislative acts McDougald (/Comatose Patient) v. Garber (/Physician): Damages Pain and Suffering of a decedent BLL: An ability to experience pain and suffering is a condition precedent to pain an suffering damages, and loss of the pleasures of life is not a separate category of damages Facts: s malpractice left the in a permanent comatose condition Issue: Whether loss of enjoyment of life for a person who is not aware serves a compensatory purpose whether pain and suffering is a separate damage from loss of enjoyment of life Ruling: Whether loss of enjoyment of life for a person who is not aware serves a compensatory purpose - NO Reasoning for damages To compensate the victim, not to punish the wrongdoer Purely punitive damages (non-compensatory) are prohibited unless the tortious act was malicious, willful, or intentional When damages cease to serve as compensatory, they become punitive Awareness An award has no meaning or utility to that person Court recognizes that parties may be tempted to think that a smaller award would be given to someone with more brain damage Do not require the court to determine the level of recognition The real temptation will be to punish the in proportion to the harm However, cannot compare tort law to criminal law Awareness is a prerequisite. Do not require the court to determine the level of recognition It is a good standard for all non-pecuniary losses A more complex instruction would only lead to speculation Whether they are separate claims Has not traditionally been treated separately Reasons to separate: They are distinguishable Pain and suffering subjective, need to be conscious Loss of enjoyment objective, need not be conscious Can manage excessive awards through careful jury instructions Listing the awards would help appellate review Reasons not to separate: Duplicative and excessive awards would result Cannot calculate non-pecuniary awards with precision Cannot translate human suffering into dollars It is impossible to lose the enjoyment of life without experiencing conscious pain and suffering Suffering is included in the inability to enjoy ones life Whether one must be conscious: Damages for the loss of enjoyment of life require no awareness it is purely objective Dissent:

They are separate: They are distinguishable Pain and suffering subjective, need to be conscious Loss of enjoyment objective, need not be conscious The loss of the right to enjoy ones life is comparable to the loss of a limb In contrast, pain and suffering has always been treated as requiring a showing of cognitive capacity Need not be conscious While pain and suffering compensates the victim for the physical and mental discomfort associated with the injury, the loss of enjoyment of life compensates the victim for the limitations placed on his life by the injury Notes: Scales feels that there is a distinction between pain and suffering (negative) and loss of enjoyment of life (positive) But, the court does likely reach the right decision People set up will, benefits, etc. for other people so that they will not have to suffer much Arambula ( /injured) v. Wells ( /caused accident): Damages- Collateral Source Rule BLL: To promote the charitable impulse, we apply the collateral source rule to gratuitous payments
(including moneys to cover lost wages) by family or friends to assist tort victims through difficult times

Facts: Arambula was injured in a rear-end automobile accident caused Wells. Despite missing work because of his injuries, Arambula continued to receive his $2,800 weekly
salary. He testified his brother "wished" to be reimbursed, but he had not promised to do so. Arambula sued Wells for negligence. His claim for damages included loss of earnings during the period of his disability.

His wife sued for loss of consortium.

Wells admitted fault, and the case went to trial on causation and damages alone. Arambula contended he sustained a severe brain injury as a result of the accident. Wells denied this. Based on Helfend, judge instructed the jury not to award damages for lost earnings "because his employer paid for the time he was off without any requirement to do so and there was no agreement by to pay him back.

The jury awarded $54,334 to Arambula, but nothing to his wife. Both appeal Issue: Ruling: Collateral source rule: The liability of the cannot be reduces because the has obtained money from a third party

s in personal injury actions can still recover full damages even though they already have received
compensation for their injuries from such "collateral sources" as medical insurance.

Most commonly from insurance

Reasoning: Tortfeasors should not recover a windfall from the thrift and foresight of persons who have
actually or constructively secured insurance, pension or disability benefits

Inadequacies of damage awards for personal injuries Legal 'compensation' for personal injuries does not actually compensate A substantial attorney's fee usually comes out of award Only covers economic damages, therefore no double recovery for intangibles like pain and suffering

Argument against rule: it misallocates liability for tort-caused losses and discourages people from
obtaining benefits from independent collateral sources.

s argue: Helfend should be used to limit the collateral source rule to situations where plaintiffs incurred an
expense, obligation or liability in obtaining the services for which they seek compensation even though it says nothing about gratuitous wage payments.

Footnote 5 states that the court did not intend for the collateral source rule to apply to gratuitous payments and services

Why the collateral source rule applies to gratuitous donations : There are five specific reasons why footnote five fails to show the thoroughness of the courts analysis in light of its prior expressions on the same subject : 1) There is existing California law (prior to Helfend ), which made no special distinction for purely gratuitous collateral benefits. "The fact that either under contract or gratuitously such treatment has been paid for by another
does not defeat the cause of action of the injured party to recover the reasonable value of such treatment from the tortfeasor." If the Supreme Court decided to overrule its previous decisions, it would have done so through something more than a footnote 2) No other courts have construed it that way

3) Majority of jurisdictions and many commentators are in accord.

Most of the cases do not subtract a gift from tort damages." 4) The rule "expresses a policy judgment in favor of encouraging citizens to purchase and maintain insurance for personal injuries and for other eventualities" Were we to permit a tortfeasor to mitigate damages because of a third party's charitable gift, the plaintiff would be in a worse position than had nothing been done. Why would a family member (or a stranger) freely give of his or her money or time if the wrongdoer would ultimately reap the benefits of such generosity? Helps the state: the state does not have to help them The court should adhere to the intent of the donor The may repay the donor out of similar generosity preventing double recovery Also, a large portion of the award will go to the attorneys We do not decide whether the collateral source rule extends to payments from a public source (ex: free gov benefits). However, collateral source rule applies to future special education benefits by gov because of uncertainty of their continued existence The collateral source rule operates both as a substantive rule of damages and as a rule of evidence . As a rule of evidence
a "persuasive showing" that such evidence is of "substantial probative value" for purposes other than reducing damages.

it precludes the introduction of evidence of the being compensated by a collateral source unless there is
Medical Injury Compensation Reform Act of 1975 (MICRA) abrogated the collateral source rule as a rule of evidence in medical malpractice cases. MICRA allows s to introduce evidence the s had received collateral source benefits, and it prohibits "collateral sources" from obtaining reimbursement from malpractice s It is to the discretion of the trial judge whether it is admissible

Our decision does not automatically bar evidence For example, evidence of collateral source income relevant to show employee had motive other
than physical disability not to work Notes: The collateral source rule has weakened in some states for certain types of damages Some s will argue that the extra from the is for pain and suffering

Punitive Damages
Punitive Damages: Def: Damages exceeding the actual injury suffered for the purpose of punishment, deterrence, and comfort to the Damages that do not seek to compensate; they seek to punish the or to deter others from similar behavior Usually only for intentional unjustified conduct The jury always has total discretion whether to award them Usually the issue is not whether the award was excessive, but was the s conduct purely negligent, or reckless and wanton Taylor v. Superior Court: Punitive damages in tort cases of malice, fraud, or oppression are acceptable

BLL: Driving while intoxicated evinces the malice essential to an award of punitive damages Facts: hit the while under the influence was an alcoholic that had been arrested many times for drunk driving, and he knew of his problem he accepted employment where alcohol was sold, and he was drinking at the time of the accident (in the truck) Issue: Can punitive damages be awarded in tort cases where the knowingly puts others in danger? Ruling: YES 3294 CA Civil Code: authorizes recovery of punitive damages where the is guilty of oppression, fraud, or malice in law or fact( express (evidence of hatred) or implied (inferences of the jury)) malice: an act conceived in the spirit of mischief or with criminal indifference towards others intentional or deliberate with the character of outrage need more than just a tort must be willful or wanton Gombos: Punitive damages can be based upon a finding of a s conscious of the safety of others Justification for punitive damages: To punish the for his poor behavior To deter others from similar behavior Serve as an example Things to consider The severe threat to public safety Since the was extremely dangerous, and drunk driving causes such a high # of accidents, punitive damages are acceptable Dissent: Courts should be hesitant to award punitive damages because 1) is fully compensated through compensatory damages. Any additional $ may be unjust enrichment 2) criminal law should be used to impose punitive damages, not civil law 3) interfere with trial principles is able to bring forth info about the s financial situation 4) a lot of situations would involve only marginal deterrence there may already be a criminal statute to handle it 5) to award punitive damages might nullify insurance coverage an insurer is not liable for loss caused intentionally by the insured 6) conflicts with comparative fault a guilty of willful misconduct may not recover any damages against a negligent Notes: Unidentifiable : Market share liability and other times when the cannot identify the courts normally deny punitive damages Comparative Fault: Generally, in a comparative fault state, the courts reduce the s compensatory fault award from fault, but not the punitive damages Employer liability for punitive damages: In some states, punitive damages flow with vicarious liability Others follow Rest. 2nd 909: Basically where the employer will have to pay punitive damages if it acts intentionally, etc. Liability for punitive damages when the tortfeasor dies : The majority of states deny recovery of punitive damages from the estate of a deceased tortfeasor DPC of the Constitution: The Constitution imposes limits on punitive damages because of the DPC Cannot have excessive damages provide guideposts The Supreme Court said that punitive damages should rarely if ever exceed a 10:1 ratio when compared to compensatory damages

BMW v. Gore: BLL: Due Process requires that a person be given adequate notice that his conduct is subject to punishment and of the severity of that punishment Facts: s car had been repainted after suffering damage from acid rain during transport BMW has a policy that if the cost to repair a new car is less than 3% of the value of the car, they fix it without disclosing the repair Court awarded $4K in compensatory and $2 million in punitive damages Issue: Was the punitive damages award grossly excessive such that it violated the DPC? Ruling: YES State Interests: Punitive damages may be awarded to further the States interests in punishing unlawful conduct and deterring repetition States have flexibility in determining the amount Only when the award is grossly excessive does it violate the DPC Identification of the States interests here State wants to prevent and deter deceptive trade Must take into account The ability of the trade to handle it on their own The cost of regulation Preventing lawsuits over minor issues State to State comparison not allowed: A State may not impose economic sanctions on violators of its laws with the intent of changing conduct in other States Cannot try to induce BMW to change a nationwide policy Can only compare conduct to violators/damages of this States law How to determine if the award is excessive: The person must receive notice that his conduct will subject him to punishment, and what the severity of the punishment will be How to determine if it is grossly excessive 1) the degree of reprehensibility of the s conduct most important factor punitive damages cannot be grossly out of proportion to the compensatory damages Ex: violent crimes are more reprehensible than non-violent crimes, and recurring offenders should be punished more than first time offenders Analysis The harm was purely economic There was no potential of effect on its performance or safety features because of new paint There was no reckless disregard by BMW for the safety of others There was no deliberate false statements, only misrepresentations BMW had a good faith reason to believe that they did not need to disclose this info BMWs conduct was not sufficiently egregious 2) the ratio of actual or potential harm inflicted on the and the punitive damages award there must be a reasonable relationship between the two TXO: Suggested a good ratio was not more than 10:1 Analysis: $2million is 500 times more than $4K there is no claim that there may have been any more potential harm there can be no clear line Ex: a horrendous act with very little economic damage The punitive damage award here exceed reasonableness of the DPC 3) the ratio of the remedy and civil or criminal penalties issued in comparable cases the normal award in the State for deceptive trade practice is $2K. In other States from $5-10K There is no reason to say that a lesser deterrent would not have worked Are any punitive damages acceptable? YES

The State has an interest in protecting its consumers from deceptive trade practice However, it should not be so excessive Notes:

Insurance
Hypo intentionally hit The very first thing the s attorney does during discovery is to look at the s insurance policy complaint intentionally ran him down try to get punitive damages not likely covered by insurance negligently ran him down not really, but the s insurance policy only covers that will likely emphasize this one if the does not have a lot of $ because it is covered by insurance s insurance will want to complain that it was not an accident, since thats what they cover Duty of insurance company 1) indemnity - to pay 2) duty to defend most valuable since it costs so much This provides incentive for the and the to work together : make sure there is insurable events : make sure that the insurance will cover it underlitigation: going for negligence under emphasize the intentional parts of the claim has a health insurer insurer will want part of the award Ex: insurer pays $50K claims $100K right of subrogation one party stands in the shoes of another for the claim the health insurer is subrogated gets parts of the s claim health insurer will assert a lien against the award wants to try to keep the $ and the s insurance company have divergent interests wants to keep it below the cap, and insurer doesnt care either way because it will only pay the max insurance company gets to decide when to settle too usually the cap amount is offered solution B<PL insurer has the obligation to take the settlement when the settlement offer is below the cap if it doesnt, and the jury returns a verdict for higher than the cap, the would be liable for the excess amount (excess judgment) the insurance company may claim that it does not have to defend since it was intentional however, the duty to defend is broader than the duty to indemnify (pay) it must defend weapon for - bad faith settlement when the insurer used bad faith in deciding not to settle the can agree to settle with the for $1 million the can go after the insurance company can say that the agreed to $1 million settlement, and that is owed to the another way the and can cooperate can claim more damages emotional distress and pain and suffering were a large amount

small amount for medical bills if the will not have to split it up, it may cost the less if is happy with his net sum can claim that the settlement was not enough claiming that he was not made whole subrogation fails problems subrogation agreements says nothing about the becoming whole or getting to go first how do you figure out what the right value would be? Cant retry the damages however, sometimes courts do

Tax Get the award tax-free Under 104(a) exclusion s insurance gets an equivalent deduction for a business expense can invest the $ the proceeds are taxable gets too much in the beginning however, if the gave money at a later date, it would not be worth as much the correct method depends on what you do with the $ and what your tax situation is most awards are given in lump sums structured settlement pay over time, but it vests tax-free can fund it by buying an annuity 130 allows him to buy an annuity without paying taxes on it now, the can pay less for the annuity because it wont be taxed might actually be more for the Often the will not know how it works out, just what the end result is Factoring Companies Will turn a structured settlement into a cash settlement Charging very high interest rates Insurance companies object to this

S-ar putea să vă placă și